You are on page 1of 111

Anatomy

Question 8: Anatomy - Cortex and Connections

Discussion:

The most prominent efferent pathway from the amygdaloid nuclear complex is the stria terminalis. Most of the fibers arise from
the corticomedial part of the nuclear complex. They arch along the medial border of the caudate near its junction with the
thalamus. The largest termination is the nucleus of the stria terminalis. This is located lateral to the columns of the fornix and
dorsal to the anterior commissure. Other terminations include the anterior hypothalamic nuclei and the medial preoptic area.

References:

Nolte J. The human brain. 4th ed. St Louis: Mosby, 1999.

Benarroch EE, Westmoreland BF, Daube JR, et al. Medical neurosciences--an approach to anatomy, pathology and physiology
by systems and levels. 4th ed. New York: Lippincott, Williams & Wilkins, 1999.

Question 19: Anatomy - Brainstem/Cerebellum

Discussion:

The largest cerebellar efferent bundle, the superior cerebellar peduncle, is formed by fibers from the dentate, emboliform, and
globose nuclei. Most fibers enter and surround the contralateral red nucleus.

References:

Carpenter MB. Core text of neuroanatomy. Baltimore: William & Wilkins, 1991.

Question 21: Anatomy - Peripheral Nervous System

Discussion:

The posterior cord of the brachial plexus gives off the thoracodorsal and subscapular nerves and terminates by splitting into the
axillary and radial nerves. Any muscles innervated by these branches may be weakened with a lesion in the posterior cord of the
plexus.

References:

Campbell WW. DeJong's the neurologic examination. Philadelphia: Lippincott, Williams & Wilkins, 2005.

Question 45: Anatomy - Spinal Cord

Discussion:

The rubrospinal tract descends in the lateral portions of the spinal cord, close to the lateral corticospinal tract. The
vestibulospinal, tectospinal, and reticulospinal tracts and the medial longitudinal fasciculus descend in the anterior portions of the
spinal cord.

References:

Carpenter MB. Core text of neuroanatomy. Baltimore: William & Wilkins, 1991.

Question 49: Anatomy - Cortex and Connections


Discussion:

Gerstmann's syndrome includes agraphia, finger agnosia, right-left disorientation, and acalculia. When all four features are
present, the lesion is most often in the dominant (usually left) inferior parietal lobule which includes the supramarginal and
angular gyri.

References:

Carpenter MB. Core text of neuroanatomy. Baltimore: William & Wilkins, 1991.

Blumenfeld H. Neuroanatomy through clinical cases. 1st ed. Sunderland: Sinauer Associates, 2002.

Question 56: Anatomy - Basal Ganglia and Thalamus

Discussion:

The amygdalofugal fibers pass through the inferior thalamic peduncle to the magnocellular division of the mediodorsal nucleus.

References:

Benarroch EE, Westmoreland BF, Daube JR, et al. Medical neurosciences--an approach to anatomy, pathology and physiology
by systems and levels. 4th ed. New York: Lippincott, Williams & Wilkins, 1999.

Nolte J. The human brain. 4th ed. St Louis: Mosby, 1999.

Question 79: Anatomy - Peripheral Nervous System

Discussion:

The long thoracic nerve innervates the serratus anterior muscle. Winging of the scapula results when the patient pushes against a
wall, especially with the shoulder adducted. There may also be inability to abduct the arm beyond 90 degrees.

References:

Brazis PW, Masdeu JC, Biller J. Localization in clinical neurology. 5th ed. Philadelphia: Lippincott, Williams & Wilkins, 2006.

Question 92: Anatomy - Spinal Cord

Discussion:

The pattern of bilateral upper extremity weakness affecting distal muscles more than proximal ones with decreased pinprick
sensation with retained fine touch, ie, a dissociated sensory level in a "cape-like" distribution is most suggestive of a central cord
syndrome. A central cord lesion affecting the anterior commissure will affect the crossing spinothalamic tracts resulting in
sensory deficits only at the level of the lesion, without causing more caudal sensory deficits, and will spare vibration and light
touch since the dorsal columns are spared.

References:

Brazis PW, Masdeu JC, Biller J. Localization in clinical neurology. 5th ed. Philadelphia: Lippincott, Williams & Wilkins, 2006.

Campbell WW. DeJong's the neurologic examination. Philadelphia: Lippincott, Williams & Wilkins, 2005.

Question 107: Anatomy - Basal Ganglia and Thalamus


Discussion:

The thalamus is supplied mainly by the branches of the posterior cerebral, posterior communicating, and posterior choroidal
arteries.

References:

Campbell WW. DeJong's the neurologic examination. Philadelphia: Lippincott, Williams & Wilkins, 2005.

Brazis PW, Masdeu JC, Biller J. Localization in clinical neurology. 5th ed. Philadelphia: Lippincott, Williams & Wilkins, 2006.

Question 109: Anatomy - Cranial Nerves

Discussion:

The cochlear nerve originates from the spiral ganglion, enters the brainstem at the cerebellopontine angle and terminates on the
dorsal and ventral cochlear nuclei. The pattern of tonotopic localization is evident in both the nerve fibers and their termination in
the cochlear nuclei. The vestibular (Scarpa) ganglion gives rise to the vestibular portion of cranial Nerve VIII. The geniculate
ganglion contains special visceral afferent cell bodies for the 7th nerve. The pterygopalatine ganglion contains synapses for the
parasympathetic component of the 7th nerve.

References:

Carpenter MB. Core text of neuroanatomy. Baltimore: William & Wilkins, 1991.

Campbell WW. DeJong's the neurologic examination. Philadelphia: Lippincott, Williams & Wilkins, 2005.

Question 121: Anatomy - Cranial Nerves

Discussion:

The cranial preganglionic outflow arises from visceral cell groups of the brain stem associated with the oculomotor, facial,
glossopharyngeal and vagus nerves. All of these are parasympathetic.

References:

Campbell WW. DeJong's the neurologic examination. Philadelphia: Lippincott, Williams & Wilkins, 2005.

Question 129: Anatomy - Spinal Cord

Discussion:

The posterior spinocerebellar tract arises from the dorsal nucleus of Clarke. The fibers enter the cerebellum via the inferior
cerebellar peduncle. None of the impulses conveyed by this tract reaches conscious levels.

References:

Carpenter MB. Core text of neuroanatomy. Baltimore: William & Wilkins, 1991.

Question 147: Anatomy - Basal Ganglia and Thalamus

Discussion:
Axonal fibers arising from the subthalamic nucleus terminate on the globus pallidum, internal segment and the substantia nigra,
pars reticulata. Neurons of the subthalamus contain glutamate and exert excitatory input into the globus pallidum, pars internal.
This connection is one component of the indirect loop that decreases thalamocortical excitation.

References:

Carpenter MB. Core text of neuroanatomy. Baltimore: William & Wilkins, 1991.

Campbell WW. DeJong's the neurologic examination. Philadelphia: Lippincott, Williams & Wilkins, 2005.

Gilman S, Newman SW. Essentials of clinical neuroanatomy and neurophysiology. 10th ed. Philadelphia: FA Davis, 2003.

Question 159: Anatomy - Blood Supply of Brain/Spinal Cord

Discussion:

The superior temporal gyrus is supplied by the middle cerebral artery. The posterior cerebral artery supplies the posterior
hippocampus as well as the infero-medial aspects of the temporal lobe. The anterior-medial aspect of the temporal lobe is
supplied by the anterior choroidal artery.

References:

Carpenter MB. Core text of neuroanatomy. Baltimore: William & Wilkins, 1991.

Question 165: Anatomy - Blood Supply of Brain/Spinal Cord

Discussion:

The lateral inferior or caudal pontine syndrome due to occlusion of the anterior inferior cerebellar artery (AICA syndrome)
involves lesions in the fascicles of cranial nerve VII, the spinal tract, and nucleus of cranial nerve V, the lateral spinal thalamic
tract, descending sympathetic fibers (lateral reticular nucleus), the middle cerebellar peduncle, the inferior surface of the
cerebellum, and, in addition, the inner ear and cochlear nerve due to occlusion of the labyrinthine artery, a common branch of the
AICA. Clinical findings include ipsilateral ataxia, loss of pain and temperature sensation of the face, Horner syndrome, deafness,
and contralateral pain and temperature loss of the limbs.

References:

Campbell WW. DeJong's the neurologic examination. Philadelphia: Lippincott, Williams & Wilkins, 2005.

Question 166: Anatomy - Basal Ganglia and Thalamus

Discussion:

The posterior hypothalamus is the "sympathetic area". A lesion in this area would produce an ipsilateral Horner syndrome.

References:

Blumenfeld H. Neuroanatomy through clinical cases. 1st ed. Sunderland: Sinauer Associates, 2002.

Brazis PW, Masdeu JC, Biller J. Localization in clinical neurology. 5th ed. Philadelphia: Lippincott, Williams & Wilkins, 2006.

Question 173: Anatomy - Blood Supply of Brain/Spinal Cord

Discussion:
The first major branch of the internal carotid artery is the ophthalmic artery. The ICA does not have any branches extracranially.

References:

Carpenter MB. Core text of neuroanatomy. Baltimore: William & Wilkins, 1991.

Haines DE. Fundamental neuroscience. 2nd ed. New York: WB Saunders, 2002.

Question 176: Anatomy - Blood Supply of Brain/Spinal Cord

Discussion:

The anterior choroidal artery arises from the internal carotid artery distal to the origin of the posterior communicating artery. It
has a long subarachnoid course, enters the inferior horn of the lateral ventricle through the choroidal fissure, and supplies the
amygdala, hippocampal formation, globus pallidus, and the ventrolateral portion of the posterior limb and the entire
retrolenticular portion of the internal capsule.

References:

Parent A. Carpenter's human neuroanatomy. 9th ed. Baltimore: Williams & Wilkins, 1996.

Carpenter MB. Core text of neuroanatomy. Baltimore: William & Wilkins, 1991.

Question 194: Anatomy - Brainstem/Cerebellum

Discussion:

Afferent mossy fibers to the cerebellum synapse on the dendrites of granule cells in the granule layer. Axons from the granule
cell travel into the molecular layer as the parallel fibers and synapse on the Purkinje cell dendrites.

References:

Carpenter MB. Core text of neuroanatomy. Baltimore: William & Wilkins, 1991.

Question 203: Anatomy - Cranial Nerves

Discussion:

The abducens nerve (VI) is contained in the sheath of the internal carotid artery at the level of the cavernous sinus. The
oculomotor (III), trochlear (IV) and ophthalmic (V-1) nerves are located in the lateral wall of the cavernous sinus. The optic
nerve occupies the optic foramen but not the cavernous sinus.

References:

Parent A. Carpenter's human neuroanatomy. 9th ed. Baltimore: Williams & Wilkins, 1996.

Questions 214 - 218: Anatomy - Peripheral Nervous System

Discussion:

The ventral divisions of the L2-4 anterior primary rami give rise to the obturator nerve, whereas the dorsal divisions give rise to
the femoral nerve. After all motor branches from the femoral nerve are given off in the thigh; the nerve continues to descend into
the lower leg as the saphenous nerve, supplying cutaneous sensation to the medial leg region. Three cutaneous nerves are derived
from the L1 dermatome: genitofemoral, ilioinguinal, and iliohypogastric nerves. The iliacus (femoral nerve), psoas (direct
branches from Lumbar plexus), and tensor fascia lata (superior gluteal nerve) all flex the hip. Only the TFL additionally
internally rotates the thigh, and has an attachment to the iliotibial band. Sciatic mononeuropathies can clinically imitate common
peroneal mononeuropathies because the peroneal portion of the sciatic tends to be predominantly affected due to fascicular
vulnerability. An isolated common peroneal mononeuropathy will not cause hypo- or areflexia of the Achilles deep tendon reflex
(DTR).

References:

Aldo O. Perotto. Anatomical Guide for the electromyographer. 3rd ed. Springfield, Illinois: Charles C Thomas Publisher, 1994.

Campbell WW. DeJong's the neurologic examination. Philadelphia: Lippincott, Williams & Wilkins, 2005.

Question 246: Anatomy - Brainstem/Cerebellum

Discussion:

The T2-weighted MR image was an axial cut at the superior collicular level of the midbrain. The red nucleus, round and
hypointense on T2, gives rise to the central tegmental tract.

References:

Carpenter MB. Core text of neuroanatomy. Baltimore: William & Wilkins, 1991.

Question 258: Anatomy - Cortex and Connections

Discussion:

The structure indicated by the arrow is the anterior commissure, which connects the temporal lobes and also contains fibers from
the amygdala and the olfactory system.

References:

Nolte J. The human brain. 4th ed. St Louis: Mosby, 1999.

Blumenfeld H. Neuroanatomy through clinical cases. 1st ed. Sunderland: Sinauer Associates, 2002.

Haines DE. Neuroanatomy: an atlas of structures, sections and systems. 2nd ed. Baltimore: Urban & Schwarzenberg, 1987.

Question 291: Anatomy - Basal Ganglia and Thalamus

Discussion:

The marked structure is the lateral geniculate nucleus which receives visual input from the optic tract.

References:

Haines DE. Neuroanatomy: an atlas of structures, sections and systems. 2nd ed. Baltimore: Urban & Schwarzenberg, 1987.

Nolte J. The human brain. 4th ed. St Louis: Mosby, 1999.

Question 311: Anatomy - Blood Supply of Brain/Spinal Cord

Discussion:
The arrow is pointing to the vein of Labbe, which courses across the temporal lobe to connect the superficial middle cerebral vein
with the transverse sinus.

References:

Nolte J. The human brain. 4th ed. St Louis: Mosby, 1999.

Haines DE. Neuroanatomy: an atlas of structures, sections and systems. 2nd ed. Baltimore: Urban & Schwarzenberg, 1987.

Question 314: Anatomy - Cranial Nerves

Discussion:

The structure indicated by the arrow is the trochlear (fourth) nerve which completely decussates before it exits from the dorsal
aspect of the brainstem. The trochlear nerve innervates the superior oblique muscle and paralysis of this muscle results in
extorsion of the ipsilateral eye.

References:

Haines DE. Neuroanatomy: an atlas of structures, sections and systems. 2nd ed. Baltimore: Urban & Schwarzenberg, 1987.

Wilson-Pauwels L, Akesson E, Stewart P. Cranial nerves: anatomy and clinical components. Toronto: BC Decker Inc, 1988.

Question 325: Anatomy - Brainstem/Cerebellum

Discussion:

The structure indicated by the arrow is the chief sensory nucleus for the trigeminal nerve. The motor nucleus of the trigeminal
lies immediately medial to the chief sensory nucleus while the spinal nucleus of the trigeminal is located caudally

References:

Nolte J. The human brain. 4th ed. St Louis: Mosby, 1999.

Haines DE. Neuroanatomy: an atlas of structures, sections and systems. 2nd ed. Baltimore: Urban & Schwarzenberg, 1987.

Question 334: Anatomy - Spinal Cord

Discussion:

The tract indicated by the arrow is the anterior corticospinal tract which carries fibers arising from the ipsilateral precentral gyrus.
It is the uncrossed component of the pyramidal tract.

References:

Haines DE. Neuroanatomy: an atlas of structures, sections and systems. 2nd ed. Baltimore: Urban & Schwarzenberg, 1987.

Blumenfeld H. Neuroanatomy through clinical cases. 1st ed. Sunderland: Sinauer Associates, 2002.

Question 348: Anatomy - Cortex and Connections

Discussion:
The inferior frontal gyrus is divided into the pars orbitalis, pars triangularis, and the pars opercularis. The pars opercularis and
triangularis of the dominant hemisphere contain the motor (Broca) speech area. The frontal eye field occupies principally the
caudal part of the middle frontal gyrus.

References:

Campbell WW. DeJong's the neurologic examination. Philadelphia: Lippincott, Williams & Wilkins, 2005.

Carpenter MB. Core text of neuroanatomy. Baltimore: William & Wilkins, 1991.

Question 352: Anatomy - Cortex and Connections

Discussion:

Fifty patients with elevations of serum cardiac troponin levels had strokes involving the right posterior, superior medial insula,
and the right inferior parietal lobule. Among patients with right middle cerebral artery strokes, the insular cortex was involved in
88% of patients with elevated serum cardiac troponin but in only 33% of patients without the elevation.

References:

Ay H, Koroshetz WJ, Benner T, et al. Neuroanatomic correlates of stroke-related myocardial injury. Neurology 2006; 66: 13256.

Question 359: Anatomy - Cortex and Connections

Discussion:

On the medial aspect of the occipital lobe, the calcarine fissure separates the cuneus superiorly and the lingual gyrus (medial
occipitotemporal gyrus) inferiorly. The calcarine cortex (primary visual cortex or striate cortex), is comprised of these two
structures. The cuneus receives visual impulses from the upper quadrant of the ipsilateral side of both retinas. The lingual gyrus
receives impulses from the lower quadrants of the retina.

References:

Campbell WW. DeJong's the neurologic examination. Philadelphia: Lippincott, Williams & Wilkins, 2005.

Gilman S, Newman SW. Essentials of clinical neuroanatomy and neurophysiology. 10th ed. Philadelphia: FA Davis, 2003.

Question 364: Anatomy - Basal Ganglia and Thalamus

Discussion:

The dorsomedial nucleus connects prefrontal, limbic, and olfactory structures with prefrontal cortex. The intralaminar nuclei
project to the cerebral cortex and the basal ganglia. The lateral dorsal nucleus receives input from the hippocampus and projects
to the cingulate gyrus. The pulvinar is an association nucleus that receives inputs from parietal, temporal, and occipital cortex and
then projects to these same areas. The reticular nucleus projects to other thalamic nuclei but not to the cortex.

References:

Nolte J. The human brain. 4th ed. St Louis: Mosby, 1999.

Carpenter MB. Core text of neuroanatomy. Baltimore: William & Wilkins, 1991.

Question 370: Anatomy - Cortex and Connections


Discussion:

Asomatognosia is a category of neglect in which the patient denies ownership of a limb contralateral to a lesion of the
supramarginal gyrus of the parietal lobe (usually nondominant).

References:

Brazis PW, Masdeu JC, Biller J. Localization in clinical neurology. 4th ed. Philadelphia: Lippincott, Williams & Wilkins, 2001.

Question 372: Anatomy - Cranial Nerves

Discussion:

The facial nerve (CN VII) gives off three branches prior to entering the stylomastoid foramen. The first branch is the greater
superficial petrosal nerve, the second is the nerve to the stapedius, and the third is the chorda tympani.

References:

Campbell WW. DeJong's the neurologic examination. Philadelphia: Lippincott, Williams & Wilkins, 2005.

Carpenter MB. Core text of neuroanatomy. Baltimore: William & Wilkins, 1991.

Question 376: Anatomy - Cortex and Connections

Discussion:

The dentate gyrus (DG) is contiguous with the hippocampal formation (HF); both are three-layered cortex (archipallium). The
cortical zones from the DG and HF, through the prosubiculum, subiculum, presubiculum, and parahippocampal gyrus show a
gradual transition from three-layer to six-layer cortex. The entorhinal region (area 28) is six-layered cortex.

References:

Carpenter MB. Core text of neuroanatomy. Baltimore: William & Wilkins, 1991.

Gilman S, Newman SW. Essentials of clinical neuroanatomy and neurophysiology. 10th ed. Philadelphia: FA Davis, 2003.

Question 388: Anatomy - Peripheral Nervous System

Discussion:

Although they are part of the sympathetic nervous system, the postganglionic fibers to the sweat glands (sudomotor fibers) are
cholinergic.

References:

Haines DE. Fundamental neuroscience. 2nd ed. New York: WB Saunders, 2002.

Question 398: Anatomy - Spinal Cord

Discussion:

Of the listed tracts, only the dorsal columns are somatotopically organized such that information from the lower extremities is
laminated medial to the information from the upper extremities (the fasciculus gracilis is medial to the fasciculus cuneatus).
References:

Nolte J. The human brain. 4th ed. St Louis: Mosby, 1999.

Blumenfeld H. Neuroanatomy through clinical cases. 1st ed. Sunderland: Sinauer Associates, 2002.

Question 416: Anatomy - Cortex and Connections

Discussion:

The dorsolateral prefrontal cortex mediates executive cognitive function. Fibers from this region project to the dorsolateral head
of the caudate nucleus. From the caudate, fibers project via direct and indirect loops that will synapse on the dorsomedial nucleus
of the thalamus, which then relay fibers back to the prefrontal cortical region.

References:

Watts RL, Koller WC. Movement disorders. 2nd ed. New York: McGraw-Hill, 2004.

Mega MS, Cummings JL. Frontal-subcortical circuits and neuropsychiatric disorders. J Neuropsychiatry Clin Neurosci
1994;6:358-370.

Question 420: Anatomy - Brainstem/Cerebellum

Discussion:

Decerebrate posturing is due to dysfunction of the red nucleus and the rubrospinal tract with preserved function of the lateral
vestibular nucleus and the lateral vestibular spinal tracts. The rubrospinal tract facilitates motor neurons in the cervical cord
subserving flexor muscles of the upper extremities. The lateral vestibulospinal tract facilitates motor neurons in the lumbosacral
cord subserving extensor muscles of the lower extremities.

References:

Carpenter MB. Core text of neuroanatomy. Baltimore: William & Wilkins, 1991.

Campbell WW. DeJong's the neurologic examination. Philadelphia: Lippincott, Williams & Wilkins, 2005.

Question 428: Anatomy - Embryology

Discussion:

The prosencephalon divides into the diencephalon and the telencephalon. The diencephalon gives rise to the thalamus and
hypothalamus. The telencephalon gives rise to the striatum and cerebral cortex.

References:

Parent A. Carpenter's human neuroanatomy. 9th ed. Baltimore: Williams & Wilkins, 1996.

Question 439: Anatomy - Basal Ganglia and Thalamus

Discussion:

Of the structures mentioned, only the median forebrain bundle passes through the hypothalamus. The external capsule lies
between the claustrum and the putamen, the internal capsule is bordered by the lentiform nucleus (putamen and globus pallidus)
laterally and the caudate and the thalamus medially, the mamillothalamic tract connects the mamillary bodies with the anterior
thalamic nucleus, and the uncinate fasciculus connects the amygdala with the orbitofrontal cortex.

References:

Blumenfeld H. Neuroanatomy through clinical cases. 1st ed. Sunderland: Sinauer Associates, 2002.

Behavioral/Psychiatry

Question 1: Behavioral/Psychiatry - Language/Speech Abnormalities

Discussion:

This woman demonstrates a case of expressive aprosodia where she has a deficit in expressing the musical intonation of speech
leaving her monotone with limited facial expressions required to express anger, sadness, joy, surprise, or inquisitiveness. This
deficit is located in the analogous region of the Broca cortex in the nondominant frontal lobe.

De Clerambault syndrome is a delusional belief usually in older women that an older, more influential male is in love with her
despite evidence to the contrary. Confabulation is where incorrect answers are given in response to questions regarding
unremembered information. Involuntary emotional expression disorder (IEED) is an emotional disinhibition syndrome
characterized by crying or laughing episodes that are unrelated to or out of proportion to the eliciting stimulus.

References:

Cummings JL, Mega MS. Neuropsychiatry and behavioral neuroscience. New York: Oxford University Press, 2003.

Mesulam MM. Principles of behavioral and cognitive neurology. 2nd ed. New York: Oxford University Press, 2000.

Cummings JL, Arciniegas DB, Brooks BR, et al. Defining and diagnosing involuntary emotional expression disorder. CNS
Spectr 2006;11:1-7.

Question 2: Behavioral/Psychiatry - Frontal Systems Syndromes

Discussion:

This woman demonstrates a case of expressive aprosodia where she has a deficit in expressing the musical intonation of speech,
leaving her monotone with limited facial expressions required to express anger, sadness, joy, surprise, or inquisitiveness. This
deficit is located in the analogous region of the Broca cortex in the nondominant frontal lobe.

References:

Cummings JL, Mega MS. Neuropsychiatry and behavioral neuroscience. New York: Oxford University Press, 2003.

Question 3: Behavioral/Psychiatry - General Psychiatry

Discussion:

This woman has obsessive-compulsive disorder, and fluorodeoxyglucose PET consistently shows hypermetabolic activity in the
caudate, anterior cingulate, and orbitofrontal cortex.

References:

Baxter LR, Phelps ME, Mazziotta JC, et al. Local cerebral glucose metabolic rates in obsessive-compulsive disorder: a
comparison with rates in unipolar depression and normal controls. Arch Gen Psychiatry 1987;44:211-218.
Question 11: Behavioral/Psychiatry - Language/Speech Abnormalities

Discussion:

In clinical practice disorders of language and articulation must be differentiated. Patients with aphasia have variable degrees of
awareness of their deficit and have impairment in other general language functions to include comprehension, naming, reading,
etc). Disorders of articulation can be further broken down into dysarthrias and apraxia of speech. Patients with dysarthria
typically have impairment in resonance and errors are generally consistent and predictable. In apraxia of speech, the articulatory
errors are variable and repeated attempts to correctly pronounce words impair the normal flow and prosody of speech.

References:

Ogar J, Willock S, Baldo J, et al.. Clinical and anatomical correlates of apraxia of speech. Brain Lang 2006;97:343-350

Question 17: Behavioral/Psychiatry - General Psychiatry

Discussion:

Somatization disorder: A history of many physical complaints beginning before age 30 years that occur over a period of several
years and result in treatment being sought or significant impairment in social, occupational, or other important areas of
functioning. These individuals must have a history of pain related to at least four different sites or functions (eg, head, abdomen,
back, joints, extremities, chest, rectum, during menstruation, during sexual intercourse, or during urination); a history of at least
two gastrointestinal symptoms other than pain (eg, nausea, bloating, vomiting other than during pregnancy, diarrhea, or
intolerance of several different foods); a history of at least one sexual or reproductive symptom other than pain (eg, sexual
indifference, erectile or ejaculatory dysfunction, irregular menses, excessive
Discussion:

While valproic acid, clonazepam, and olanzapine can help control aggressive behavior, topiramate is the best choice. Topiramate
is renally excreted and has a side effect of decreasing appetite which would be beneficial in this patient who has had excessive
weight gain.

References:

Klein KM, Theisen F, Knake S, et al. Topiramate, nutrition, and weight change: a prospective study. J Neurol Neurosurg
Psychiatr 2008;79:590-593.

Question 80: Behavioral/Psychiatry - Developmental Disorders

Discussion:

Attention deficit hyperactivity disorder (ADHD) is a highly heritable, disruptive, childhood-onset condition, the etiology and
pathogenesis of which is poorly understood. There have been relatively few genome-wide linkage studies, and no chromosomal
region has yet been unequivocally implicated. In contrast, evidence from pharmacological, neuroimaging, and animal studies has
suggested the involvement of specific neurotransmitter systems, notably dopaminergic pathways. Meta-analyses or pooled data
analyses have supported association between ADHD and polymorphisms in DRD4, DRD5, and SLC6A3, which encode
dopamine D4 and D5 receptors and the dopamine transporter, respectively.

References:

Waldman ID, Gizer IR. The genetics of attention deficit hyperactivity disorder. Clin Psychol Rev 2006;26:396-432.

Question 84: Behavioral/Psychiatry - Dementia

Discussion:

Transactivating responsive sequence DNA-binding protein (TDP-43) is a protein encoded by the TARDBP gene on chromosome
1. Numerous studies have linked this protein to the syndromes of amyotrophic lateral sclerosis, frontotemporal dementia with
motor neuron disease and frontotemporal dementia with ubiquitin inclusions. Microtubule associated protein tau and valosin-
containing protein have also been found in some cases of frontotemporal dementia, however rarely in those also exhibiting motor
neuron disease.

References:

Geser F, Martinez-Lage M, Trojanowshi JQ. Amyotrophic lateral sclerosis, frontotemporal dementia and beyond: the TDP-43
diseases. J Neurol 2009;256:1205-1214.

St George-Hyslop, Peter. Genetics of Dementia. Continuum 2008;14:29-48

Question 87: Behavioral/Psychiatry - Occipital Syndromes

Discussion:

Anton syndrome or denial of blindness may result from retention of visual input to the parietal cortex by way of the superior
colliculus and pulvinar. It is often called "blind sight". Elementary visual hallucinations may result from primary visual cortex
lesions. Release hallucinosis may occur in the hemianopic field. It is not thought that the lateral geniculate nucleus can bring
visual input to consciousness. The optic radiations do not project to either parietal or temporal cortex.

References:

Milner AD and Goodale MA. The visual brain in action. New York: Oxford University Press, 1995
Question 89: Behavioral/Psychiatry - Neurobiology of Behavior

Discussion:

Various brain regions have been implicated in the processing of several different emotions. There is growing evidence that the
insula is involved in the perception of disgust and the recognition of facial and vocal expressions of disgust.

References:

Sambataro F, Dimalta S, Di Giorgio, et al.. Preferential responses in amygdala and insula during presentation of facial contempt
and disgust. Eur J Neurosci 2006;24:2355-2362.

Berle D, Phillips ES. Disgust and obsessive-compulsive disorder: an update. Psychiatry 2006;69:228-238.

Question 97: Behavioral/Psychiatry - Neurobiology of Behavior

Discussion:

Incoming sensory signals arriving at the sensory association cortices are simultaneously presented to the hippocampus and the
amygdala. The hippocampus helps to recall specific facts of the sensory experience and links them to past events. Affective
components are attached by the amygdala where recruitment of endocrine and autonomic responses are elicited.

References:

Question 103: Behavioral/Psychiatry - Behavioral Complications of Systemic Disease

Discussion:

This patient is presenting with the clinical syndrome of Hashimoto encephalitis. In this condition TSH and T3/T4 levels are
frequently normal; however, antithyroglobulin (antimicrosomal antibody) antibodies are typically elevated. This condition should
be in the differential of any patient with a rapidly progressive dementia especially in the presence of seizures. Hashimoto
encephalitis responds well to intravenous corticosteroid treatment and plasmapheresis.

References:

Schiess N, Pardo CA. Hashimoto's encephalopathy. Ann NY Acad Sci 2008;1142:254-265.

Question 117: Behavioral/Psychiatry - Behavioral Complications of Systemic Disease

Discussion:

Mercury toxicity can occur from paper or thermostat manufacturing, ingestion of contaminated seafood or from breathing
metallic mercury vapor. Organic mercury intoxication results in a dementia associated with depression and hallucinations. On an
elemental neurologic exam, one may find visual field deficits, blindness, deafness, dysarthria, choreoathetosis, and ataxia.
Sometimes a syndrome mimicking amyotrophic lateral sclerosis may be seen.

References:

Mendez MF, Cummings JL. Dementia: a clinical approach. 3rd ed. Philadelphia: Butterworth-Heinemann, 2003.

Question 120: Behavioral/Psychiatry - Temporal-Limbic Syndromes

Discussion:
Capgras delusions and psychosis in general are typically thought to originate in the temporal lobe. Lesions in the occipital-
parietal, posterior cingulate, and the superior parietal lobule are all unlikely to cause Capgras delusions. Occasionally frontal
pathology makes it difficult to resolve conflicting information which may result in delusional ideas. However, a tumor in the left
dorsolateral frontal lobe causing a seizure would typically cause speech arrest due to involvement of the Broca area.

References:

Fried I. Auras and experiential responses arising in the temporal lobe. J Neuropsychiatry Clin Neurosci 1997;9:420-428.

Question 140: Behavioral/Psychiatry - Dementia

Discussion:

Donepezil is approved for mild to severe probable Alzheimer disease. Rivastigmine and galantamine are approved only for mild
to moderate probable Alzheimer disease. All patients with probable Alzheimer disease should be offered treatment with
cholinesterase inhibitors. Definite Alzheimer disease is only diagnosed after death and so treatment is not indicated.
Cholinesterase has been used off label for mild cognitive impairment, vascular dementia and dementia with Lewy bodies but
does not have FDA approval.

References:

Wilkinson D, Doody R, Helme R, et al. Donepezil in vascular dementia: a randomized, placebo-controlled study. Neurology
2003;61:479-486.

McKeith I, Del Ser T, Spano P, et al. Efficacy of rivastigmine in dementia with Lewy bodies: a randomized, double-blind,
placebo-controlled international study. Lancet 2000;356:2031-2036.

Birks J, Flicker L. Donepezil for mild cognitive impairment. Cochrane Database Syst Rev. 2006 Jul 19;3:CD006104.

Question 148: Behavioral/Psychiatry - Neurobiology of Behavior

Discussion:

This clinical condition is prosopagnosia, or failure visually to identify familiar faces (or classes of objects such as cars) despite
good visual acuity. Auditory recognition remains intact. The fusiform gyrus located in the occipitotemporal cortex is usually
impaired.

References:

Cummings JL, Mega MS. Neuropsychiatry and behavioral neuroscience. New York: Oxford University Press, 2003.

Question 153: Behavioral/Psychiatry - Frontal Systems Syndromes

Discussion:

The orbitofrontal syndrome is characterized by increased risk-taking behavior, impulsivity, changes in emotion, and personal
regulation. The circuit begins in the orbitofrontal cortex and then projects to ventromedial head of the caudate. Projections then
go to the substantia nigra, thalamus, and finally back to the cortex.

References:

Miller BL, Cummings JL, editors. The human frontal lobes: functions and disorders. New York: The Guilford Press, 1999.

Question 155: Behavioral/Psychiatry - Language/Speech Abnormalities


Discussion:

Speech remains intact as language deteriorates with advancing Alzheimer dementia, eventually producing an aphasia in which
the patient is fluent and paraphasic, their speech is empty, and they have limited comprehension but repeat well, which is typical
of transcortical sensory aphasia.

References:

Cummings JL, Darkins A, Mendez M, et al. Alzheimer's disease and Parkinson's disease: comparison of speech and language
alterations. Arch Neurol 1988;38:680-684.

Question 157: Behavioral/Psychiatry - Behavioral Complications of Systemic Disease

Discussion:

Anti-Ma2 typically from testicular cancer can cause limbic, brainstem encephalitis. Anti-Ri associated with breast, gynecologic,
lung, and bladder cancers can cause ataxia with or without opsoclonus/myoclonus. Anti-VGCC associated with small-cell lung
cancers can cause Lambert-Eaton myasthenic syndrome. Anti-VGKC associated with thymoma, prostate and small-cell lung
cancer can cause seizures, vertigo, peripheral neuropathy, and neuromyotonia. Anti-MAG associated with Waldenstrom
macroglobulinemia can cause peripheral neuropathy.

References:

Darnell RB, Posner JB. Paraneoplastic syndromes involving the nervous system. New Engl J Med 2003;349:1543-1554.

Foster AR, Caplan JP. Paraneoplastic limbic encephalitis. Psychosomatics 2009;50:108-113.

Question 164: Behavioral/Psychiatry - Dementia

Discussion:

Semantic dementia is a subtype of frontotemporal lobar degeneration. Patients early on have fluent spontaneous speech but have
increasing difficulty understanding nouns and recognizing objects. Some patients will also develop visual agnosia as well as
prosopagnosia. With time, patients usually demonstrate symptoms typical of frontotemporal dementia.

References:

van der Zee J, Sleegers K, Van Brockhoven C. Invited article: the Alzheimer disease-frontotemporal lobar degeneration
spectrum. Neurology 2008;71:1191-1197.

Kertesz A. Frontotemporal dementia: a topical review. Cogn Behav Neurol 2008;21:127-133.

Question 167: Behavioral/Psychiatry - Behavioral Complications of Systemic Disease

Discussion:

Creutzfeldt-Jakob disease is characterized by a rapidly progressive dementia condition which has both cortical and subcortical
involvement. West Nile encephalitis can be severe and present with headache, high fever, neck stiffness, stupor (sleepiness),
disorientation, coma, tremors, occasional convulsions, paralysis, and rarely death. Multifocal cortical and subcortical features are
not usually present. Huntington disease is a slowly progressive condition with chorea and dementia. Herpes simplex encephalitis
typically involves temporal/insular or frontal regions on one side. Hallervorden-Spatz disease is a rare disorder characterized by
progressive extrapyramidal dysfunction including rigidity, dystonia, choreoathetosis, tremors and dementia. Onset is most
commonly in late childhood or early adolescence.

References:
Geschwind MD, Shu H, Hasman A, et al. Clinical update: rapidly progressive dementia. Ann Neurol 2008;64:97-108.

Scharre DW. Metabolic, infectious and other dementia conditions. American Academy of Neurology Annual Meeting: non-
Alzheimer dementia course. Seattle, Washington; May 1, 2009.

Question 170: Behavioral/Psychiatry - Dementia

Discussion:

Numerous studies have found a serotonergic deficit in patients with frontotemporal dementia (FTD). Experts in the field will
often treat these patients with selective serotonin reuptake inhibitors (SSRIs) even in the absence of depression. There is no
evidence of a cholinergic deficit in FTD, and studies evaluating the efficacy of cholinesterase inhibitors have been largely neutral
or negative. Purported neuroprotective and antioxidant compounds have also not been found to be beneficial.

References:

Graff-Radford N, Woodruff B. Frontotemporal dementia. Continuum 2004;10:63-65.

Litvan I. Therapy and management of frontal lobe dementia patients. Neurol 2001;56(Suppl 4):S41-S45.

Question 174: Behavioral/Psychiatry - Behavioral Complications of Systemic Disease

Discussion:

Wernicke-Korsakoff syndrome, related to thiamine deficiency, produces an amnestic disorder with abnormalities limited to
memory and learning rather than a dementia syndrome. The syndrome occurs most commonly among alcoholics with thiamine-
deficient diets, but it may be seen in other patients who have malnutrition or gastrointestinal absorption disorders. Alcoholic
dementia and Wernicke-Korsakoff syndrome commonly coexist. Paraneoplastic syndromes include limbic encephalitis where
individuals can develop prominent memory, behavioral, and mood disturbances; however, ophthalmoplegia and gait ataxia are
not typically present. The classic symptoms of botulism include double vision, blurred vision, drooping eyelids, slurred speech,
difficulty swallowing, dry mouth, and muscle weakness. Symptoms generally begin 18 to 36 hours after eating a contaminated
food and mental status is usually normal. Marchiafava-Bignami Disease is seen most often in alcoholics and results in corpus
callosum damage. It may present with sudden onset of stupor or coma and seizures. Other patients have acute, subacute, or
chronic onset of dementia, gait problems, psychiatric disturbances, incontinence, hemiparesis, aphasia, and apraxia of the left
hand (due to callosal disconnection). Ophthalmoplegia is not common and there is no specific treatment which provides partial
recovery. Myasthenia gravis may cause ophthalmoplegia or even ataxia but it does not result in delirium.

References:

Scharre DW. Metabolic, infectious and other dementia conditions. American Academy of Neurology Annual Meeting: non-
Alzheimer dementia course. Seattle, Washington; May 1, 2009.

Kopelman MD. The Korsakoff syndrome. Br J Psychiatry 1995;166:154-173

Question 195: Behavioral/Psychiatry - Neurobehavioral/Neuropsychological Exam

Discussion:

Apraxia refers to a disorder of learned movement that cannot be explained by deficits of weakness, sensory loss, or attention.
There are two principal types of apraxia: (1) ideational, in which the patient cannot correctly pantomime a multistep command
and (2) ideomotor, in which the patient cannot perform on command simple actions such as waving good-bye or using a hammer.

References:

Koski L, Iacoboni M, Mazziotta JC. Deconstructing apraxia: understanding disorders of intentional movement after stroke. Curr
Opin Neurol 2002;15:71-77.
Question 196: Behavioral/Psychiatry - Behavioral Complications of Systemic Disease

Discussion:

Toluene toxicity is often the result of "recreational" abuse by sniffing and inhaling fumes from spray paint cans. Patients who
chronically inhale toluene vapors develop dementia (clinically consistent with subcortical dementia), cerebellar ataxia and long
tract findings, and in some cases cranial nerve palsies. It is not uncommon for these patients to also manifest a paranoid
psychosis. MRI demonstrates a diffuse leukoencephalopathy, cerebral atrophy, and T2 hypointense lesions of the thalamus and/or
the basal ganglia.

References:

Filley CM. The behavioral neurology of white matter. New York: Oxford University Press, 2001.

Question 201: Behavioral/Psychiatry - General Psychiatry

Discussion:

Delusional disorder is differentiated from schizophrenia by the lack of bizarre delusions, absence of prominent auditory/visual
hallucinations, and the absence of significant impairment in psychosocial functioning. The diagnosis of schizoaffective disorder
requires the presence of either a depressive, manic or mixed episode during the illness.

References:

American Psychiatric Association. Diagnostic and statistical manual of mental disorders (DSM IV-TR). 4th ed. Washington DC:
American Psychiatric Publishing, Inc.; 2000.

Question 204: Behavioral/Psychiatry - Dementia

Discussion:

Most patients with Alzheimer disease (AD) have a sporadic late-onset form of the disease. A small percentage of patients,
however, have familial disease produced by one of three autosomal dominant genes. Among familial AD patients, 50% to 70%
have the presenilin-1 mutation or an associated mutation on chromosome 14; 5% to 10% have the presenilin-2 mutation. A small
percentage will have a mutation in the gene that codes for amyloid precursor protein.

References:

Bird TD. Genetic factors in Alzheimer s disease. N Eng J Med 2005;352:862-864.

Question 205: Behavioral/Psychiatry - Developmental Disorders

Discussion:

Asperger syndrome is a pervasive developmental disorder (PDD) which is characterized by individuals with persistent
impairment in social interaction, stereotypies, repetitive behavior patterns, and restricted interests. Childhood disintegrative
disorder (CDD) is also classified as a pervasive developmental disorder (PDD) and is characterized by late onset (>2 years of
age) of developmental delays in language, social function, and motor skills. Asperger syndrome differs from childhood
disintegrative disorder by its relative preservation of linguistic and cognitive development. Both show no delay in acquiring
initial language skills but after age 2, those with CDD regress in their language and communications skills. Both of these
developmental disorders display impairment in social functioning skills and show repetitive behavior and restricted interests.

References:

McPartland J, Klin A. Asperger's syndrome. Adolesc Med Clin 2006;17:771 788.


Filipek PA, Accardo PJ, Ashwal S. Practice parameter: screening and diagnosis of autism: report of the Quality Standards
Subcommittee of the American Academy of Neurology and the Child Neurology Society. Neurology 2000;55:468-479.

Volkmar F, Rutter M. Childhood disintegrative disorder: results of the DSM-IV autism field trial. J Am Acad Child Adolesc
Psychiatry 1995;34:1092-1095.

Question 206: Behavioral/Psychiatry - Psychopharmacology

Discussion:

Olanzapine is an atypical antipsychotic that frequently causes significant weight gain. Quetiapine, risperidone, haloperidol, and
molindone are less likely to do so.

References:

Puzantian T, Stimmel G. Review of psychotropic drugs. New York: McMahon, 2001.

Question 347: Behavioral/Psychiatry - Neurobehavioral/Neuropsychological Exam

Discussion:

Semantic memory is best described as the memory for factual knowledge that is not related to any specific memory. Recall of this
information is typically not associated with recollection of "how" and "when" the concept was learned.

References:

Mesulam MM. Principles of behavioral and cognitive neurology. 2nd ed. New York: Oxford University Press, 2000.

Budson A. Understanding memory dysfunction. Neurologist 2009;15:71-79.

Question 349: Behavioral/Psychiatry - Parietal Syndromes

Discussion:

Several behavioral responses have been observed in patients following right hemisphere lesions. These include anosodiaphoria
(indifference to a deficit), autotopagnosia (inability to localize stimuli accurately on the affected side of the body), anosognosia
(denial of a deficit), misoplegia (hatred of the paralyzed limbs), and somatoparaphrenia (denial of ownership of the paralyzed
limbs).

References:

Heilman KM, Valenstein E. Clinical neuropsychology. New York: Oxford University Press, 2003.

Cummings JL, Mega MS. Neuropsychiatry and behavioral neuroscience. New York: Oxford University Press, 2003.

Question 363: Behavioral/Psychiatry - Frontal Systems Syndromes

Discussion:

An anterior callosal lesion disconnects the left hand (right motor strip) from the verbal left hemisphere, yielding an apraxia to
verbal commands confined to the left hand (alien hand syndrome).

References:
Cummings JL, Mega MS. Neuropsychiatry and behavioral neuroscience. New York: Oxford University Press, 2003.

Question 365: Behavioral/Psychiatry - Dementia

Discussion:

This is a case of normal pressure hydrocephalus where you typically see hydrocephalus on neuroimaging, subcortical dementia,
incontinence, and apraxia of gait. Patients with subcortical dementia classically have retrieval type memory deficits and have
prominent affective disturbances as well.

References:

Friedland, RP. Normal pressure hydrocephalus and the saga of the treatable dementias. JAMA 1989;262:2577-2593.

Question 368: Behavioral/Psychiatry - Language/Speech Abnormalities

Discussion:

The transcortical aphasias frequently result from damage to the extrasylvian language areas. These infarcts represent watershed
infarcts secondary to high-grade stenosis, hypotension and hypoperfusion. Occasionally, they can be secondary to
thromboembolic disease. The feature that differentiates all of the transcortical aphasias from the classic aphasias is the retained
ability to repeat.

References:

Benson DF, Ardila A. Aphasia: a clinical perspective. New York: Oxford University Press, 1996.

Question 371: Behavioral/Psychiatry - General Psychiatry

Discussion:

The greatest loss of brain tissue in schizophrenic patients is in the hippocampus, amygdala and the parahippocampal gyrus
regions. The fusiform gyrus is in the temporal lobe, but caudal to the hippocampal formation.

References:

Bertelsen A. Schizophrenia and related disorders: experience with current diagnostic systems. Psychopathology 2002;35:89-93.

Velakoulis D, Pantelis C, McGorry PD, et al. Hippocampal volume in first-episode psychoses and chronic schizophrenia: a high-
resolution magnetic resonance imaging study. Arch Gen Psychiatry 1999;56:133-140.

Question 373: Behavioral/Psychiatry - Behavioral Complications of Systemic Disease

Discussion:

HIV infection can result in minor cognitive and motor disorder, HIV-associated mild neurocognitive disorder, and HIV-
associated dementia. The earliest symptoms revolve around mental slowing and processing speed. Tests that assess processing
speed, including Trails A and B, grooved pegboard, Symbol Digit Modalities Test, and the HIV Dementia Scale are likely to be
abnormal early in the disease course.

References:

Mendez MF, Cummings JL. Dementia: a clinical approach. 3rd ed. Philadelphia: Butterworth-Heinemann, 2003.
Question 380: Behavioral/Psychiatry - Neurobehavioral/Neuropsychological Exam

Discussion:

The Wisconsin Card Sort Test (WCST), which challenges a patient to change cognitive sets without warning, is particularly
sensitive to frontal damage. Language skills may be unaffected, and vocabulary is often spared. Face recognition and visual
perception abnormalities such as hemi-inattention syndromes are most often associated with damage to the parietal, temporal, or
occipital lobes, rather than the frontal lobes. Therefore, the best answer is the WCST.

References:

Kimberg DY, D'Esposito M, Farah MJ. Frontal lobes: cognitive neuropsychological aspects. In: Feinberg TE, Farah MJ, editors.
Behavioral neurology and neuropsychology. New York: McGraw-Hill, 1997.

Question 381: Behavioral/Psychiatry - Psychopharmacology

Discussion:

Mirtazapine often results in weight gain. It is the antidepressant of choice for patients with anorexia. Mirtazapine is associated
with weight gain both in the short- and long-term. Patients taking mirtazapine often report a voracious appetite, with intense
cravings for carbohydrates.

References:

Laimer M, Kramer-Reinstadler K, Rauchenzauner M, et al. Effect of mirtazapine treatment on body composition and
metabolism. J Clin Psychiatry 2006;67:421-424.

Question 385: Behavioral/Psychiatry - Behavioral Complications of Systemic Disease

Discussion:

Psychosis (delusions and hallucinations) is common in vitamin B12 deficiency. Hypothyroidism often causes memory loss. A left
middle cerebral artery stroke usually causes aphasia and hemiparesis. Frontal meningiomas may cause anosmia, disinhibition, or
apathy. Psychosis is usually not associated with attention-deficit hyperactivity disorder (ADHD).

References:

Herr K, Norris E, Frankel B. Acute psychosis in a patient with vitamin B12 deficiency and coincident cervical stenosis.
Psychosomatics 2002;43:234-236.

Question 392: Behavioral/Psychiatry - Behavioral Complications of Systemic Disease

Discussion:

Wilson disease is an autosomal recessive disorder caused by a mutation in the ATP7B gene on chromosome 13. Clinical
symptoms are neurological, psychiatric, hepatic, or ocular. Other disorders affecting basal ganglia circuitry can mimic Wilson
disease; however, only Wilson disease will manifest with copper abnormalities on laboratory screens. These syndromes include
pantothenate kinase disease (caused by a mutation in the PANK2 gene) and Huntington disease (trinucleotide repeat disorder on
chromosome 4).

References:

Kitzberger R, Madl C, Ferenci P. Wilson disease. Metab Brain Dis 2005;20:295-302.


Online Mendelian Inheritance in Man (OMIM) TM [homepage on the Internet]. Baltimore: McKusick-Nathans Institute for
Genetic Medicine, Johns Hopkins University; Bethesda: National Center for Biotechnology Information, National Library of
Medicine; c2006 [cited 2006 Aug 4;]. Available from: www.ncbi.nlm.nih.gov/omim/.

Question 395: Behavioral/Psychiatry - Language/Speech Abnormalities

Discussion:

Deep dyslexia most commonly results from large lesions involving the perisylvian region with extension to the frontal lobes. The
hallmark feature is semantic errors. Patients also demonstrate a "part of speech" effect where nouns and high imageability words
are read better and modifiers which in turn are read better than verbs and functors and low imageability words. Patients struggle
to read non-sense words and make lexicalization errors ("phone" for "phope") when attempting to do so.

References:

Coslett H. Acquired dyslexia. Semin Neurol 2000;20:419-426.

Bub D. Alexia and related reading disorders. Neuro Clin N Am 2003;21:549-568.

Question 401: Behavioral/Psychiatry - Behavioral Complications of Systemic Disease

Discussion:

Paroxysmal autonomic instability with dystonia (PAID) is a common symptom cluster similar to malignant hyperthermia and
neuroleptic malignant syndrome. It commonly appears following severe traumatic or hypoxic brain injury. Treatment generally
consists of beta-adrenergic blockers, opioid analgesia, dopamine agonists, and benzodiazepines. Dopamine antagonists can
precipitate symptoms similar to PAID. Drugs acting on cholinergic and serotonin systems have not been found to be effective.

References:

Blackman JA, Patrick PD, Buck ML, Rust RS. Paroxysmal autonomic instability with dystonia after brain injury. Arch Neurol
2004;61:321-328.

Question 406: Behavioral/Psychiatry - Psychopharmacology

Discussion:

The serotonin syndrome results from concomitant administration of medications that enhance serotonin transmission via
decreased breakdown or increased production. Medication combinations to use cautiously include monoamine oxidase inhibitor
agents with selective serotonin reuptake inhibitors, tricyclic antidepressants, or dextromethorphan. The serotonin syndrome can
be differentiated from neuroleptic malignant syndrome by the presence of shivering and myoclonus in the former.

References:

Christensen RC. Identifying serotonin syndrome in the emergency department. Am J Emerg Med 2005;23:406-408.

Boyer EW, Shannon M. Current concepts: the serotonin syndrome. New Eng J Med 2005;352:1112-1120.

Question 410: Behavioral/Psychiatry - Dementia

Discussion:

The angular gyrus syndrome results from an infarct or mass lesion affecting the angular gyrus on the dominant side. The
syndrome consists of difficulty with naming, reading, writing, memory and contains the elements of a Gerstmann syndrome.
Patients with semantic dementia lose conceptual knowledge of memories and words. They tend to have frequent pauses in their
speech and can present with prosopagnosia. Patients with posterior cortical atrophy present with progressive deficits that affect
the dorsal and ventral streams of vision. They tend to have preserved insight and lack the characteristic memory deficits which
are pathognomonic for Alzheimer disease.

References:

McMonagle P, Deering F, Berliner Y, Kertesz A. The cognitive profile of posterior cortical atrophy. Neurology 2006;66:331-
338.

Question 419: Behavioral/Psychiatry - Parietal Syndromes

Discussion:

Gerstmann syndrome is classically described as a tetrad of findings (finger agnosia, right-left confusion, acalculia, and agraphia)
localized to the dominant hemisphere. In clinical practice, it is common to see only two-to-three components of this syndrome
following a lesion in the supramarginal gyrus.

References:

Mayer E, Martory M, Pegna AJ, et al. A pure case of Gerstmann syndrome with a subangular lesion. Brain 1999;122:1107-1120.

Clinical Adult

Question 6: Clinical Adult - Demyelinating Disease

Discussion:

This patient most likely has neuromyelitis optica (Devic disease). The disease in most cases limits itself to the optic nerves and
the spinal cord. This disorder is characterized by the presence of antibodies to the aquaporin-4 protein in the CSF and serum.

References:

Cree B. Neuromyelitis optica: diagnosis, pathogenesis, and treatment. Curr Neurol Neurosci Rep 2008;8:427-433.

Question 12: Clinical Adult - Neuromuscular Disorders

Discussion:

This patient most likely has wound botulism from injection of contaminated black tar heroin. Botulism is characterized by
descending paralysis starting in the extra-ocular and bulbar muscles with pupil dilatation and other evidence of autonomic
involvement. None of the other disorders listed would be associated with pupillary dilatation. Black widow spider envenomation
is associated with pain and paresthesia, abdominal cramping and nausea without weakness.

References:

Werner SB, Passaro D, McGee J, et al. Wound botulism in California, 1951-1998: recent epidemic in heroin injectors. Clin Infect
Dis 2000;31:1018-1024.

Question 13: Clinical Adult - Other Pain Syndromes

Discussion:
This patient has glossopharyngeal neuralgia, possibly from nerve infiltration by a skull base tumor. This disorder is characterized
by neuropathic pain in the sensory distribution of CN IX (pharynx and external auditory meatus). Syncope may be associated and
presumably results from reflex bradycardia due to stimulation of the tractus solitarius.

References:

Erman AB, Kejner AE, Hogikyan ND, Feldman EL. Disorders of cranial nerves IX and X. Semin Neurol 2009;29:85-92.

Question 25: Clinical Adult - Epilepsy

Discussion:

This patient is having gustatory hallucinations, usually arising from a lesion of the parietal operculum.

References:

Brazis PW, Masdeu JC, Biller J. Localization in clinical neurology. 5th ed. Philadelphia: Lippincott, Williams & Wilkins, 2006.

Question 26: Clinical Adult - Epilepsy

Discussion:

This patient with frequent partial-complex seizures, most likely from a unilateral mesial temporal focus, has continued seizures
despite what appears to have been adequate trials of two anticonvulsants. Patients such as this are unlikely to respond adequately
to a trial of a third anticonvulsant. Referral to an epilepsy center for video-EEG monitoring, as part of an assessment for his
candidacy for seizure surgery (eg, temporal lobectomy) is most appropriate.

References:

Thadani VM, Taylor J. Surgical treatment for epilepsy. Continuum: Lifelong Learning in Neurology 2007;13:152-176.

Question 27: Clinical Adult - Dementia

Discussion:

The clinical scenario of nonfluent primary progressive aphasia followed gradually by alterations of personal grooming behavior
and rigidity in a relatively young patient suggests frontotemporal dementia (FTD). While defective processing of more than one
protein may be found in FTD, the most commonly reported genetic mutations are tauopathies.

References:

Graff-Radford N, Woodruff B. Frontotemporal dementia. In: Petersen RC, editor. Dementia. Continuum: Lifelong Learning in
Neurology 2004;10:64-73.

Question 29: Clinical Adult - Epilepsy

Discussion:

Juvenile myoclonic epilepsy is the most common form of idiopathic epilepsy and typically presents in adolescence or early
adulthood. Patients typically present with generalized seizures precipitated by alcohol or sleep deprivation. Myoclonic jerks and
bursts of 4 to 6 Hz spike-and-wave discharges are characteristic. Valproate is very effective at treating both the seizures and
myoclonus.

References:
Ropper AH, Brown RH. Adams and Victor's principles of neurology. 8th ed. New York: McGraw-Hill, 2005.

Question 34: Clinical Adult - Neurotoxicology

Discussion:

Nitrous oxide toxicity may cause a progressive myelopathy of the posterior and lateral columns that mimics that of vitamin B12
deficiency. Nitrous oxide irreversibly oxidizes the cobalt moiety of methylcobalamin, interfering with methylation reaction,
including transformation of homocysteine to methionine.

References:

Hadzic A, Glab K, Sanborn KV, Thys DM. Severe neurologic deficit after nitrous oxide anesthesia. Anesthesiology 1995;83:863-
866.

Question 39: Clinical Adult - Headache

Discussion:

This patient has a history of migraine, a straightforward current history, and a normal neurologic examination. No features
suggest elevated ICP or meningeal irritation. No testing is necessary in this setting.

References:

Noseworthy J. Neurological therapeutics: principles and practice. New York: Taylor & Francis, 2006.

Question 41: Clinical Adult - Cerebrovascular Disease

Discussion:

This patient has suffered a transient ischemic event most likely related to either intracranial or extracranial atheromatous disease.
There is no evidence to suggest a cardiac embolic source. The most appropriate therapy at this time is an antiplatelet agent. There
is no advantage of clopidogrel over aspirin. Aspirin is less expensive. There is no role in this case for either anticoagulation or
tissue plasminogen activator (tPA).

References:

Noseworthy J. Neurological therapeutics: principles and practice. New York: Taylor & Francis, 2006.

Question 51: Clinical Adult - Epilepsy

Discussion:

Stroke is the most common underlying cause of new seizures in the elderly. Brain tumors are the most common cause in the 4th
and 5th decade and trauma is the most common cause in the 2nd and 3rd decades.

References:

Engel J, Pedley TA, editors. Epilepsy: a comprehensive textbook. Philadelphia: Lippincott-Raven, 1998.

Question 65: Clinical Adult - Epilepsy

Discussion:
Antiepileptic drugs (AEDs) are known to cross the placenta and may present a risk for major congenital malformations (MCM)
when used during pregnancy. Valproate, phenobarbital, carbamazepine and phenytoin are all associated with MCM. Valproate, in
particular, should be avoided during the first trimester and is associated with a higher risk of congenital malformations when used
in polytherapy and at higher doses.

References:

Harden CL, Hopp J, Ting TY, et al. Management issues for women with epilepsy-focus on pregnancy (an evidence-based
review): I. Obstetrical complications and change in seizure frequency: Report of the Quality Standards Subcommittee and
Therapeutics and Technology Assessment Subcommittee of the American Academy of Neurology and the American Epilepsy
Society. Epilepsia 2009;50:1229-1236.

Question 70: Clinical Adult - Dementia

Discussion:

Although the diagnosis of normal pressure hydrocephalus (NPH) can be problematic, some studies have stratified both positive
and negative predictors of outcome from shunting. Patients whose initial presentation is gait disorder have a better chance of
successful outcome (if other confounding diagnoses, such as parkinsonism, are excluded), while those with dementia of greater
than 2 years' duration, dementia preceding the gait disorder, or other factors which are more consistent with Alzheimer dementia
(aphasia and loss of hippocampal volume) predict a poorer outcome from shunting. Presence of urinary incontinence is not
predictive of improvement from shunting.

References:

Graff-Radford N. Dementia: normal pressure hydrocephalus. Continuum: lifelong learning in neurology 2007;13:146-148.

Question 85: Clinical Adult - Motor Neuron/Nerve

Discussion:

This patient has peroneal compression neuropathy due to significant weight loss and leg crossing. The appropriate management
would be to caution him against leg crossing.

References:

Rubin DI, Kimmel DW, Cascino TL. Outcome of peroneal neuropathies in patients with systemic malignant disease. Cancer
1998;83:1602-1606.

Question 86: Clinical Adult - Spinal and Root Disorders

Discussion:

Saddle anesthesia, sphincter loss, and loss of ankle reflexes after a fall signify midline disk herniation with compression of the
cauda equina.

References:

Bradley WG, Daroff RB, Fenichel GM, et al, editors. Neurology in clinical practice. 3rd ed. New York: Butterworth-Heinemann,
1999.

Question 88: Clinical Adult - Spinal and Root Disorders

Discussion:
Metastatic epidural spinal cord compression presents most commonly with pain and is associated with weakness, bowel and
bladder incontinence and sensory loss that can be localized to the spinal cord. Emergent treatment in patients with evidence of
myelopathy should be instituted with high dose dexamethasone in order to maintain and preserve ambulation.

References:

Schiff D, O'Neill BP. Principles of neuro-oncology. 1st ed. New York: McGraw Hill, 2005.

Question 91: Clinical Adult - Neuromuscular Disorders

Discussion:

This patient most likely has ciguatera poisoning from ingestion of a large reef fish (grouper or barracuda). The toxin is produced
by a marine microorganism which passes up the food chain and is concentrated in the flesh of large reef fish. Ciguatera poisoning
is characterized by nausea, vomiting, and diarrhea within 8 hours of ingestion. Over the next 18 to 24 hours, painful paresthesias
and myalgias develop in the extremities and periorally. Many patients note paradoxic sensation. Ciguatoxin is believed to open
sodium channels in the peripheral nerve membrane causing spontaneous depolarization. The disorder is self-limited and therapy
is not required or available.

References:

Pearn J. Neurology of ciguatera. J Neurol Neurosurg Psychiatry 2001;70:4-8.

Question 95: Clinical Adult - Headache

Discussion:

This patient has idiopathic intracranial hypertension (pseudotumor cerebri). The most concerning sequela of this condition is
visual loss from compression of the optic nerves. The best long-term therapy is weight loss although acetazolamide can provide
relief of symptoms until the weight loss can be achieved. Optic nerve decompression and VP shunting are reserved for refractory
cases or those who are experiencing precipitous visual loss.

References:

Noseworthy J. Neurological therapeutics: principles and practice. New York: Taylor & Francis, 2006.

Question 96: Clinical Adult - Dementia

Discussion:

The patient has many clinical features consistent with a frontal dementia. The trails B test is a measure of executive function. It
assesses sustained attention, visual scanning, and the ability to change sets (inhibit certain stimuli while facilitating others). It is a
timed task, so speed of performance is also important. Finger tapping assesses motor speed. The Wechsler Memory Scale and
California Verbal Learning Test assess memory. The Boston Naming Test evaluates language.

References:

Bartleson JD, Schierman B, Edlund W, editors. Summaries of AAN clinical practice guidelines for adult neurology:
neuropsychological testing. 2006-2007 ed. St. Paul, MN: American Academy of Neurology.

Feinberg TE, Farah MJ. Behavioral neurology and neuropsychology. 2nd ed. New York: McGraw-Hill, 2003.

Question 101: Clinical Adult - Cerebrovascular Disease

Discussion:
The syndrome characterized by ipsilateral Horner syndrome and neck pain with contralateral hemiparesis is most suggestive of a
carotid dissection. Dissections often occur in the setting of neck trauma and disorders of connective tissue including
fibromuscular dysplasia.

References:

Schievink WI. Spontaneous dissection of the carotid and vertebral arteries. N Engl J Med 2001;344:898-906.

Question 102: Clinical Adult - Neuro-oncology

Discussion:

Signs and symptoms of elevated intracranial pressure in conjunction with focal neurologic deficits, in a patient with known
malignancy should raise suspicion of intracranial metastases. Brain metastases occur in as many as 20% of patients with cancer,
whereas paraneoplastic disease occurs in less than 1% of patients. The other disorders listed would also be less likely in this
setting.

References:

Lassman AB, DeAngelis LM. Brain metastases. Neurol Clin 2003;21:1-23,vii.

Question 104: Clinical Adult - Headache

Discussion:

The patient described has cluster headache and is being considered for prophylaxis of this condition. Ergotamine is an abortive
treatment. Atenolol has little CNS penetration and is not recommended for headache prophylaxis; imipramine is more appropriate
for migraine prophylaxis. Verapamil, a calcium channel blocker, is currently the preferred agent for prophylaxis of cluster
headache.

References:

Bradley WG, Daroff RB, Fenichel GM, et al, editors. Neurology in clinical practice. 3rd ed. Boston: Butterworth-Heinemann,
2000.

Question 105: Clinical Adult - Motor Neuron/Nerve

Discussion:

Almost 90% of patients with Guillain-Barr頳yndrome demonstrate elevation of spinal fluid protein with no leukocytosis at time
of presentation.

References:

Mendell JR, Kissel JT, Cornblasth DR. Diagnosis and management of peripheral nerve disorders. 1st ed. New York: Oxford
University Press, 2001.

Question 111: Clinical Adult - Spinal and Root Disorders

Discussion:

Copper deficiency can produce a clinical syndrome very similar to subacute combined degeneration. Clinical settings in which
this occurs include malabsorption (such as from gastric bypass as in this patient) or as a consequence of excess zinc ingestion.
References:

Kumar N, Gross JB, Ahlskog JE. Copper deficiency myelopathy produces a clinical picture like subacute combined degeneration.
Neurology 2004;63:33-39.

Question 113: Clinical Adult - Demyelinating Disease

Discussion:

A clinically isolated syndrome is the term used to describe an initial clinical neurologic event suggestive of demyelination.
Patients such as this one with a clinically isolated syndrome and MRI evidence of dissemination in space are at high risk for
experiencing subsequent demyelinating attacks (dissemination in time) and therefore ultimately fulfilling diagnostic criteria for
the diagnosis of multiple sclerosis. The McDonald criteria allow for either a second clinical attack or a new MRI lesion on
follow-up scans to demonstrate dissemination in time to diagnose multiple sclerosis.

References:

Rinker JR, Cross AH. Diagnosis and differential diagnosis of multiple sclerosis. Continuum: lifelong learning in neurology
2007;13(5):13-34.

Question 118: Clinical Adult - Movement Disorders

Discussion:

A variety of impulse control behaviors, including gambling and other compulsive behaviors, have been reported in patients with
Parkinson disease apparently related to dopamine agonist therapy. Patients should be informed of these behaviors prior to starting
agonist therapy and should be monitored for this side effect during follow-up visits.

References:

Voon V, Potenza MN, Thomsen T. Medication-related impulse control and repetitive behaviors in Parkinson's disease. Curr Opin
Neurol 2007;20:484-492.

Question 119: Clinical Adult - Neuro-oncology

Discussion:

Anticonvulsant medications are not effective in preventing first seizures in patients with newly diagnosed brain tumors.
Prophylaxis is therefore not recommended due to potential side effects and it is appropriate to taper and discontinue prophylactic
anticonvulsants after the first post-operative week.

References:

Glantz MJ, Cole BF, Forsyth PA, et al. Practice parameter: anticonvulsant prophylaxis in patients with newly diagnosed brain
tumors. Report of the Quality Standards Subcommittee of the American Academy of Neurology. Neurology 2000;54:1886-1893.

Question 122: Clinical Adult - Cerebrovascular Disease

Discussion:

In the absence of clear contraindications, anticoagulation with warfarin is the most appropriate agent for the prophylaxis against
recurrent strokes in a patient with atrial fibrillation who has had a stroke or TIA.

References:
Albers GW, Dalen JE, Laupacis A, et al. Antithrombotic therapy in atrial fibrillation. Chest 2001;199 (1 Suppl):194S 206S.

Question 124: Clinical Adult - Neuromuscular Disorders

Discussion:

This patient s symptoms are most suggestive of myotonia congenita, which occurs due to muscle chloride channel dysfunction
and presents as muscle stiffness and cramps beginning in childhood. The stiffness of myotonia congenita typically is very
prominent after rest and improves with exercise. The hypertrophic calves described in this patient are commonly seen in
myotonia congenita.

References:

Chrestian N, Puymirat J, Bouchard J, Dupre N. Myotonia congenita--a cause of muscle weakness and stiffness. Nat Clin Pract
Neurol 2006;2:393-399.

Question 125: Clinical Adult - Neuromuscular Disorders

Discussion:

This patient most likely has a left radial neuropathy from compression of the nerve in the radial groove. It is common in patients
who have been intoxicated and therefore sleeping in one position for a prolonged period ("Saturday night palsy"). Generally the
condition will improve spontaneously over a period of weeks to months and investigations are not required.

References:

Spinner RJ, Poliakoff MB, Tiel RL. The origin of "Saturday night palsy"? Neurosurgery 2002;51:737-741.

Question 130: Clinical Adult - Neuro-ophthalmology/Neuro-otology

Discussion:

This patient's symptoms of a pupillary-involving third cranial nerve palsy are most consistent with, and concerning for,
aneurysmal compression of the third cranial nerve by a posterior communicating artery or carotid artery aneurysm.

References:

Brazis PW, Masdeu JC, Biller J. Localization in clinical neurology. 4th ed. Philadelphia: Lippincott, Williams & Wilkins, 2001.

Question 132: Clinical Adult - Headache

Discussion:

Orgasmic headaches are a cause of severe intense headache occurring in the context of sexual activity. As with any patient with a
thunderclap headache, less benign causes (such as aneurysmal subarachnoid hemorrhage) should be considered and excluded
before making the diagnosis of orgasmic headache.

References:

Wvans RW, Pascual J. Expert opinion: orgasmic headaches: clinical features, diagnosis, and management. Headache
2000;40:491-494.

Question 133: Clinical Adult - Dementia


Discussion:

The syndrome described is dementia with Lewy bodies (DLB) which is associated with episodic confusion, parkinsonian
syndrome, visual hallucinations and REM behavior sleep disorder (RBSD). Visual loss may lead to visual hallucination in other
cases (Bonnet syndrome), while prosopagnosia is associated with Alzheimer disease and other cortical syndromes. Diplopia is
found in progressive supranuclear palsy; DLB is not associated with aphasia.

References:

Boeve BF. Dementia with Lewy bodies. Continuum: lifelong learning in neurology 2004:10:84-89.

Question 137: Clinical Adult - Movement Disorders

Discussion:

A levodopa challenge should be considered when the diagnosis of Parkinson disease is in doubt. The other tests listed (olfaction,
repeat MRI, PET, and autonomic testing) are not clinically useful in distinguishing Parkinson disease from other parkinsonian
syndromes based upon insufficient evidence from clinical studies.

References:

Suchowersky O, Reich S, Perlmutter J, et al. Practice parameter: diagnosis and prognosis of new onset Parkinson disease (an
evidence-based review). Report of the Quality Standards Subcommittee of the American Academy of Neurology. Neurology
2006;66:968-975.

Question 138: Clinical Adult - Dementia

Discussion:

Mild cognitive impairment (MCI) is a mild form of a primary neurodegenerative cognitive disorder. Diagnosis of MCI or early
dementia in highly intelligent/educated persons can be difficult and often requires a high index of suspicion and
neuropsychological testing. MMSE is a useful general screening tool but has low sensitivity in this population. Obtaining
collateral history is paramount, and this patient s collateral history of decline in forming new memories and a 27/30 MMSE
score (borderline abnormal for age and education) preclude normal aging. A diagnosis of MCI requires slightly abnormal
cognitive function in single or multiple domains, without evidence of significant abnormalities in activities of daily living (ADL).
The diagnosis of Alzheimer disease (AD) requires a history of slow onset and gradually progressive decline in cognition,
function, or behavior; objective evidence of deficits in memory and one other cognitive domain; evidence of significant
functional decline that interferes with ADL, social, or occupational functions; deficits that cannot be better explained by another
central nervous system or systemic illness and cannot be made in the context of encephalopathy (delirium). Major risk factors for
AD include advancing age and family history; senility is not a consequence of normal aging and should raise suspicion for
positive family history of dementia. Vascular-ischemic dementia (VaID) encompasses a heterogeneous group of syndromes; it is
not defined by a specific pattern of cognitive deficits. Acute onset and step-wise decline of cognitive symptoms, history of known
stroke or hypertension, and focal neurologic symptoms and signs are highly consistent with VaID but are not always present.
Correlation of symptoms with brain infarction location or severe leukoaraiosis on brain imaging should be verified prior to
diagnosis. Polyneuropathy and may contribute to her cognitive symptoms but are unlikely to be primarily causative.

References:

Kawas CH. Clinical practice. Early Alzheimer s disease. N Engl J Med 2003;349:1056-1063.

Question 141: Clinical Adult - Movement Disorders

Discussion:
This patient has essential tremor and would be best initially treated with primidone. Propranolol would be relatively
contraindicated in a patient with diabetes. Medication should be tried first before botulinum toxin for head tremors. The other
listed medications are not used for essential tremor.

References:

Victor M, Ropper A. Adams and Victor's principles of neurology. 7th ed. New York: McGraw-Hill, 2001.

Question 144: Clinical Adult - Cerebrovascular Disease

Discussion:

Cerebral amyloid angiopathy is a cause of lobar intracerebral hemorrhage in elderly patients, and is implicated as a potential
cause of chronic microbleeds on gradient echo MRI.

References:

Chao CP, Kotsenas AL, Broderick DF. Cerebral amyloid angiopathy: CT and MR imaging findings. Radiographics
2006;26:1517-1531.

Question 152: Clinical Adult - Spinal and Root Disorders

Discussion:

This patient most likely has cervical myeloradiculopathy, secondary to cervical spondylosis with central canal and foraminal
stenosis.

References:

Noseworthy J. Neurological therapeutics: principles and practice. New York: Taylor & Francis, 2006.

Question 154: Clinical Adult - Critical Care/Trauma

Discussion:

This patient is suffering from carbon monoxide poisoning, most likely from a malfunctioning home furnace. High flow oxygen
should be administered immediately. Because of the tight binding of the carboxy group to hemoglobin, oxygen ideally should be
administered under hyperbaric conditions.

References:

Hardy KR, Thom SR. Pathophysiology and treatment of carbon monoxide poisoning. J Toxicol Clin Toxicol 1994;32:613-629.

Question 180: Clinical Adult - Neuro-ophthalmology/Neuro-otology

Discussion:

This patient's symptoms of isolated brief episodes of vertigo precipitated by head movement (including turning in bed or looking
up) are most compatible with the diagnosis of benign paroxysmal positional vertigo (BPPV), most likely due to canalithiasis
affecting a posterior semicircular canal. In the absence of other signs or symptoms of brainstem dysfunction during the episodes,
vertebrobasilar TIAs are unlikely; in addition her symptoms while supine are not consistent with orthostatic hypotension. The
diagnostic procedure for BPPV is the Dix-Hallpike maneuver, which will likely cause (after a few second latency) recapitulation
of the patient's symptoms along with brief rotatory nystagmus.
References:

Fife T. Recurrent positional vertigo. Continuum: lifelong learning in neurology 2006;12:92-115.

Question 183: Clinical Adult - Movement Disorders

Discussion:

Cortical-basal ganglionic degeneration can have a similar presentation as idiopathic Parkinson disease in addition to cortical
features including hand apraxia and cortical sensory loss. Patients tend to not respond to levodopa. MRI, in some patients,
demonstrates marked cortical asymmetry. Pathology demonstrates neuronal loss and classic ballooned neurons.

References:

Jankovic JJ, Tolosa E. Parkinson's disease and movement disorders. 4th ed. Philadelphia: Lippincott, Williams and Wilkins,
2002.

Question 184: Clinical Adult - Neuromuscular Disorders

Discussion:

Spinal bulbar muscular atrophy (Kennedy disease) is characterized by lower motor neuron findings, especially affecting cranial
musculature associated with decreased libido and gynecomastia. Female carriers are asymptomatic. Muscle biopsy shows
nondiagnostic neuropathic changes. Testicular biopsy shows diminished spermatozoids with abnormal motility and mild
elevation of FSH and LH. Diagnosis is by DNA analysis which shows expanded CAG repeats translated into polyglutamine
repeats.

References:

Ashizawa T, Zoghbi HY. Diseases with trinucleotide repeat expansion. In: Appel SH, editor. Current neurology. (Vol 17)
Amsterdam: IOS Press, 1997.

Question 185: Clinical Adult - Critical Care/Trauma

Discussion:

This patient has herpes simplex encephalitis, the most commonly identified cause of sporadic viral encephalitis in the United
States. The early development of seizures is typical of this disorder because of the tendency to involve the temporal lobes. The
lymphocyte-predominant CSF with xanthochromia and normal glucose are typical. The bacterial agents listed would produce a
polymorphonuclear cell-predominant pleocytosis and low glucose. West Nile virus usually does not have xanthochromic CSF.

References:

Tyler KL. Herpes simplex virus infections of the central nervous system: encephalitis and meningitis, including Mollaret's.
Herpes 2004;11(Suppl 2):57A-64A.

Question 186: Clinical Adult - Spinal and Root Disorders

Discussion:

This patient has a C5-C6 disc herniation producing a relatively mild acute C6 radiculopathy. The diagnosis is clear from the
information provided, and no additional diagnostic testing (EMG, myelography) should be required. In most cases the symptoms
will resolve spontaneously without need for cervical discectomy. Surgery would be required if the patient had significant
radicular weakness, cervical myelopathy, or intractable radicular pain. Epidural steroid injection does not have a defined role in
the treatment of radiculopathy but may improve pain control in patients with radicular pain that is severe or does not respond to
oral analgesics.

References:

Bradley WG, Daroff RB, Fenichel G, Jankovic J. Neurology in clinical practice. 4th ed. Boston: Butterworth-Heinemann, 2003.

Question 188: Clinical Adult - Epilepsy

Discussion:

This is a single unprovoked generalized seizure in a patient with a normal neurologic examination and a normal evaluation. The
etiology is likely idiopathic, and his risk for recurrent seizure is low (about 30% over the next 5 years); therefore, observation is
warranted.

References:

Marson A, Jacoby A, Johnson A, et al. Immediate versus deferred antiepileptic drug treatment for early epilepsy and single
seizures: a randomized controlled trial. Lancet 2005;365(9476):2007- 2013.

Sirven J. Antiepileptic drug therapy for adults: when to initiate and how to choose. Mayo Clin Proc 2002;77(12):1367-1375.

Question 189: Clinical Adult - Neuro-ophthalmology/Neuro-otology

Discussion:

This patient's history is most compatible with a unilateral Adie's tonic pupil. An Adie's pupil is typically initially large (although
becomes smaller over time) and shows minimal reaction to light but normal constriction to near--called light-near dissociation.
Unlike normal pupils, which will show no response to instillation of dilute pilocarpine, Adie's pupils will constrict to this agent
due to denervation supersensitivity from ciliary ganglion dysfunction. Unlike Adie's pupils, Argyll-Robertson pupils due to
neurosyphilis are typically bilaterally small. Adie's pupils are often associated with impaired muscle stretch reflexes; this
combination of findings is referred to as the Holmes-Adie syndrome.

References:

Disorders of the pupil. Continuum: lifelong learning in neurology: neuro-ophthalmology. October 2003;9:149-174.

Question 192: Clinical Adult - Dementia

Discussion:

Amnestic mild cognitive impairment (aMCI) is, neuropathologically, most often a precursor state to Alzheimer disease. As such,
there is medial temporal lobe pathology demonstrated. This is reflected in MRI studies which have shown decreased hippocampal
volume as an independent predictor for the development of dementia in subjects with aMCI.

References:

Petersen RC. Mild cognitive impairment. Continuum: lifelong learning in neurology 2004;10(1):19.

Question 193: Clinical Adult - Neurology of Systemic Disease

Discussion:
Diabetic lumbosacral radiculoplexus neuropathy (also known as diabetic amyotrophy, proximal diabetic neuropathy, or diabetic
polyradiculopathy) typically presents as severe acute anterior thigh pain and proximal lower extremity weakness predominantly
involving the quadriceps. It typically begins unilaterally but in many patients ultimately involves the contralateral leg as well.
This syndrome is felt to be due to a microvasculopathy, with recent evidence suggesting a microvasculitis, involving these
proximal nerve structures.

References:

Briemberg HR. Peripheral nerve complications of medical disease. Semin Neurol 2009;29:124-135.

Question 199: Clinical Adult - Neurology of Systemic Disease

Discussion:

There is no established role for antiplatelet agents or anticoagulants in the treatment of ischemic strokes occurring as a
complication of embolism from bacterial endocarditis. Antibiotic therapy for the endocarditis is the mainstay of medical
treatment of this syndrome

References:

Baddour LM, Wilson WR, Bayer AS, et al. Infective endocarditis: diagnosis, antimicrobial therapy, and management of
complications. Circulation 2005;111:e394-e434.

Question 202: Clinical Adult - Epilepsy

Discussion:

This patient most likely has frontal lobe seizures. Frontal lobe seizures are commonly nocturnal and have prominent motor
manifestations and automatisms. There may be no postictal confusion.

References:

Herman S. Classification of epileptic seizures. Continuum: lifelong learning in neurology 2007;13:13-47.

Question 207: Clinical Adult - Neuromuscular Disorders

Discussion:

The clinical history and examination are most consistent with Lambert-Eaton myasthenic syndrome, which is associated with
malignancy (usually small cell lung cancer) in two-thirds of patients.

References:

Petty R. Lambert Eaton myasthenic syndrome. Pract Neurol 2007;7:265-267.

Question 208: Clinical Adult - Headache

Discussion:

This patient's headache fulfills the criteria for chronic paroxysmal hemicrania which is usually exquisitely responsive to
indomethacin.

References:
Goadsby PJ, Lipton RB. A review of paroxysmal hemicranias, SUNCT syndrome and other short-lasting headaches with
autonomic feature, including new cases. Brain 1997;120(Pt 1):193-209.

Question 263: Clinical Adult - Headache

Discussion:

Clinically, this patient's symptoms are most consistent with intracranial hypotension and the MRI shows diffuse dural
enhancement typical for that disorder. In this case, the hypotension most likely resulted from a spontaneous tear in the spinal
dura.

References:

Mokri B. Spontaneous intracranial hypotension. Curr Neurol Neurosci Rep 2001;1:109-117.

Noseworthy J. Neurological therapeutics: principles and practice. New York: Taylor & Francis, 2006.

Question 266: Clinical Adult - Dementia

Discussion:

This patient has Wernicke encephalopathy caused by thiamine deficiency. Clinically, patients with Wernicke's have confusion,
ataxia, and nystagmus or ophthalmoplegia. The MRI head shows enhancement of the mamillary bodies characteristic of this
disorder.

References:

Thomson AD, Cook CC, Guerrini I, et al. Wernicke's encephalopathy: 'Plus ca change, plus c'est la meme chose'. Alcohol
Alcohol 2008;43:180-186.

Question 275: Clinical Adult - Demyelinating Disease

Discussion:

The patient presents with a clinically isolated syndrome of optic neuritis with an abnormal brain MRI. According to the Optic
Neuritis Treatment Trial (ONTT), 10-year follow-up would predict a 56% chance of being diagnosed with clinically definite
multiple sclerosis (CDMS).

References:

Naismith RT, Cross AH. Magnetic resonance imaging of multiple sclerosis. Continuum: lifelong learning in neurology
2007;13:120-121.

Question 293: Clinical Adult - Neurogenetics

Discussion:

This patient has neurofibromatosis type 2 (NF-2) which is due to a mutation on chromosome 22.

References:

Ferner RE. Neurofibromatosis 1 and neurofibromatosis 2: a twenty first century perspective. Lancet Neurol 2007;6:340-351.

Question 305: Clinical Adult - Critical Care/Trauma


Discussion:

This patient has a classic picture of epidural hematoma. A concussive type head injury is followed by a "lucid interval". Then,
signs of progressive increased intracranial pressure develop typically with signs of uncal herniation. This condition is usually due
to laceration of the middle meningeal artery from a cranial fracture.

References:

Noseworthy J. Neurological therapeutics: principles and practice. New York: Taylor & Francis, 2006.

Question 361: Clinical Adult - Cerebrovascular Disease

Discussion:

This patient presents with stroke-like signs and is evaluated rapidly in an emergency setting. She appears to be a potential
candidate for receiving rt-PA based on her stroke scale, time window and absence of contraindications to the drug; however, her
blood pressure exceeds the guidelines for use of r-TPA and needs to be lowered acutely. Heparin and dipyridamole/ASA would
not be indicated; rt-PA is indicated, but blood pressure parameters need to be achieved first. The recommended agent for this
would be intravenous labetalol.

References:

Khatri P, Levine J, Jovin T. Intravenous thrombolytic therapy for acute ishemic stroke. Continuum: lifelong learning in
neurology 2008;14:52.

Question 379: Clinical Adult - Sleep

Discussion:

Augmentation in restless legs syndrome refers to symptoms occurring earlier in the day and sometimes spreading from the legs to
the arms. It should be distinguished from rebound by calculating the duration of therapy of the medication taken and the time it is
taken. In this case, rebound is unlikely given the one time dosing and half-life of the drug.

References:

Walters AS. Restless legs syndrome and periodic limb movements in sleep. Continuum: lifelong learning in neurology
2007;13(3):115-138.

Question 386: Clinical Adult - Infectious Disease

Discussion:

This is a case of progressive multifocal leukoencephalopathy (PML), caused by the JC virus. Highly active antiretroviral therapy
(HAART) is the only potentially effective treatment of HIV-associated PML. Pyrimethamine is used to treat toxoplasmosis.
Amphotericin B is an antifungal agent. The other drugs (AraC and interferon alpha) are not effective for HIV-associated PML.

References:

Evaluation and Management of Intracranial Mass Lesions in AIDS. Report of the Quality Standards Subcommittee of the
American Academy of Neurology. Neurology 1998;50:21-26.

Engsig FN, Hansen AB, Omland LH, et al. Incidence, clinical presentation, and outcome of progressive multifocal
leukoencephalopathy in HIV-infected patients during the highly active antiretroviral therapy era: a nationwide cohort study. J
Infect Dis 2009;199:77-83.
Question 396: Clinical Adult - Neurorehabilitation

Discussion:

Although the pattern of recovery following a stroke may be variable, the earliest evidence of a return of neurologic function is
usually an increase in tone. This generally occurs before improvement in strength or a return of reflexes.

References:

DeLisa JA, Gans BM. Rehabilitation medicine: principles in practice. 2nd ed. Philadelphia: Lippincott, 1993;458.

Question 400: Clinical Adult - Demyelinating Disease

Discussion:

This patient has retrobulbar neuritis. Oral prednisone at 1mg/kg/day has no proven efficacy in the recovery of visual function in
acute optic neuritis (ON). Higher dose parenteral methylprednisolone may hasten the speed and degree of recovery of visual
function in persons with acute monosymptomatic ON.

References:

Bartleson JD, Schierman B, Edlund W, editors. Summaries of AAN Clinical Practice Guidelines for Adult Neurology. 2005-2006
ed. St. Paul, MN: American Academy of Neurology.

Beck RW, Cleary PA, Anderson M, et al. A randomized, controlled trial of corticosteroids in the treatment of acute optic neuritis.
The Optic Neuritis Study Group. N Engl J Med 1992;326:581-588.

Kaufman D, Trobe J, Eggenberger E, Whitaker J. Practice parameter: the role of corticosteroids in the management of acute
monosymptomatic optic neuritis: Report of the Quality Standards Subcommittee of the Amerian Academy of Neurology.
Neurology 2000;54:2039-2044.

Question 403: Clinical Adult - Cerebrovascular Disease

Discussion:

This patient has a lateral medullary syndrome caused by occlusion of the posterior inferior cerebellar artery. There is ipsilateral
facial anesthesia, Horner syndrome, and reduced gag reflex. There is ipsilateral limb ataxia and contralateral hemibody
anesthesia.

References:

Brazis PW, Masdeu JC, Biller J. Localization in clinical neurology. 5th ed. Philadelphia: Lippincott, Williams & Wilkins, 2006.

Question 411: Clinical Adult - Cerebrovascular Disease

Discussion:

This patient has a cerebral infarct in the distribution of the right anterior cerebral artery. If contrast is administered, subacute
infarcts commonly enhance, usually with a gyriform pattern.

References:

Ropper AH, Brown RH. Adams and Victor's principles of neurology. 8th ed. New York: McGraw-Hill, 2005; 261-264.
Question 421: Clinical Adult - Spinal and Root Disorders

Discussion:

Disturbance of the S1 nerve root results in pain radiating down the posterior aspect of the lower extremity, sensory disturbance of
the little toe, lateral foot, and most of the sole of the foot, weakness in several muscles including the gastrocnemius and soleus
muscles, and reduction or absence of the ankle jerk.

References:

Brazis PW, Masdeu JC, Biller J. Localization in clinical neurology. 5th ed. Philadelphia: Lippincott, Williams & Wilkins, 2006.

Question 422: Clinical Adult - Infectious Disease

Discussion:

This patient has post-transplant lymphoproliferative disease (PTLD) resulting from chronic immunosuppression and activation of
Epstein-Barr virus (EBV) infection. EBV causes monoclonal transformation of B-lymphocytes and in turn can result in primary
CNS lymphoma.

References:

Noseworthy J. Neurological therapeutics: principles and practice. New York: Taylor & Francis, 2006.

Question 425: Clinical Adult - Headache

Discussion:

This patient has medication overuse headache related to daily use of an acetaminophen/caffeine/butalbital medication. Although
the other interventions might help her headache problem, they would be less likely to do so than tapering the analgesic
medication.

References:

Boes CJ, Black DF, Dodick DW. Pathophysiology and management of transformed migraine and medication overuse headache.
Semin Neurol 2006;26:232-241.

Question 432: Clinical Adult - Neurogenetics

Discussion:

This patient has Friedreich ataxia, the most common autosomal recessive inherited ataxia. The underlying mutation is an
expansion repeat of both alleles of chromosome 9. Cardiomyopathy is the most common cause of death in these patients.

References:

Klockgether T. Hereditary ataxias and other cerebellar degenerations. In: Noseworthy JH, editor. Neurologic therapeutics:
principles and practice. London, New York: Martin Dunitz, 2003;2689.

Question 437: Clinical Adult - Movement Disorders

Discussion:
This patient has restless legs syndrome (RLS) most likely secondary to iron deficiency. A serum iron and ferritin should be
checked.

References:

Trenkwalder C, Hening WA, Montagna P, et al. Treatment of restless legs syndrome: an evidence-based review and implications
for clinical practice. Mov Disord 2008;23:2267-2302.

Question 441: Clinical Adult - Neuromuscular Disorders

Discussion:

This patient's clinical signs and symptoms are most compatible with Isaac syndrome, or neuromyotonia. This condition is an
autoimmune and sometimes paraneoplastic condition, felt to be related to antibodies to the voltage-gated potassium channel,
although these are demonstrated on clinical testing in only approximately 50% of patients. The syndrome is characterized by
muscle cramps and stiffness as well as the presence in some patients of marked myokymia causing undulating wave-like
spontaneous muscle activity evident on examination. EMG may show characteristic myokymic discharges with doublets and
triplets with a high interburst frequency. Isaac syndrome can also be associated with sensory disturbances (paresthesias) and
dysautonomia.

References:

Newsom-Davis J, Buckley C, Clover L, et al. Autoimmune disorders of neuronal potassium channels. Ann N Y Acad Sci
2003;998:202-210.

Ryan A, Mullins G, Scott J, et al. A 45-year history of acquired autoimmune neuromyotonia. J Neurol 2006;253:1243-1245.

Clinical Pediatrics

Question 7: Clinical Pediatrics - Cerebral Palsy

Discussion:

The manifestations of full-blown kernicterus (bilirubin encephalopathy) include choreoathetoid cerebral palsy, abnormal
voluntary eye movements with lack of upgaze, and sensorineural hearing loss. In otherwise well term infants, indirect bilirubin is
generally higher than 25 mcg/ml prior to any neurologic sequelae, and jaundice is clinically obvious. The manifestations of the
listed inborn errors of metabolism may include choreoathetosis, but cognitive function is invariably affected in untreated patients.

References:

Mitchell WG. Diseases of the nervous system in childhood. Aicardi J, editor. 3rd ed. London: Mac Keith Press, 2009.

Question 9: Clinical Pediatrics - Neuromuscular

Discussion:

The congenital myasthenic syndromes are not related to an immune process but are caused by genetic defects affecting the
neuromuscular junction. These include defects in acetylcholine synthesis and packaging (familial infantile myasthenia gravis),
end-plate deficiency of acetylcholinesterase, acetylcholine receptor deficiency, and the slow channel syndrome.

References:

Swaiman KF, Ashwal S, Ferriero DM, editors. Pediatric neurology: principles and practice. 4th ed. Philadelphia: Mosby, 2006.

Dubowitz V. Muscle disorders in childhood. Philadelphia: WB Saunders, 1995.


Question 16: Clinical Pediatrics - Epilepsy

Discussion:

Formerly called "5th day fits", benign idiopathic neonatal convulsions are an autosomal dominant disorder which has been
strongly linked to potassium channel defects. The normal exam makes hypoxia unlikely as most are lethargic. Most children with
metabolic disorders are lethargic or even comatose. Benign rolandic epilepsy presents in adolescence.

References:

Volpe JJ. Neurology of the newborn. 5th ed. New York: Elsevier, 2008.

Question 28: Clinical Pediatrics - Hereditary and Metabolic Disorders

Discussion:

Glutaric aciduria typically presents with macrocephaly, with or without developmental delay, and hypotonia or a hyperkinetic
movement disorder (the latter may slowly progress or present acutely as a catastrophic deterioration, usually in the context of
intercurrent infection). Treatment with carnitine may be effective in preventing deterioration. Krabbe disease and Rett syndrome
are associated with microcephaly; obstructive hydrocephalus is associated with enlarged ventricles; and myoclonic epilepsy with
ragged red fibers (MERRF) is associated with ataxia, myoclonus, seizures, and myopathy.

References:

Swaiman KF, Ashwal S, Ferriero DM, editors. Pediatric neurology: principles and practice. 4th ed. Philadelphia: Mosby, 2006.

Question 30: Clinical Pediatrics - Neurosurgery, Critical Care and Tumors

Discussion:

Medulloblastomas are a type of primitive neuroectodermal tumor arising in the cerebellum, most often occurring in the fourth
ventricle. The typical neuroimaging is of a tumor that is dense on noncontrast CT and diffusely enhances on CT or MRI. MR
spectroscopy characteristically has high choline peak, taurine peak, and low to absent N-acetylaspartate. So-called average risk
medulloblastomas are those that do not have evidence of leptomeningeal spread at the time of diagnosis and have been
completely or nearly completely surgically resected.

References:

Finlay JL, Packer RJ, Erdreich-Epstein A. Progress in the treatment of childhood brain tumors: no room for complacency. Pediatr
Hematol Oncol 2007;24:79-84.

Question 37: Clinical Pediatrics - Develop, Learn, Lang, Behav, Psych Disorders

Discussion:

This child has tuberous sclerosis complex, an autosomal dominant disorder associated with mutations in the tuberin and merlin
genes. Intellectual outcome is best correlated with seizure control.

References:

Goh S, Kwiatkowski DJ, Dorer DJ, Thiele EA. Infantile spasms and intellectual outcomes in children with tuberous sclerosis
complex. Neurology 2005;65:235-238.

Gomez MR, Sampson J, Whittemore V, editors. Tuberous sclerosis complex. 3rd ed. New York: Oxford University Press, 1999.
Curatolo P, Verdecchia M, Bombardieri R. Vigabatrin for tuberous sclerosis complex. Brain Dev 2001;23(7):649-653.

Question 42: Clinical Pediatrics - Neuromuscular

Discussion:

Nemaline rod myopathy has a static or slowly progressive course with rod-like inclusions on trichrome studies. Creatine kinase is
normal or mildly elevated; EMG may be normal. The most common cause is a mutation in the alpha-actin gene.

References:

Swaiman KF, Ashwal S, Ferriero DM, editors. Pediatric neurology: principles and practice. 4th ed. Philadelphia: Mosby, 2006.

Dubowitz V. Muscle disorders in childhood. Philadelphia: WB Saunders, 1995.

Question 55: Clinical Pediatrics - Neuromuscular

Discussion:

Only daily oral corticosteroids have been shown to prolong ambulation in Duchenne muscular dystrophy (DMD). The effect is to
induce small amounts of dystrophin, not as an antiinflammatory treatment, so alternate day treatment is not helpful. Intravenous
gentamicin has been proposed for one of the rare point mutations causing DMD. Trials of oral carnitine and valproic acid are
underway in SMA.

References:

Moxley RT, Ashwal S, Pandya A. Practice parameter: corticosteroid treatment in Duchenne dystrophy: report of the Quality
Standards Subcommittee of the American Academy of Neurology and the Practice Committee of the Child Neurology Society.
Neurology 2005;64:13-20.

Question 66: Clinical Pediatrics - Neonatal

Discussion:

Congenital myotonic dystrophy is inherited from the mother, who may be relatively asymptomatic. EMG and muscle biopsy of
the baby is not helpful. EMG of the mother can support the diagnosis. The diagnosis of the baby is confirmed by finding an
abnormal expansion of a trinucleotide repeat on chromosome 19. Children with congenital myotonic dystrophy have delayed
motor milestones. Mental retardation is common.

References:

Swaiman KF, Ashwal S, Ferriero DM, editors. Pediatric neurology: principles and practice. 4th ed. Philadelphia: Mosby, 2006.

Question 72: Clinical Pediatrics - Cerebral Palsy

Discussion:

Prenatal or postnatal middle cerebral artery infarction is the most common cause of congenital hemiparesis. About 1/6000
uncomplicated live births have an arterial infarction. In most instances no clear etiologic factors can be identified. Children are
usually normal at birth, although those with perinatal infarctions may manifest seizures, usually restricted to the hand. Far less
commonly encountered etiologies include subdural hemorrhage, arteriovenous malformations, or hemimegalencephaly, and in
such instances additional findings or a more eventful history are often encountered. Unilateral infantile venous sinus thrombosis
is rare and is seldom an occult process.

References:
Swaiman KF, Ashwal S, Ferriero DM, editors. Pediatric neurology: principles and practice. 4th ed. Philadelphia: Mosby, 2006.

Question 75: Clinical Pediatrics - Neurosurgery, Critical Care and Tumors

Discussion:

Cerebellar mutism, also called posterior fossa syndrome, is a complication of any surgical procedure within the posterior fossa,
but more likely to occur following surgeries that involve tumor resection, particularly if the vermis is involved. While various
cranial nerve palsies may be seen, the syndrome is also seen without other evidence of structural damage. Typically, onset of
severe dysarthria progressing to mutism is delayed hours to days after initial surgical recovery. Severe irritability is common.
Recovery takes weeks to months and may never be complete, as many children with this manifestation show ongoing cognitive
and behavioral problems years after successful surgery.

References:

Koh S, Turkle SB, Baram TZ. Cerebellar mutism in children: report of six cases and potential mechanisms. Pediatr Neurol
1997;16:218-219.

Mitchell WG. Diseases of the nervous system in childhood. Aicardi J, editor. 3rd ed. London: Mac Keith Press, 2009.

Question 98: Clinical Pediatrics - Headache/Other Paroxysmal Disorders

Discussion:

The combination of dysconjugate, highly variable nystagmus, head nodding and head tilt without ophthalmologic abnormalities,
and with normal neuroimaging, is diagnostic of spasmus nutans. Latent nystagmus is a jerk nystagmus that is evoked or enhanced
by covering one eye. Congenital nystagmus is usually conjugate and suppressed by convergence (in contrast to spasmus nutans,
which is typically increased by convergence). Opsoclonus describes chaotic, conjugate saccades, seen classically in the
paraneoplastic syndrome associated with neuroblastoma. Whipple disease produces a convergence-divergence nystagmus with
associated movements of the muscles of the head and neck (oculomasticatory myorhythmia).

References:

Swaiman KF, Ashwal S, Ferriero DM, editors. Pediatric neurology: principles and practice. 4th ed. Philadelphia: Mosby, 2006.

Question 99: Clinical Pediatrics - Neurosurgery, Critical Care and Tumors

Discussion:

The Presidents Commission recommended 100% oxygen ventilation for 10 minutes, followed by passive 100% oxygen for a
period long enough to achieve a PCO2 of 60. The other tests have high incidence of false-negative results.

References:

Swaiman KF, Ashwal S, Ferriero DM, editors. Pediatric neurology: principles and practice. 4th ed. Philadelphia: Mosby, 2006.

Question 100: Clinical Pediatrics - Infectious Disease

Discussion:

In children, neurocysticercosis most commonly is a single lesion, presenting with seizures at the time the organism begins to
involute and cause inflammatory response. The usual course is acute seizures only, with about 35% having ongoing epilepsy. The
lesion on CT or MRI is much more impressive than the clinical findings. Toxoplasmosis most commonly has multiple lesions and
doesn't present acutely except in immunosuppressed patients. Pilocytic astrocytoma usually would have a larger cystic
component. Bacterial brain abscess is unlikely to have a thin rim of enhancement.
References:

Mitchell WG. Neurocysticercosis and acquired cerebral toxoplasmosis in children. Semin Pediatr Neurol 1999;6:267-277.

Question 106: Clinical Pediatrics - Neurosurgery, Critical Care and Tumors

Discussion:

Fifteen percent of children with Down syndrome have atlantoaxial instability. Other heritable conditions with similar
predisposition are Klippel-Feil, Morquio, and Larsen syndromes, achondroplasia and previous cervical spinal surgery.

References:

Caviness AC. Evaluation of cervical spine injuries in children and adolescents. Up To Date online available from:
http://www.uptodateonline.com/patients/content/topic.do?topicKey=~0OOHfrn43NuAZ0 [accessed Feb 2010].

Question 112: Clinical Pediatrics - Headache/Other Paroxysmal Disorders

Discussion:

Idiopathic increased intracranial pressure or pseudotumor cerebri can occur at any age or sex, but is more common in adolescent
and mature females. The increased pressure often causes VI or IV nerve palsies. The normal CT does not completely rule out a
posterior fossa tumor, but makes it less likely. Most posterior fossa tumors will cause some degree of hydrocephalus. An MRI
with MRV followed by a spinal tap would be appropriate management.

References:

Menkes JH, Sarnat HB, Maria BL, editors. Child neurology. 7th ed. Philadelphia: Lippincott, Williams & Wilkins, 2006.

Question 116: Clinical Pediatrics - Epilepsy

Discussion:

Panayiotopoulos syndrome is a benign (non-lesional), age-related focal epilepsy of childhood. It differs from benign rolandic
epilepsy primarily in the length of episodes, the frequent occurence of vomiting, and the lack of typical central-temporal spikes
on EEG. While acute, confusional migraine may include vomiting, episodes do not typically begin during sleep.

References:

Mitchell WG. Diseases of the nervous system in childhood. Aicardi J, editor. 3rd ed. London: MacKeith Press, 2009.

Question 131: Clinical Pediatrics - Headache/Other Paroxysmal Disorders

Discussion:

A continuous, unilateral headache is most likely hemicrania continua, one of the trigeminal cephalgias. It is very responsive to
indomethacin, and this can be used as both a treatment and diagnostic trial.

References:

Cohen A, Matharu, M, Goadsby P. Trigeminal autonomic cephalalgias: current and future treatments. Headache 2007;47:969-
980.

Question 146: Clinical Pediatrics - Vascular and Inflammatory Disorders


Discussion:

Cerebrovascular disease occurs in 25% of sickle cell disease. Eighty percent of events occur at or before age 15 and most are
caused by a progressive cerebral vasculopathy that can be partially arrested by chronic transfusion therapy to keep hemoglobin S
below 30%. Most events are thrombotic.

References:

Swaiman KF, Ashwal S, Ferriero DM, editors. Pediatric neurology: principles and practice. 4th ed. Philadelphia: Mosby, 2006.

Question 190: Clinical Pediatrics - Movement Disorders

Discussion:

The darting random eye movements are consistent with opsoclonus, and the rapid presentation and symptom complex is most
consistent with opsoclonus-myoclonus-ataxia syndrome. Typical nystagmus with rapid lateral eye movements is seen in other
forms of acute ataxia such as postinfectious. Ataxia telangiectasia and neuropathy, ataxia, retinitis pigmentosa, and ptosis
(NARP) do not have opsoclonus and have slower onset.

References:

Menkes JH, Sarnat HB, Maria BL, editors. Child neurology. 7th ed. Philadelphia: Lippincott, Williams & Wilkins, 2006.

Question 200: Clinical Pediatrics - Epilepsy

Discussion:

Benign rolandic epilepsy remits completely by adolescence in about 80% of patients, although other epileptic syndromes,
including juvenile myoclonic epilepsy (JME), have been seen in some patients. Childhood absence remits in about two-thirds of
children, although a few develop generalized convulsive epilepsy in adolescence or adulthood. JME often requires prolonged or
indefinite treatment, with only about 20% remitting. Infants with prolonged febrile seizures followed by multiple seizure types
are often manifesting Dravet syndrome and respond poorly to treatment. Lennox-Gastaut syndrome rarely remits, and is often
intractable.

References:

Mitchell WG. Diseases of the nervous system in childhood. Aicardi J, editor. 3rd ed. London: MacKeith Press, 2009.

Questions 219 - 223: Clinical Pediatrics - Develop, Learn, Lang, Behav, Psych Disorders

Discussion:

Night terrors typically occur in deep sleep, often 90 minutes to 3 hours after sleep onset. The child appears awake but is not.
Autonomic symptoms are prominent. The child has no memory of the event.

Masturbation by infants is not often recognized immediately, causing excessive diagnostic testing. While most often seen in
infant and toddler girls, boys may have similar manifestations. Usually the behavior is specific to the setting (often in highchair
or car seat) but may also occur with the child lying on the floor, often with the legs extended or crossed. Facial flushing and
sweating is common. Although the behavior can be interrupted, the child typically returns to it as soon as they are not deterred.

Paroxysmal kinesiogenic choreoathetosis presents with unilateral choreic or dystonic movements with initiation of voluntary
movement such as walking. There is no alteration of consciousness.

Cyanotic breath-holding spells occur in infants after sudden pain or fright. Brief convulsion may be precipitated by the syncope.
Hemiplegic migraine is often dominantly inherited, consists of hemiplegia followed by headache and vomiting. Some are mapped
to a calcium channel gene.

References:

Mitchell WG. Diseases of the nervous system in childhood. Aicardi J, editor. 3rd ed. London: MacKeith Press, 2009.

Question 229: Clinical Pediatrics - Vascular and Inflammatory Disorders

Discussion:

MRI shows extensive bilateral lesions in both gray and white matter. MRS demonstrates marked elevation of lactate and decrease
in N-acetylaspartate (NAA) in an involved area. The course is chronic and while there is some relapsing/remitting quality, she
does not return to normal. Hearing loss is also associated with several of the mutations causing MELAS.

References:

Mitchell WG. Diseases of the nervous system in childhood. Aicardi J, editor. 3rd ed. London: Mac Keith Press, 2009.

Swaiman KF, Ashwal S, Ferriero DM, editors. Pediatric neurology: principles and practice. 4th ed. Philadelphia: Mosby, 2006.

Question 233: Clinical Pediatrics - Neonatal

Discussion:

When Vein of Galen aneurysms present in the neonate, it is usually with congestive heart failure, due to shunting of 25% or more
of cardiac output through the malformation. Vein of Galen malformations presenting later in infancy present with progressive
hydrocephalus, with expanding cranium and bulging fontanelle. Similar malformations later in childhood or adolescence may
present with Parinaud syndrome and/or mental retardation, multifocal infarctions due to ischemia from steal phenomena.

References:

Mitchell WG. Diseases of the nervous system in childhood. Aicardi J, editor. 3rd ed. London: Mac Keith Press, 2009.

Question 317: Clinical Pediatrics - Hereditary and Metabolic Disorders

Discussion:

Aicardi Syndrome includes agenesis of the corpus callosum and "punched out" retinal colobomas. It is an x-linked dominant
condition presumed to be fatal in males. The findings of the characteristic colobomas in the peripheral retina are diagnostic.
Although some girls with Aicardi Syndrome have more extensive brain malformation, they do not generally have hypothalamic-
pituitary dysfunction. The DCX mutation is associated with laminar heterotopias in girls, not agenesis of the corpus callosum,
and lissencephaly in boys. Although mitochondrial disorders may present with prenatal onset CNS migration disorders, agenesis
of the corpus callosum would not be expected. Organic acidurias do not generally present with cerebral malformations such as
agenesis of the corpus callosum.

References:

Mitchell WG. Diseases of the nervous system in childhood. Aicardi J, editor. 3rd ed. London: Mac Keith Press, 2009.

Question 320: Clinical Pediatrics - Movement Disorders

Discussion:
The MRI demonstrates a characteristic "eye of the tiger" with low T2 signal centered in a high T2 signal area. The characteristic
dystonic movements in PKAN2 may follow an initial period of apparent spasticity. Oromandibular dystonia is prominent.
Huntington disease presents with prominent dystonic features in children, rather than chorea, but the MRI usually shows atrophy
of caudate as well as cortical atrophy. DYT1 typically has no MRI findings. In glutaric aciduria I, onset of dystonia is earlier,
MRI shows necrotic lesions in basal ganglia and bitemporal atrophy. Patients with dopa-responsive dystonia often have early
findings suggestive of spasticity, progressing to dystonia, but typically have normal MRI.

References:

Mitchell WG. Diseases of the nervous system in childhood. Aicardi J, editor. 3rd ed. London: MacKeith Press, 2009.

Question 341: Clinical Pediatrics - Headache/Other Paroxysmal Disorders

Discussion:

MRI shows diffuse pachymeningeal enhancement, a finding suggestive of low pressure (ie, post LP) headaches. She probably
initially had either migraine or headache associated with acute viral illness, but evolved after having two spinal taps to have
findings consistent with low intracranial pressure.

References:

Schievink WI, Maya MM, Louy C. Cranial MRI predicts outcome of spontaneous intracranial hypotension. Neurology
2005;64:1282-1284.

Question 350: Clinical Pediatrics - Epilepsy

Discussion:

The combination of clusters of spasms which often are predominantly upon awakening in a child of this age is strongly
suggestive of infantile spasms. Reports of loss of, or decrease, in visual attention at the onset of spasms is common. The child's
measurements suggest significant microcephaly, implying the possibility of underlying brain malformation, previous infection, or
other acquired condition limiting brain growth. Benign infantile myoclonus often occurs while eating, and is not associated with
regression. Sandifer syndrome is usually a dystonic extensor spasm related to gastroesophageal reflux disease (GERD). Early
infantile epileptic encephalopathy (EIEE), or Ohtahara syndrome, is a more severe condition, often with fragmentary myoclonic
jerks. Lennox-Gastaut syndrome (LGS) may follow infantile spasms, but the typical evolution into LGS occurs at about age 2
years.

References:

Mitchell WG. Diseases of the nervous system in childhood. Aicardi J, editor. 3rd ed. London: Mac Keith Press, 2009.

MacKay MT, Weiss SK, Adams-Webber T, et al. Practice parameter: Medical treatment of infantile spasms: Report of the
American Academy of Neurology and the Child Neurology Society. Neurology 2004;62:1668-1681.

Question 351: Clinical Pediatrics - Hereditary and Metabolic Disorders

Discussion:

Notably, the infant's head size is well below normal. Of the choices provided, only lissencephaly is associated with microcephaly.
Perinatally acquired cytomegalovirus infection (CMV) is not generally associated with infantile spasms. By definition,
cryptogenic infantile spasms are those occurring in an otherwise normal infant. Tuberous sclerosis is frequently a cause of
infantile spasms, but is not generally associated with microcephaly. Aicardi syndrome generally does not occur in boys. The
diagnosis of cryptogenic infantile spasms implies no other abnormalities are found, which is not consistent with severe
microcephaly.

References:
MacKay MT, Weiss SK, Adams-Webber T, et al. Practice parameter: Medical treatment of infantile spasms: Report of the
American Academy of Neurology and the Child Neurology Society. Neurology 2004;62:1668-1681.

Mitchell WG. Diseases of the nervous system in childhood. Aicardi J, editor. 3rd ed. London: MacKeith Press, 2009.

Question 353: Clinical Pediatrics - Headache/Other Paroxysmal Disorders

Discussion:

Gamma hydroxybutyrate (sodium oxybate) diminishes cataplexy in many patients with narcolepsy. Modafinil or methylphenidate
may help daytime sleepiness.

References:

Robinson DM, Keating GM. Sodium oxybate: a review of its use in the management of narcolepsy. CNS Drugs 2007;21:337-
354.

Question 354: Clinical Pediatrics - Neurosurgery, Critical Care and Tumors

Discussion:

Optic gliomas are the most common CNS tumor for children with neurofibromatosis type 1, but other cranial nerves can be
affected by neurofibromas or schwannomas. Astrocytomas, meningiomas, medulloblastomas, ependymomas, and hamartomas
also occur with increased frequency in patients with neurofibromatosis type 1.

References:

Menkes JH, Sarnat HB, Maria BL, editors. Child neurology. 7th ed. Philadelphia: Lippincott, Williams & Wilkins, 2006.

Question 377: Clinical Pediatrics - Infectious Disease

Discussion:

A delayed onset, progressive hearing loss may occur in those children with an asymptomatic congenital cytomegalovirus (CMV)
infection that may not be noted until the child is faced with a delay in language development. As summarized by Volpe (2008),
hearing loss may occur in up to 11% of children with an asymptomatic congenital CMV infection.

References:

Volpe JJ. Neurology of the newborn. 5th ed. New York: Elsevier, 2008.

Question 393: Clinical Pediatrics - Epilepsy

Discussion:

The patient most likely has juvenile myoclonic epilepsy (JME). Although she has had only a single convulsion, the morning
"twitchiness" is most likely myoclonus, and she has a high chance of having further events. Phenytoin and carbamazepine often
worsen myoclonus in JME. Valproic acid provides good control but has embryotoxic effects and often contributes to obesity, so
is a poor choice in an adolescent female that is already obese. Lamotrigine must be started at a very low dose and gradually
increased. Risk of allergic reaction is increased by higher starting doses or rapid titration. Felbamate is never an appropriate first
medication. Topiramate could be considered but is poorly tolerated when started at full dosage and has not been labeled by the
US Food and Drug Administration for JME.

References:
Wyllie, E. The treatment of epilepsy, principles and practice. 2nd ed. Baltimore: Williams and Wilkins, 1997.

Question 397: Clinical Pediatrics - Hereditary and Metabolic Disorders

Discussion:

Pompe disease is a lysosomal glycogen storage disease that affects practically all tissues and results from a defect of 1,4-
glucosidase (acid maltase). Hypotonia, failure to thrive, and decreased reflexes develop during the first few months of life.
Cardiomegaly is prominent in infantile forms, which more commonly present with pulmonary insufficiency. Unlike other
glycogenoses, the liver is normal in size or only slightly enlarged, and there are no abnormalities of glucose homeostasis. PAS-
positive glycogen is seen in membrane-bound vacuoles in muscle, hepatocytes, and Schwann cells, but no abnormalities are seen
in myelin sheaths.

References:

Swaiman KF, Ashwal S, Ferriero DM, editors. Pediatric neurology: principles and practice. 4th ed. Philadelphia: Mosby, 2006.

Menkes JH, Sarnat HB, Maria BL, editors. Child neurology. 7th ed. Philadelphia: Lippincott, Williams & Wilkins, 2006.

Question 407: Clinical Pediatrics - Infectious Disease

Discussion:

Frequent seizures with a mildly slow EEG are the hallmark of catscratch encephalitis. Lyme disease is not associated with
seizures; Rocky Mountain spotted fever and the arboviruses usually have fever and prominent meningeal signs.

References:

Bradley WG, Daroff RB, Fenichel GM, et al, editors. Neurology in clinical practice. 3rd ed. Boston: Butterworth-Heinemann,
2000.

Question 408: Clinical Pediatrics - Hereditary and Metabolic Disorders

Discussion:

MELAS - mitochondrial encephalomyopathy, lactic acidosis and stroke-like episodes is one of several mitochondrial
encephalopathies. In cases where a clear maternal inheritance has been established, there is a mutation in the mitochondrial tRNA
leu. Cranial CT and MR scans in patients with MELAs reveal infarct-like areas bilaterally in the posterior cerebrum and almost
always hypodensities or calcifications in the basal ganglia region.

References:

Swaiman KF, Ashwal S, Ferriero DM, editors. Pediatric neurology: principles and practice. 4th ed. Philadelphia: Mosby, 2006.

Question 412: Clinical Pediatrics - Neuromuscular

Discussion:

Spinal muscular atrophy I usually presents with hypotonia, areflexia, paradoxical respiration with narrow upper chest, tongue
fasciculations, and progresses to difficulty swallowing, then respiratory distress. While infants with cerebral palsy may be
initially hypotonic, they are generally hyperreflexic. Infants with infantile Gaucher disease generally present with stiffness, not
initial hypotonia, and have organomegaly at onset.

References:
Swaiman KF, Ashwal S, Ferriero DM, editors. Pediatric neurology: principles and practice. 4th ed. Philadelphia: Mosby, 2006.

Question 423: Clinical Pediatrics - Develop, Learn, Lang, Behav, Psych Disorders

Discussion:

Children and young adults with Asperger syndrome have intact language, but often have unusual speech patterns. Limited
interests, often with puzzles, lists, and classification of objects, predominate. Social skills are poor. Inappropriate social behavior
is due to lack of ability to perceive the social responses and emotions of others. Autism is characterized by significant
impairments in language, not seen in Asperger syndrome. While some patients with Asperger syndrome may be dyslexic, the
impairments typically go far beyond a specific learning disability for reading. Childhood disintegrative disorder is a severe
disorder marked by dramatic regression, usually in the preschool years.

References:

Swaiman KF, Ashwal S, Ferriero DM, editors. Pediatric neurology: principles and practice. 4th ed. Philadelphia: Mosby, 2006.

McPartland J, Klin A. Asperger's syndrome. Adolesc Med Clin 2006;17:771-788.

Volkmar FR. Autism and pervasive developmental disorders. 2nd ed. New York: Cambridge University Press, 2007.

Question 426: Clinical Pediatrics - Develop, Learn, Lang, Behav, Psych Disorders

Discussion:

Rett syndrome is seen in young girls and is characterized by developmental arrest, stereotypic hand movements, deceleration of
head growth and loss of communication skills. It is an X-linked disorder and 80% to 85% of classic Rett syndrome patients will
have an abnormality in the MECP2 gene, while approximately 50% of "atypical" Rett patients will have an abnormality of this
gene.

References:

Swaiman KF, Ashwal S, Ferriero DM, editors. Pediatric neurology: principles and practice. 4th ed. Philadelphia: Mosby, 2006.

Question 429: Clinical Pediatrics - Epilepsy

Discussion:

The 12-year-old boy has a characteristic history for juvenile myoclonic epilepsy with early-morning myoclonic and generalized
tonic-clonic seizures, beginning in the second decade of life. Absence seizures may also occur. The EEG reveals generalized, fast
(4 Hz to 6 Hz) spike-and-wave complexes. Valproic acid is the treatment of choice and recurrence of seizures is likely to occur if
treatment is stopped.

References:

Menkes JH, Sarnat HB. Textbook of child neurology. 6th ed. Philadelphia: Lippincott, Williams & Wilkins, 2000.

Question 430: Clinical Pediatrics - Epilepsy

Discussion:

This 12-year-old boy has a characteristic history for juvenile myoclonic epilepsy with early-morning myoclonic and generalized
tonic clonic seizures, beginning in the second decade of life. Absence seizures may also occur. The EEG reveals generalized, fast
(4 Hz to 6 Hz) spike-and-wave complexes. Valproic acid is the treatment of choice and recurrence of seizures is likely to occur if
treatment is stopped.
References:

Menkes JH, Sarnat HB. Textbook of child neurology. 6th ed. Philadelphia: Lippincott, Williams & Wilkins, 2000.

Contemporary Issues

Question 31: Contemporary Issues - Practice

Discussion:

The serotonin syndrome often presents with autonomic instability, mental status change, and motor hyperexcitability. It is most
commonly associated with combinations of monoamine oxidase inhibitors and selective serotonin reuptake inhibitors.
Withdrawal of the offending medications often resolves the issue though some patients can progress to rhabdomyolysis,
respiratory failure and seizures.

References:

Boyer EW, Shannon M. The Serotonin syndrome.. NEJM 2005;352:1112-1120.

Kinsella L. Libby Zion's Lesson: Adverse Drug Reactions and Interactions. 7/14/2009
http://www.aan.com/news/?event=read&article_id=7955

Question 35: Contemporary Issues - Practice

Discussion:

Physical abuse is the most commonly REPORTED form of abuse. Emotional abuse is the most common form of abuse overall
but is less likely to be reported. Addressing the history of abuse is important in this evaluation as it has significant impact on
treatment and recovery.

References:

US Department of Justice, Office of Justice Programs, Bureau of Justice Statistics Special Report, Intimate Partner Violence,
2002.. 7/14/09 http://www.ojp.usdoj.gov/bjs/pub/pdf/ipv.pdf

Hohler AD, Rush SR. The role of the neurologist in cases of abuse and neglect. Neurologist 2007;13:73-78.

Question 78: Contemporary Issues - Core Competencies

Discussion:

Inquiry in the case-control study design begins with the outcome and then asks about exposure and nonexposure. The two cohort
designs begin with exposure and then ask about outcome (disease). Randomized trials are not observational studies.

References:

Evidence Based Medicine. 7/24/09 http://www.aan.com/education/ebm/

Question 90: Contemporary Issues - Ethics

Discussion:
Continuous on-site duty, including in-house call, must not exceed 24 consecutive hours. Residents may remain on duty for up to
6 additional hours to participate in didactic activities, transfer care of patients, conduct outpatient clinics, and maintain continuity
of medical and surgical care.

References:

Resident Duty hours. 7/20/09 http://www.acgme.org/acWebsite/home/home.asp- duty hours

Question 94: Contemporary Issues - Business

Discussion:

The answer is generalized (grand mal) status epilepticus. This patient has no prior history of seizures and therefore a diagnosis of
epilepsy is not made, making abnormal involuntary movements an incorrect diagnosis. Alteration of consciousness is incorrect in
that we know that the episode was a seizure, and this would not be coding to the highest level of specificity which is required by
HIPAA. The same is true for the answer "other convulsions", since the duration certainly qualifies for the diagnosis of status
epilepticus. Complex partial seizure is incorrect for lack of specificity, but also because this is a code category title only. A fifth
digit is required to describe if this alteration of consciousness is coma (1), transient alteration of awareness (2), persistent
vegetative state (3), or "other" (9), all being incorrect codes for this situation.

References:

The Official ICD-9-CM Guidelines for Coding and Reporting. 7/14/09


http://www.cdc.gov/nchs/datawh/ftpserv/ftpicd9/ftpicd9.htm

Question 115: Contemporary Issues - Driving

Discussion:

This case highlights two driving concerns. This patient has a seizure disorder related to an underlying neoplasm that currently is
well controlled but needs to be monitored. He has met the states time off the road after a seizure. He also has right leg weakness.
This second issue may also significantly impact his safety behind the wheel and is a separate concern that also needs to be
addressed. He may be safe to drive based on his therapy evaluations with or without an assistive device. A road driving test will
help to clarify this.

References:

Drive Safety. 7/28/09 http://www.drivesafety.com/HealthcareProfessionals/RehabilitationOT.aspx

Question 136: Contemporary Issues - Ethics

Discussion:

The neurology resident is an impaired provider after consuming alcohol while on call. She should not take the call. The second
resident has an obligation to ensure that the chief resident and the program director are informed.

References:

Bernat, James L. Ethical issues in Neurology. 3rd edition. Philadelphia:Wolters Kluwer Lippincott Williams &Wilkins, 2008

Question 187: Contemporary Issues - Core Competencies

Discussion:
Misclassification bias is a form of error that results from systematic inaccuracy in measurement, and it applies to either
dichotomous or continuous measures. It is also known as information bias. Interviewer bias and diagnostic bias are two examples
of misclassification bias. The radiologists in this example may not be using consistent techniques to assess the degree of stenosis.
Variability in their techniques could easily lead to diagnostic bias, which is the problem of the cases with stenosis being classified
as not having stenosis (and vice versa). Similarly, those with exposure may be classified as unexposed (and vice versa).

References:

Evidence Based Medicine. 7/24/09 http://www.aan.com/education/ebm/

Question 355: Contemporary Issues - Practice

Discussion:

If possible, avoidance of valproate (VPA) and antiepileptic drug (AED) polytherapy during the first trimester of pregnancy
should be considered to decrease the risk of major congenital malformations. If possible, avoidance of VPA and AED
polytherapy throughout pregnancy should be considered to prevent reduced cognitive outcomes.

References:

Harden CL, et al.. Practice Parameter Update: Management issues for women with epilepsy-focus on pregnancy (an evidence-
based review): Teratogenesis and perinatal outcomes. 7/20/09 http://www.aan.com/go/practice/guidelines
http://www.neurology.org/cgi/rapidpdf/WNL.0b013e3181a6b312v1.pdf

Question 360: Contemporary Issues - Practice

Discussion:

When a patient is unable to communicate his or her medical history, its value is limited. The neurologic examination also cannot
be as complete if the patient is not able to understand the commands given or report on sensory testing. Cultural competency
involves utilizing information about language and heritage to best treat a patient. An interpreter with training in medical
information is the best interpreter for assessing and translating patient information. A relative may not be able to provide the
detail needed to determine a diagnosis accurately and important details may not be understood or accurately communicated.
Conducting the interview in English will provide very limited results. While old records may be useful they will most likely not
give the complete story.

References:

Cultural Competency in Medicine. 7/14/09 http://www.amsa.org/programs/gpit/cultural.cfm

Question 374: Contemporary Issues - Practice

Discussion:

Carbamazepine or oxcarbazepine have the highest levels of evidence for pain control in trigeminal neuralgia while baclofen and
lamotrigine may be considered useful.

References:

Gronseth G, et al.. Oct 2008 Practice Parameter: The diagnostic evaluation and treatment of trigeminal neuralgia. 7/28/09
http://www.aan.com/go/practice/guidelines

Question 394: Contemporary Issues - HIPAA

Discussion:
As much as we sympathize with the plight of a family member, we must adhere to the patient s wishes, especially as she is a
mentally competent individual. This is the principle of confidentiality and is protected under HIPAA laws.

References:

United States Department of Health and Human Services Office for Civil Rights--HIPAA. Standards for Privacy of Individually
Identifiable Health Information [45 CFR Parts 160 and 164]. http://www.hhs.gov/ocr/hipaa/guidelines/guidanceallsections.pdf.

Question 405: Contemporary Issues - End-of-Life/Palliative Care

Discussion:

This patient who is able to make her own decisions has advanced disease and is no longer interested in medical or surgical
options. She is likely a candidate for hospice.

References:

National Hospice and Palliative Care Organization. 7/20/09 http://www.nhpco.org/templates/1/homepage.cfm

Question 435: Contemporary Issues - HIPAA

Discussion:

A health care provider or health plan may share relevant information with family members or friends involved in the patients
health care or payment for health care, if you tell the provider or plan that it can do so, or if you do not object to sharing of the
information. Even if family or friends accompany a patient to an appointment, this does not constitute approval to share medical
information without the patient's knowledge at a later date directly to those individuals.

References:

US Department of Health and Human Services Heath Information Protection. 7/24/09


http://www.hhs.gov/ocr/privacy/hipaa/understanding/consumers/familyfriends.html

Question 445: Contemporary Issues - End-of-Life/Palliative Care

Discussion:

Organ donation most commonly occurs in the context of severe brain injury and brain death. Public awareness and support of
organ donation and transplantation has increased in the last decade. As a result, families often recognize that the patient's
circumstances may be such that organ donation is a possibility. The family's request is not illegal, nor does it indicate they are
inappropriate decision makers for the patient because the patient has previously indicated the desire to be an organ donor. While
it is usually considered improper for physicians to raise the topic of organ donation with a family before brain death has been
determined, it is proper to respond to family requests about organ donation whenever they are made. Organ donation can occur
even for "coroner's cases" as the coroner can be in the operating room at the time the organs are removed for transplantation.
Because patients must be evaluated for medical suitability for organ donation, and because conversations with families about
organ donation require considerable skill, physicians should work closely with representatives of organ procurement
organizations when organ donation is considered a possibility.

References:

Williams MA, Lipsett PA, Rushton CH, et al. The physician's role in discussing organ donation with families. Crit Care Med
2003;31(5):1568-1573.

Neuroimaging
Question 23: Neuroimaging - Stroke

Discussion:

Technically adequate multidetector CT is sensitive to greater than 95% of acute subarachnoid hemorrhages within the initial 24
hours, with sensitivity gradually dropping off as time goes by. CT will find MOST, but not all cases, and thus lumbar puncture is
indicated when subarachnoid hemorrhage is clinically suspected but the CT is equivocal, negative, or not adequate technically.
Xanthochromia is NOT reliably determined by visual inspection alone, but rather by spectrophotometric evaluation.

References:

Edlow JA, Caplan LR. Avoiding pitfalls in diagnosis of subarachnoid hemorrhage. N Engl J Med 2000;342:29-36.

Question 224: Neuroimaging - Critical Care/Trauma

Discussion:

Venous infarction, secondary to thrombosis, is often underdiagnosed. Acutely thrombosed cerebral veins lack flow void and are
isointense on T-1 WI. There is lack of contrast enhancement in the anterior portion of the superior sagittal sinus. Venous
infarctions do not follow a strict arterial vascular distribution. Venous thrombosis can result in hemorrhagic infarction which is
not shown above. There is no evidence of amyloid angiopathy. Malignant hypertension can result in bilateral hemispheric
changes but most commonly in the parietal and occipital lobes

References:

Bakshi R, Lindsay BD, Kinkel PR. Brain magnetic resonance imaging in clinical neurology. In: Joynt RJ, Griggs RC, editors.
Baker's clinical neurology. Philadelphia: Lippincott, Williams & Wilkins, 1998;1(4A).

Question 225: Neuroimaging - Stroke

Discussion:

A 1.6 x 0.9 centimeter mass is seen that represents a saccular aneurysm of the left internal carotid artery that projects medially.
After diagnosis, this aneurysm was successfully treated with endovascular coil.

References:

Osborn A, Blaser S, Saltzman K, editors. Diagnostic imaging: brain. Salt Lake City: Amirsys, 2004.

Question 226: Neuroimaging - Dementia

Discussion:

There is marked enlargement of the third and lateral ventricles, with a normal-sized fourth ventricle. Note also the peculiar
configuration of the quadrigeminal plate, which appears displaced and compressed superiorly, but normally inferiorly, reflecting
dilation of the proximal portion of the aqueduct only. The axial T2 image through the midbrain also fails to demonstrate the
normal flow void seen at the level of the aqueduct that originated from high velocity of CSF traveling through the patent
aqueduct. These findings are consistent with a diagnosis of obstructive hydrocephalus, with obstruction at the level of the
aqueduct (excluding the choices of atrophy or communicating hydrocephalus). In this case, there is no evidence of a mass in the
quadrigeminal plate, and obstruction is due to stenosis.

References:

Bradley WB, Quencer RM. Hydrocephalus, atrophy, and intracranial CSF flow. In: Stark DD, Bradley WG. Magnetic resonance
imaging, Vol. 1. 2nd ed. St Louis: CV Mosby, 1992.
Question 227: Neuroimaging - CSF Circulation Disorders

Discussion:

These images demonstrate an arteriovenous malformation. The left middle cerebral artery is hypertrophied and gives multiple
feeding vessels to the nidus. Medial to that are large midline draining veins (thalamostriate vein and internal cerebral vein). The
nidus region is hyperdense due to polymerized n- butyl cyanoacrylate embolization material (onyx).

References:

Osborn A, Blaser S, Saltzman K, editors. Diagnostic imaging: brain. Salt Lake City: Amirsys, 2004.

Question 228: Neuroimaging - Infection

Discussion:

Axial T2-weighted image demonstrates a mass with central fluid signal and extensive surrounding edema resulting in mass effect
in the left frontal lobe. Gadolinium contrast enhanced axial T1-weighted image shows ring enhancement in the lesion. Diffusion-
weighted image shows marked restriction of diffusion, and thus narrows the otherwise broad differential diagnosis of a ring-
enhancing lesion to entities having restricted diffusion (acute infarction, infection). The signal intensity characteristics do not
support a diagnosis of hemorrhage, as signal intensities of blood breakdown products are not visible.

References:

Gaviani P, Schwartz RB, Hedley-Whyte T, et al. Diffusion-weighted imaging of fungal cerebral infection. AJNR Am J
Neuroradiol 2005; 26:1115 1121.

Question 230: Neuroimaging - Critical Care/Trauma

Discussion:

The most likely diagnosis is subarachnoid hemorrhage. The high density of the recent bleeding outlines the subarachnoid space
around the brain. Its shape rules out the other types of hemorrhage mentioned as options. There is hydrocephalus, but it is not
idiopathic; rather, it is caused by the subarachnoid hemorrhage. The air with pneumocephalus would have a much lower density
than any of the structures seen on the image.

References:

Greenberg JO, editor. Neuroimaging: a companion to Adams and Victor's principles of neurology. 2nd ed. New York: McGraw-
Hill, 1999.

Question 232: Neuroimaging - Critical Care/Trauma

Discussion:

The clinical history is inconsistent with the (much higher) severity of the imaged injuries. The extra-axial collections are of
different signal intensities, indicating at least three different ages of subdural hematoma, strongly suggesting multiple episodes of
traumatic injury. In this age group, these findings indicate nonaccidental trauma, unless several episodes of well documented
accidental trauma can be substantiated (ie, multiple serious automobile accidents, which is not likely).

References:

Blaser S, Illner A, Castillo M, et al. Pocket Radiologist PedsNeuro Top 100 Diagnoses. 1st ed. Salt Lake City: WB Saunders,
2003.
Question 234: Neuroimaging - Spine

Discussion:

The conus terminated at the L5-S1 level constituting a tethered cord syndrome. There is also a homogeneous mass with brightly
increased T1- and T2-weighted signal. This signal pattern is typical of a lipoma. A tumor or Tarlov cyst would have decreased T1
signal. No abnormal signal as seen in a spinal cord infarction is seen.

References:

Atlas S. Magnetic resonance imaging of the brain and spine. 3rd ed. Philadelphia: Williams and Wilkins, 2002.

Question 238: Neuroimaging - Brain Tumors

Discussion:

This extra-axial lesion diffusely and homogenously enhances with contrast. It sits on the dura. The morphology and location
make less likely the diagnoses of acoustic neurinoma and trigeminal schwannoma. Astrocytoma and hemangioma are intra-axial
tumors. This is an en-plaque variant meningioma.

References:

Mohr JP, Gautier JC, editors. Guide to clinical neurology. New York: Churchill Livingstone, 1995.

Question 242: Neuroimaging - Spine

Discussion:

MR is the imaging modality of choice for evaluation of spinal infections. The typical MRI appearance is loss of distinction
between the endplates, disks, and adjacent vertebral bodies on T1-weighted views and increased T2-weighted signal. These areas
show enhancement. The findings with high sensitivity for spinal infections include evidence of paraspinal, or epidural,
inflammatory tissues, contrast enhancement of the disc, and hyperintensity on fluid-gradient T2-weighted views. Spinal
infections rarely involve only one vertebral body and adjacent disc or only the epidural space.

References:

Ledermann H, Schweitzer M, Morrison W, Carrino J. MR imaging findings in spinal infections: rules or myths? Radiology
2003;228:506-514.

Question 243: Neuroimaging - Infection

Discussion:

History is IMPORTANT. Imaging without history is consistent with either chronic subdural hematoma or subdural empyema, but
with the history, the diagnosis becomes clear. This lesion is considered a neurosurgical emergency due to its high mortality rate if
not promptly diagnosed and appropriately treated. Outcomes have been shown to be better with wide craniotomy than with small
burr hole drainage. Subdural empyema frequently develops in the setting of paranasal sinus infection.

References:

Osborn A, Blaser S, Saltzman K, editors. Diagnostic imaging: brain. Salt Lake City: Amirsys, 2004.

Hall WA, Truwit CL. The surgical management of infections involving the cerebrum. Neurosurgery 2008;62(Suppl 2):519-530.
Question 244: Neuroimaging - Epilepsy

Discussion:

Neural migrational disorders are a heterogenous group of abnormality resulting in heterotopias with associated anomalies of site
thickening, shape and organization of cerebral cortex. The case illustrated shows tissue with signal characteristics of cortical
ribbon within the subcortical white matter adjacent to the left anterior horn and contiguous to the left frontal and temporal cortical
ribbon. This heterotopic tissue has isotense signal to the remainder of the cortical ribbon on all scan sequences.

References:

Pillai JJ, Hessler RB, Allison JD, et al. Advanced MR imaging of cortical dysplasia with or without neoplasm: reported two
cases. AJNR 2002;23:1686-1691.

Question 247: Neuroimaging - Critical Care/Trauma

Discussion:

The patient's case challenges recognition of the right subdural, left epidural, bilateral brain contusion, right uncal herniation and
left scalp hematoma.

References:

Osborn A, Blaser S, Saltzman K, editors. Diagnostic imaging: brain. Salt Lake City: Amirsys, 2004.

Question 253: Neuroimaging - Brain Tumors

Discussion:

The neuroimaging studies demonstrate a heterogenously enhancing lesion in the left hemisphere with surrounding vasogenic
edema and extension into the splenium of the corpus callosum. This appearance is most suggestive of a high-grade glioma.
Cerebral abscesses may look similar, but typically do not have the lacy, fibrillary network of enhancement within the lesions such
as the glioma in this case. Primary CNS vasculitis is best demonstrated by segmental stenosis of the intracranial arteries in a
"string of beads" appearance. Neurosarcoidosis typically shows strong enhancement of the leptomeninges and granulomas of the
parenchyma. Acute intracerebral hemorrhage would be readily apparent on noncontrast CT. Arteriovenous malformation
typically shows a network of flow voids that are seen on T2-weighted imaging.

References:

Greenberg JO, editor. Neuroimaging: a companion to Adams and Victor's principles of neurology. 2nd ed. New York: McGraw-
Hill, 1999.

Question 254: Neuroimaging - Dementia

Discussion:

This is an example of frontotemporal dementia, manifest by tau accumulation. The other findings are related to different diseases.

References:

Ropper AH, Brown RH. Adams and Victor's principles of neurology. 8th ed. New York: McGraw-Hill, 2005.

Question 256: Neuroimaging - Autoimmune Disorders (Non-MS)


Discussion:

The diffusion-weighted views demonstrate multiple focal areas of acute infarction. These involve both the deep gray and white
matter along with scattered cortical areas of infarction. These regions of infarction are in a multifocal vascular distribution
atypical for branch large vessel disease and similarly atypical for small vessel infarctions or a cardioembolic etiology, which tend
to be cortical. An MRA scan of the head performed concomitant with the study illustrated demonstrated marked beading along
numerous arterial structures typical of vasculitis. This is best seen and illustrated in Figure C on the conventional cerebral
angiogram.

References:

Pomper MG, Miller TJ, Stone JH, Tidmor WC et al.. CNS Vasculitis in Autoimmune Disease: MR Imaging findings in
correlation with angiography. AJNR 1999: 20:75-85..

Question 257: Neuroimaging - Metabolic

Discussion:

Superficial siderosis is a condition in which hemosiderin is deposited in the subpial layer of the brain and/or spinal cord as a
result of repeated, slow hemorrhage into the subarachnoid space. The classic triad includes sensorineural hearing loss, ataxia and
myelopathy. This MRI scan demonstrates hemosiderin deposition over the cerebellar folia, seen as patchy, ill-defined areas of
decreased signal intensity on T2 and T2- weighted views.

Arteriovenous malformations would demonstrate well formed T2 flow voids . Hepatolenticular degeneration (aka Wilson
disease) is an autosomal recessive disorder due to mutations in the ATP7B gene that results in abnormal copper deposition in the
lenticular nuclei and liver, resulting in movement disorders and liver dysfunction. MRI may demonstrate abnormal lesions in the
putamen and midbrain on T2WI, occasionally resulting in the face of the giant panda sign . Neurodegeneration with brain iron
accumulation (NBIA-I) is a rare neurodegenerative condition, associated with mutations in the pantothenate kinase gene
(PANK2) that results in abnormal iron deposition in the globus pallidus and substantia nigra. Patients present with progressive
rigidity, dysarthria, and involuntary movements. MRI may demonstrate hypointensity with a central region of hyperintensity in
the medial globus pallidus resulting in the so called eye-of-the-tiger sign .

References:

Atlas S. Magnetic resonance imaging of the brain and spine. 3rd ed. Philadelphia: Williams and Wilkins, 2002.

Kumar N, Cohen-Gadol A, Wright R. Superficial siderosis. Neurology 2006;66(8):1144-1152.

Question 259: Neuroimaging - Spine

Discussion:

Lumbar and sacral nerve roots exit below their corresponding vertebral body, but do so laterally and superiorly through the neural
foramina. Therefore, at the L4-5 disc interspace, the L4 nerve root exits above and lateral to the herniated disc. Neuroimaging
here demonstrates a right posterolateral disc extrusion that likely impinges upon the descending right L5 nerve root, which would
result in right extensor hallucis longus weakness. In contrast, a far right lateral disc herniation could theoretically result in an L4
nerve root compression, but this in not depicted in this MRI. The other answer choices correspond to physical exam findings
associated with other nerve roots (ie, adductor longus weakness [L3], decreased patellar reflex [L4], decreased ankle jerk reflex
[S1], saddle anesthesia [S3-5]).

References:

Greenberg JO, editor. Neuroimaging: a companion to Adams and Victor's principles of neurology. 2nd ed. New York: McGraw-
Hill, 1999.

Question 261: Neuroimaging - Stroke


Discussion:

The typical clinical picture for traumatic internal carotid artery dissection is a young to middle aged adult who has headache and
neck pain. Horner syndrome with headache and neck pain strongly suggest carotid dissection. Dissection narrows lumen of the
distal cervical internal carotid artery in this case.

References:

Amit Nautiyal,1 Sonal Singh,1* Michael DiSalle,1 and John O'Sullivan. Painful Horner Syndrome as a Harbinger of Silent
Carotid Dissection. [July 17, 2009] Available from:[http://www.pubmedcentral.nih.gov/articlerender.fcgi?pmid=15696206]

Osborne A. Diagnostic imaging: brain. Salt Lake City: Saunders-Elsevier, 2004.

Question 264: Neuroimaging - Stroke

Discussion:

The large pituitary mass has hyperdensity on the CT as seen with an acute hemorrhage. The mass shows increase T1- and T2-
weighted signal typical of recent hemorrhage. The mass also shows contrast enhancement typical of a large pituitary tumor. The
combination of these findings is consistent with a pituitary apoplexy.

References:

Greenberg JO, editor. Neuroimaging: a companion to Adams and Victor's principles of neurology. 2nd ed. New York: McGraw-
Hill, 1999.

Question 267: Neuroimaging - Spine

Discussion:

A hyperintense signal is seen on both T1-weighted and T2-weighted images, suggesting that the lesion is a lipoma. However,
there is also a tethered cord, suggesting that neurosurgery consultation is the most appropriate course of action. Radiation therapy
consultation is not necessary for the management of this lesion. A lumbar puncture and blood cultures are not recommended.
Intraspinal lipomas are benign and most likely clinically silent if not associated with tethered cord.

References:

Edelman R, Hesselink J, editors. Clinical magnetic resonance imaging. Philadelphia: WB Saunders, 1990.

Question 268: Neuroimaging - Stroke

Discussion:

CT perfusion images demonstrate prolonged mean transit time (MTT) in the left posterior cerebral artery territory with matched
region of decreased regional cerebral blood flow (rCBF). The regional cerebral blood volume (rCBV) map shows a
corresponding but slightly smaller region of decreased blood volume in the tissue, consistent with infarction. This patient
completed her left occipital infarction.

References:

Srinivasan A, Goyal M, Al Azri F,Lum C.. State-of-the-Art Imaging of Acute Stroke. Radiographics 2006;26:S75-S95.

Question 271: Neuroimaging - Multiple Sclerosis


Discussion:

Multiple, supratentorial and infratentorial lesions are seen, with 2 acute enhancing lesions. This is most consistent with an acute
exacerbation of multiple sclerosis. Vasculitic and other autoimmune conditions may also have similar imaging features and
further clinical and laboratory information would be needed to make this distinction. These images are not consistent with either
ischemic or hemorrhagic stroke. No leptomeningeal enhancement is present in this study. Variant Creutzfeldt-Jakob disease
(vCJD) is a rare prion disease typically seen in younger patients who present with rapidly progressive cognitive decline, visual
disturbance and may have increased signal intensity lesions on FLAIR views in the bilateral posterior thalami ( pulvinar sign ).

References:

Greenberg JO, editor. Neuroimaging: a companion to Adams and Victor's principles of neurology. 2nd ed. New York: McGraw-
Hill, 1999.

Question 272: Neuroimaging - Developmental/Neurogenetic Disorders

Discussion:

The mass lesion has heterogenous T2-weighted and T1-weighted signal characteristic of chronic hemorrhage with hemosiderin.
The lesion has a speckled or popcorn appearance typical of a cavernous hemangioma or cavernoma. An aneurysm has a smooth
margin and more homogenous central hemorrhagic pattern. A basal ganglia hemorrhage would not have a "popcorn" like
appearance, nor would a hemorrhagic metastasis as would be seen with a melanoma or a glioma.

References:

Greenberg JO, editor. Neuroimaging: a companion to Adams and Victor's principles of neurology. 2nd ed. New York: McGraw-
Hill, 1999.

Question 276: Neuroimaging - Movement Disorders

Discussion:

There are no features of either Chiari 1 or 2 malformation. Olivopontocerebellar atrophy (OPCA) would typically also
demonstrate more prominent pontine atrophy as well. There is no history of phenytoin use in the past.

References:

Osborn A. Diagnostic neuroradiology. St. Louis: Mosby, 1994.

Question 280: Neuroimaging - Critical Care/Trauma

Discussion:

Anoxia causes increased T2 signal in the globus pallidus, deep portions of the sulci, and superior cerebellum, which are all
anatomical regions that are susceptible to hypoxia. The necrotic globus pallidus enhances after gadolinium administration.

References:

Greenberg JO. Neuroimaging: a companion to Adams and Victor's principles of neurology. New York: McGraw-Hill, 1999.

Question 283: Neuroimaging - Epilepsy

Discussion:
There is increased T2-weighted signal along the right medial hippocampus, and thinning of the cortical ribbon and atrophy in that
region best seen on the T1-weighted view. These finding are typical of mesial temporal sclerosis.

References:

RI Kuzniecky. CONTINUUM: Lifelong Learning in Neurology: Neuroimaging:175-187, August 2008..

Question 284: Neuroimaging - Spine

Discussion:

This MRI demonstrates an intradural, extramedullary dural-based enhancing mass that most likely represents a cervical
meningioma.

References:

Atlas S. Magnetic resonance imaging of the brain and spine. 3rd ed. Philadelphia: Williams and Wilkins, 2002.

Question 285: Neuroimaging - Brain Tumors

Discussion:

The patient has a midline mass of the hypothalamus which is a hypothalamic hamartoma. Pituitary macroadenoma are sellar
lesions. Craniopharyngioma generally have heterogeneous T1 signal and no contrast enhancement. Aneurysm would demonstrate
flow signal void. This is the typical appearance of the hamartoma of the tuber cinereum generally associated with premature
maturity, seizures, hyperactivity and rapid changes of weight.

References:

Greenberg JO, editor. Neuroimaging: a companion to Adams and Victor's principles of neurology. 2nd ed. New York: McGraw-
Hill, 1999.

Question 286: Neuroimaging - Critical Care/Trauma

Discussion:

The patient has a subacute right hemispheric subdural hematoma. She has been clinically stable, and does not likely have any
clinically significant findings as a result. Nonetheless, fairly urgent evaluation by neurosurgery would be a reasonable next step
to ensure close follow-up and if necessary surgical intervention can be entertained.

References:

Greenberg JO, editor. Neuroimaging: a companion to Adams and Victor's principles of neurology. 2nd ed. New York: McGraw-
Hill, 1999.

Question 287: Neuroimaging - Critical Care/Trauma

Discussion:

The patient had a subdural hematoma with mixed densities, some corresponding to a more recent bleeding. The shape of the
lesion and the low density of the deeper portion are uncharacteristic of an epidural hematoma, with a lens shape and the high
density of fresh blood. The lesion is outside the brain, compressing the cortex of the right hemisphere. For this reason, infarction
is not an option. A meningioma "en plaque" would be more homogeneous and have higher density.
References:

Greenberg JO, editor. Neuroimaging: a companion to Adams and Victor's principles of neurology. 2nd ed. New York: McGraw-
Hill, 1999.

Question 289: Neuroimaging - CSF Circulation Disorders

Discussion:

There is an ovoid area with decreased T2 along the anterior third ventricle, typical of a large flow void. The structure is also well
seen as an irregular ovoid vascular structure arising from the anterior communicating artery typical of a saccular aneurysm.

References:

Atlas S. Magnetic resonance imaging of the brain and spine. 3rd ed. Philadelphia: Williams and Wilkins, 2002.

Question 292: Neuroimaging - Spine

Discussion:

The cerebellar tonsils fit criteria for Chiari malformation. There is a cystic cavity in the cervical spinal cord, related to a syrinx.

References:

Ropper AH, Brown RH. Adams and Victor's principles of neurology. 8th ed. New York: McGraw-Hill, 2005.

Question 295: Neuroimaging - Epilepsy

Discussion:

The lesion in the left hippocampus has the imaging characteristics of a cavernous angioma, with a rim of low-intensity
hemosiderin surrounding a cluster of high-intensity, fluid-containing vessels. The absence of draining veins rules out both an
arteriovenous malformation and a venous angioma. Astrocytoma and oligodendroglioma do not have the imaging characteristics
present in this MRI.

References:

Greenberg JO. Neuroimaging: a companion to Adams and Victor's principles of neurology. New York: McGraw-Hill, 1999.

Question 301: Neuroimaging - Brain Tumors

Discussion:

Axial CTs show a large mass lesion filling the right orbit, compressing and deforming the posterior margin of the globe. The
inferior cut through the maxillary sinus shows that the lateral wall of the maxilla is destroyed and is associated with the soft tissue
mass. Metastatic disease commonly involves bone and adjacent soft tissue, as in this case. Facial trauma does not produce a
disappearance of bone but fractures it. Fibrous dysplasia expands bone but does not destroy it. Hemangioma may involve bone
and soft tissue, but is not a destructive process. In the orbit, capillary hemangiomas are commonly found in the cutaneous tissues
and periorbita but do not extend retroglobar and do not destroy bone. Cavernous hemangiomas are discrete masses that may be
found intraorbital or extraorbital but do not usually cross these margins.

References:
Zimmerman RA, Bilaniuk LT. Computed tomography of the orbit. In: Lee S, Rao K, editors. Cranial computerized tomography.
New York: McGraw-Hill, 1983.

Question 303: Neuroimaging - Stroke

Discussion:

The CT scan of the head demonstrates a hyperdense lesion anterior to the midbrain in the expected location of the basilar artery.
Computerized reformatted CT angiogram shows a basilar tip aneurysm.

References:

Atlas S. Magnetic resonance imaging of the brain and spine. 3rd ed. Philadelphia: Williams and Wilkins, 2002.

Question 304: Neuroimaging - Stroke

Discussion:

A modern CT using multidetector technique and reformatted images achieves greater than 95% sensitivity in the first 24 hours
after onset of nontraumatic subarachnoid hemorrhage. CSF circulation and resorption of the blood results in reduced sensitivity to
the hemorrhage to about 50% at 1 week.

References:

Edlow JA, Caplan LR. Avoiding pitfalls in diagnosis of subarachnoid hemorrhage. N Engl J Med 2000;342:29-36.

Question 307: Neuroimaging - Multiple Sclerosis

Discussion:

The patient has a lesion in the cervical cord consistent with transverse myelitis. Appropriate workup is necessary to exclude
infectious, rheumatologic, and nutritional causes of the lesion. However, given the clinical scenario and likelihood that this will
turn out to represent idiopathic transverse myelitis, initiation of steroids is indicated. There is no surgical lesion, no evidence of
spinal cord infarct, and this lesion in isolation does not currently fit criteria for starting interferon therapy or mitoxantrone.

References:

Ropper AH, Brown RH. Adams and Victor's principles of neurology. 8th ed. New York: McGraw-Hill, 2005.

Question 308: Neuroimaging - Dementia

Discussion:

The lesion in the left temporal lobe is isodense on T1 and enhances with gadolinium. It is outside the temporal lobe. It causes
compression of the tip and medial aspect of the temporal lobe, with partial herniation of the parahippocampal gyrus. These
features are characteristic of a meningioma. Both Alzheimer disease and frontotemporal dementia would present with atrophy,
not mass effect. Both an astrocytoma and a glioblastoma are intraparenchymal tumors.

References:

Greenberg JO. Neuroimaging: a companion to Adams and Victor's principles of neurology. New York: McGraw-Hill, 1999.

Question 313: Neuroimaging - Critical Care/Trauma


Discussion:

The scans show multiple small ring-enhancing lesions consistent with cerebral abscesses located at the supratentorial gray-white
junction. In addition, a posterior occipital enhancing lesion is noted. The appearance is most consistent with septic emboli. By the
imaging appearance alone, cerebral metastases are not ruled out. In the current case, the patient had endocarditis secondary to
intravenous drug abuse. Uncomplicated pyogenic meningitis typically shows intense meningeal enhancement of the cerebral
convexity, tentorium, and falx. Multiple sclerosis may show lesions with a similar appearence, but they would not involve the
cortex, as seen on the sagittal image, and the enhancement ring would tend to open to the side of the gray matter, not the white
matter as seen in the axial image of this case. Herpes encephalitis does not present this appearance.

References:

Greenberg JO, editor. Neuroimaging: a companion to Adams and Victor's principles of neurology. 2nd ed. New York: McGraw-
Hill, 1999.

Question 315: Neuroimaging - Developmental/Neurogenetic Disorders

Discussion:

No lipoma is present. Complete corpus callosal agenesis is demonstrated. There is a high association with other brain anomalies.
Patients with callosal complete agenesis have a decreased volume of white matter compared with normals, reflecting the 9% to
10% of brain volume comprised of the corpus. If isolated, there is no significant increase (not 90%) in seizures. The ventricles are
near to parallel and relatively straightened compared to the normal shape.

References:

Hetts, SW, Sherr EH, Chao S, Gobuty S, Barkovich AJ. Anomalies of the corpus callosum: an MR analysis of the phenotypic
spectrum of associated malformations. American Journal of Roentgenology 2006;187:1343-1348.

Question 316: Neuroimaging - Brain Tumors

Discussion:

The most likely diagnosis is glioblastoma multiforme. On the FLAIR image, the lesion presents with mass effect, too pronounced
for an arteriovenous malformation, choroid plexus calcification, or an infarction. The abnormal high-intensity region extends
across the corpus callosum to the right hemisphere, a feature common in malignant gliomas. The abnormal contrast enhancement
suggests that the tumor has abnormal vessels with marked permeability, another finding in glioblastoma. However,
macroscopically dilated vessels, characteristic of an arteriovenous malformation are not present. A choroid plexus papilloma is
very unusual in this age group. A toxoplasma abscess tends to show enhancement at the periphery and with a more regular
appearance than in the present case.

References:

Greenberg JO, editor. Neuroimaging: a companion to Adams and Victor's principles of neurology. 2nd ed. New York: McGraw-
Hill, 1999.

Question 318: Neuroimaging - Dementia

Discussion:

The most striking change between the two studies is the pronounced decrease in glucose metabolism in the frontal and anterior
temporal regions. This pattern is characteristic of frontotemporal dementia. Alzheimer disease tends to decrease metabolism in
the parietotemporal association cortex, relatively spared in the scan shown. Parkinson disease does not cause a decrease in
metabolism. As in this case, in Huntington chorea there is decreased metabolism in the head of the caudate nuclei, but the frontal
and temporal lobes are not as affected. Finally, diffuse Lewy body disease causes decreased metabolism in the occipital regions,
relatively spared in this scan.
References:

Coleman RE. Positron emission tomography diagnosis of Alzheimer's disease. Neuroimaging Clin N Am 2005;15:837-846.

Kantarci K, Jack Jr C. Neuroimaging in Alzheimer disease: an evidence-based review. Neuroimaging Clin N Am 2003;13:197-
209.

Masdeu JC, Zubieta JL, Arbizu J. Neuroimaging as a marker of the onset and progression of Alzheimer's disease. J Neurol Sci
2005;236:55-64.

Question 323: Neuroimaging - Technical

Discussion:

The North American Symptomatic Carotid Endarterectomy Trial (NASCET) defines % stenosis by the equation: % stenosis =
[(b-a)/b]x100% where b=lumen diameter measured above the carotid bulb (where vessel walls are parallel). The quantity a=the
point of maximally narrowed lumen. European carotid surgical trial (ECST) criteria require a subjective guess of where the
normal lumen wall might have been at the level of stenosis, if not diseased.

References:

Osborn, Anne G.. Diagnostic Cerebral Angiography 2nd Edition. 2nd Edition, Philadelphia:Lippincott Williams & Wilkins,
1999.

Question 326: Neuroimaging - Spine

Discussion:

The fourth ventricle is of normal size without cysts and the corpus callosum is definitely present. The tectal area is normal. The
cerebellar tonsils descend below the foramen magnum. The patient has Chiari type I malformation.

References:

Question 327: Neuroimaging - Stroke

Discussion:

The combination of the patient's physical exam and neuroimaging findings is consistent with a right anterior choroidal artery
infarction. The anterior choroidal artery is a branch of the internal carotid artery and supplies the posterior limb of the internal
capsule and the lateral thalamus. The paramedian thalamic artery is a branch of the posterior cerebral artery and supplies the
medial thalamus. The ophthalmic artery is a branch of the internal carotid artery and supplies the retina.

References:

Greenberg JO, editor. Neuroimaging: a companion to Adams and Victor's principles of neurology. 2nd ed. New York: McGraw-
Hill, 1999.

Question 328: Neuroimaging - Infection

Discussion:

There are multiple ring-enhancing lesions with adjacent vasogenic edema. The enhancing rings have relatively smooth walls
more commonly seen with infectious masses as compared to tumors which more commonly have more irregular and thicker
margins. The marked increase in diffusion weighted signal is more frequently seen with infections as compared to tumor. Acute
multiple sclerosis plaques more frequently have open ring configurations unlike these closed rings. Encephalitis generally
enhances along the cortical margin and not in the pattern of multiple masses. Gliomatosis cerebri is generally an infiltrative tumor
mass and not as multiple enhancing discontinuous masses.

References:

Greenberg JO, editor. Neuroimaging: a companion to Adams and Victor's principles of neurology. 2nd ed. New York: McGraw-
Hill, 1999.

Question 329: Neuroimaging - Brain Tumors

Discussion:

MR is effective in demonstrating a displaced optic chiasm because the optic chiasm is usually slightly more intense than pituitary
tumors.

References:

Daniels DL, Haughton VM, Czervionke LF. MR of the skull base. In: Bradley WG, Stark D. Magnetic resonance imaging. St.
Louis: Mosby, 1988.

Greenberg JO. Neuroimaging: a companion to Adams and Victor's principles of neurology. New York: McGraw-Hill, 1999.

Question 331: Neuroimaging - Developmental/Neurogenetic Disorders

Discussion:

The syndrome of septooptic dysplasia (de Morsier syndrome) consists of hypoplasia of the optic nerves, hypoplasia or absence of
the septum pellucidum. Clinical presentation may include nystagmus, diminished visual acuity and approximately two-thirds of
affected patients will also have hypothalamic-pituitary dysfunction. This MRI scan demonstrates absence of the septum
pellucidum, hypoplasia of the corpus callosum and hypotelorism (closely set eyes).

References:

Barkovich AJ. Pediatric neuroimaging. Pediatric Neuroimaging. 4th ed. Lippincott Williams & Wilkins; 2005

Question 333: Neuroimaging - Critical Care/Trauma

Discussion:

Pituitary apoplexy is a syndrome of infarction or hemorrhage of the pituitary gland. It is associated with headache,
ophthalmoparesis, visual defects, and encephalopathy. It may occur acutely or subacutely and can be associated with coma or
death. It occurs more often in patients with pituitary adenomas.

References:

Rogg JM, Tung GA, Anderson G, Cortez S. Pituitary apoplexy: early detection with diffusion-weighted MR imaging. AJNR Am
J Neuroradiol 2002;23:1240-1245.

Question 337: Neuroimaging - Spine

Discussion:

Although the MRI tends to rule out an astrocytoma and ependymoma because there is only mild mass effect, it could still be an
inflammatory lesion, giving rise to the increased signal on the T2 images. A heavily T2-weighted sequence (T2-CISS) shows a
cluster of vessels on the dorsal aspect of the cord compatible with venous dilation due to arteriovenous shunting by a dural
fistula. The selective angiogram confirms this impression because the venous system is filled so rapidly by a brief arterial
injection that both arterial and venous vessels are seen on the same angio frame. This disorder corresponds to the myelopathy of
Foix-Alajouanine.

References:

Greenberg JO, editor. Neuroimaging: a companion to Adams and Victor's principles of neurology. 2nd ed. New York: McGraw-
Hill, 1999.

Koch C. Spinal dural arteriovenous fistula. Curr Opin Neurol 2006;19:69-75.

Question 338: Neuroimaging - Spine

Discussion:

The odontoid process extends well above the level of the foramen magnum and rests adjacent to the anterior medulla. This is the
pattern of basilar invagination.

References:

Atlas S. Magnetic resonance imaging of the brain and spine. 3rd ed. Philadelphia: Williams and Wilkins, 2002.

Question 339: Neuroimaging - Infection

Discussion:

Both the clinical picture and the imaging suggest a subacute process, with quick worsening. Of the given options, listeriosis and
multiple sclerosis (MS) usually have a subacute course. Of the two, the imaging is more typical of listeriosis, an infection
produced by Listeria monocytogenes that has a preference for the brainstem. MS would typically not involve the 5th cranial
nerve. In the second study, the larger abscess in the tegmentum of the pons, visible in the gadolinium-enhanced study, is
accompanied by a subependymal cluster of smaller abscesses. In addition, there is enhancement of the ependymal lining,
suggesting the infectious nature of the lesion.

References:

Antal EA, Loberg EM, Dietrichs E, Maehlen J. Neuropathological findings in 9 cases of listeria monocytogenes brain stem
encephalitis. Brain Pathol 2005;15:187-191.

Falini A, Kesavadas C, Pontesilli S, et al. Differential diagnosis of posterior fossa multiple sclerosis lesions--neuroradiological
aspects. Neurol Sci 2001;22 (Suppl 2):S79-S83.

Question 342: Neuroimaging - Dementia

Discussion:

The patient has amyloid angiopathy, most commonly seen in patients with Alzheimer disease. The other choices have no
correlation with intraparenchymal microhemorrhages.

References:

Ropper AH, Brown RH. Adams and Victor's principles of neurology. 8th ed. New York: McGraw-Hill, 2005.

Question 343: Neuroimaging - CSF Circulation Disorders


Discussion:

The basilar artery is markedly dilated and can be seen in its typical location anterior to the midbrain. The left middle cerebral
artery can be seen to supply large feeders to the arteriovenous malformation. The arteriovenous malformation has a typical
appearance that is not seen in neoplasia. Soft neurological findings are common in this setting. The basilar artery can be seen
clearly and there is no corresponding vein to be confused with this structure. The arteriovenous malformation likely involves all
three major intracerebral vessels. Developmental venous malformation and cavernous hemangioma do not have enlarged arteries.
Superficial siderosis demonstrates pial and cortical T2 hypointensity with no vascular enlargement.

References:

Question 344: Neuroimaging - Brain Tumors

Discussion:

Chordomas arise from remnants of the embryonic notochord which is a mesodermal derivative. Cranial chordomas are most
common in the third and fourth decades of life and males are affected more often than females. Cartilaginous tumors occur
between 20 and 60 years of age. They are extradural and over half arise in or adjacent to the body of the sphenoid bone.
Radiologically the normal, high signal marrow cavity of the clivus is replaced by lower signal intensity tumor. Large areas of
calcification may be seen as void-phenomenon. Meningiomas are isointense to the brain before contrast. This is an extra pontine
lesion; the brainstem is not involved. The pituitary gland is normal. Parapharyngeal abscesses are usually smoothly contoured.

References:

Greenberg JO, editor. Neuroimaging: a companion to Adams and Victor's principles of neurology. 2nd ed. New York: McGraw-
Hill, 1999.

Question 346: Neuroimaging - Brain Tumors

Discussion:

Of the available options, temporal herniation through the incisura involves transtentorial herniation by the medial temporal
structures such as the uncus and is typically called uncal herniation. The central herniation syndrome, in which there is bilateral,
severe, downward herniation, is a type of transtentorial herniation syndrome. Tonsillar herniation, subfalcine herniation, and
ascending herniation syndromes do not involve the uncus.

References:

Question 404: Neuroimaging - Stroke

Discussion:

The principle of diffusion MRI is in the microscopic motion of water protons. In acute stroke settings there is a shift of
extracellular water into the intracellular space, thereby reducing the apparent diffusion coefficient (ADC).

References:

Question 417: Neuroimaging - Technical

Discussion:

Diffusion-weighted images are hyperintense and apparent diffusion coefficient (ADC) maps are hypointense in regions of
restricted diffusion of water in tissue relative to the surrounding normal brain. This occurs with cytotoxic edema in tissue
infarction and also in dense cellular collections, such as purulent accumulations. Diffusion-weighted imaging is valuable in
diagnosis of infection for this reason.
References:

Osborn A, Blaser S, Saltzman K, editors. Diagnostic imaging: brain. Salt Lake City: Amirsys, 2004.

Grossman RI, Yousem DM. Neuroradiology the requisites. 2nd Ed. Philadelphia: Mosby, 2003. ISBN 0-323-00508-X

Pathology

Question 10: Pathology - Critical Care/Trauma

Discussion:

Beta-amyloid precursor protein immunohistochemistry has been shown to be a highly sensitive marker for diffuse axonal injury,
which is the likely substrate of this patient's neurologic deficit, and is superior to other axonal stains.

References:

Love S, Louis DN, Ellison D. Greenfield's neuropathology. London: Hodder Arnold, 2008.

Question 32: Pathology - Demyelinating Disease

Discussion:

The serum autoantibody biomarker, NMO-IgG, targets a water channel protein, aquaporin-4. This is concentrated in astrocytic
foot processes.

References:

Lucchinetti, C. Advances in the neuropathology of multiple sclerosis: evolving pathogenic insights. Continuum: lifelong learning
in neurology 2007;13:86-118.

Question 33: Pathology - Neurodegenerative Disease

Discussion:

Familial frontotemporal dementia that arises from a mutation on chromosome 17 results in a mutation of the gene encoding for
tau protein.

References:

Ellison D, Love S, Chimelli L, et al. Neuropathology: a reference text of CNS pathology. 2nd ed. Edinburgh: Mosby, 2004.

Question 58: Pathology - Neurodegenerative Disease

Discussion:

The nucleus of Onufrowicz of the sacral cord is spared in motor neuron disease. The remaining structures or cells contain motor
nuclei and are involved to varying degrees in amyotrophic lateral sclerosis.

References:

Love S, Louis DN, Ellison D. Greenfield's neuropathology. London: Hodder Arnold, 2008.
Question 93: Pathology - Prion Disease

Discussion:

Prion diseases have long incubation times and evoke no host immune response. Immunocompromised individuals are not
preferentially affected by prion diseases. The CSF does not show pleocytosis (as it does in most other infections) and neither
inflammation or microglial nodules are seen in the brain. Spongiform change, intense gliosis, and neuronal loss unassociated with
mononuclear cell inflammation is the characteristic histologic pattern for Creutzfeldt-Jakob disease.

References:

Love S, Louis DN, Ellison DW. Greenfield's neuropathology. 8th ed. London: Hodder and Arnold, 2008.

Question 135: Pathology - Neurodegenerative Disease

Discussion:

TDP-43 has been identified as a major component of the inclusions of FTLD with ubiquitin-positive, tau- and alpha-synuclein-
negative inclusions (FTLD-U), formerly called FTLD with MND-type inclusions, but without MND. This protein now defines a
novel class of neurodegenerative diseases collectively called TDP-43 proteinopathies, and TDP-43 IHC may be used to
characterize a majority of FTLD-U's, but not all.

References:

Cairns NJ et al.. (2007) Neuropathologic diagnostic and nosologic criteria for frontotemporal lobar degeneration: consensus of
the Consortium for Frontotemporal Lobar Degeneration. Acta Neuropathol 114:2 22..

Question 163: Pathology - Hypothalamus/Pituitary

Discussion:

The findings are typical of pituitary apoplexy, which is most often encountered in the setting of acute necrosis and hemorrhage of
a large nonfunctioning pituitary adenoma. Worsening headache with nausea and sometimes vomiting are characteristic. Visual
disturbance is due to chiasmal compression. Pituitary apoplexy is a medical emergency, and acute care includes treatment with
corticosteroids and surgical intervention. Spontaneous hemorrhage and necrosis of the non-neoplastic pituitary gland may occur
post-partum but is less common, as are the other choices provided.

References:

Love S, Louis DN, Ellison DW. Greenfield's neuropathology. 8th ed. London: Hodder and Arnold, 2008.

Burger P, Scheithauer B, Vogel FS. Surgical pathology of the nervous system and its coverings. 4th ed. Philadelphia: Churchill
Livingstone, 2002.

Question 169: Pathology - Basic Reactions

Discussion:

Although the etiology of hippocampal (Ammon horn) sclerosis is controversial, with some authors contending the lesions are the
cause, and others, the result of the seizures, the lesion is nonetheless the most common pathology found in patients with tissue
resected for temporal lobe epilepsy. Seizure control after temporal lobectomy is best with hippocampal sclerosis but is also very
good with neoplasms and vascular malformations, although these are less common causes of temporal lobe epilepsy. End folium
sclerosis with neuronal loss confined to the CA4 sector of the hippocampus is rare, as is Rasmussen encephalitis.

References:
Burger P, Scheithauer B, Vogel FS. Surgical pathology of the nervous system and its coverings. 4th ed. Philadelphia: Churchill
Livingstone, 2002.

Love S, Louis DN, Ellison DW. Greenfield's neuropathology. 8th ed. London: Hodder and Arnold, 2008.

Question 181: Pathology - Demyelinating Disease

Discussion:

The genetic basis and pathophysiology of most of the leukodystrophies have been known for some time. Light has recently been
shed on one of the last holdouts: Alexander disease. The morphologic hallmark of Alexander disease is the presence of profuse
numbers of Rosenthal fibers. Rosenthal fibers are composed of densely compacted glial intermediate filaments made of glial
fibrillary acidic protein (GFAP). Recent studies have demonstrated that a large percentage of Alexander disease cases are
associated with a mutation in the GFAP gene.

References:

Brenner M, Johnson AB, Boespflug-Tanguy O, et al. Mutations in GFAP, encoding glial fibrillary acidic protein, are associated
with Alexander disease. Nat Genet 2001;27:117-120.

Messing A, Goldman JE, Johnson AB, Brenner M. Alexander disease: new insights from genetics. J Neuropathol Exp Neurol
2001;60:563-573.

Fuller GN, Goodman JC. Practical review of neuropathology. Philadelphia: Lippincott Williams & Wilkins, 2001.

Question 191: Pathology - Neurodegenerative Disease

Discussion:

The epsilon 4 variant of APOE is the only established risk factor for late-onset Alzheimer disease (AD). Mutations in genes for
amyloid precursor protein and presenilin 1 and 2 are associated with early-onset familial AD. Hyperphosphorylated forms of tau
accumulate in many forms of frontotemporal dementia but genetic variants of tau have not been identifed as risk factors for late-
onset AD.

References:

Dickson D. Neurodegeneration: the molecular pathology of dementia and movement disorders.. Basel: ISN Neuropath Press,
2003.

Bertram L, Tanzi R.. Thirty years of Alzheimer's disease genetics: the implications of systematic meta-analyses.. Nature Rev
Neurosci 2008;9:768-78.

Question 231: Pathology - Cerebrovascular Disease

Discussion:

The lesion is seen in the cortex of the gyri to the right of midline in this coronal section and is a typical example of laminar
necrosis due to hypoxic/ischemic injury.

References:

Schochet SS, Gray F. Acquired metabolic disorders. In: Gray F, De Girolami U, Poirier J. Escourolle and Poirier manual of basic
neuropathology. 4th ed. Boston: Butterworth-Heinemann 2004; 198.

Question 235: Pathology - Cerebrovascular Disease


Discussion:

The photograph shows a massive basal ganglionic hemorrhage with rupture into the ventricular system. The location of this
hematoma is common for hypertensive intracerebral hemorrhage. Hypertension is a major risk factor for this type of hemorrhage,
and hypertension is associated with pathologic and electrocardiographic evidence of left ventricular hypertrophy. Prostate cancer
only rarely metastasizes to brain parenchyma and is usually not hemorrhagic. Visceral cysts are not relevant to intracerebral
hemorrhage, but are frequently seen in von Hippel-Lindau disease with associated CNS hemangioblastoma.

References:

Love S, Louis DN, Ellison D. Greenfield's neuropathology. London: Hodder Arnold, 2008.

Question 236: Pathology - Demyelinating Disease

Discussion:

Inflammatory demyelination seen in multiple sclerosis originates in perivenular regions.

References:

Love S, Louis DN, Ellison DW. Greenfield's neuropathology. 8th ed. London: Hodder and Arnold, 2008.

Question 239: Pathology - Infectious Disease

Discussion:

The photomicrograph shows acutely branching septate hyphae which most likely represent an opportunistic aspergillosis
infection. Patients with aspergillosis frequently have neutropenia and are receiving immunosuppressive therapy for neoplasms.
The organism initially infects the lung and then spreads hematogenously to involve other organs. The brain is second only to the
lung as a site of visceral involvement, and frequently manifests as hemorrhagic infarctions.

References:

Love S, Louis DN, Ellison D. Greenfield's neuropathology. London: Hodder Arnold, 2008.

Question 240: Pathology - Neurodegenerative Disease

Discussion:

Pick disease is characterized by the presence of spherical tau-immunoreactive cytoplasmic inclusions ("Pick bodies") that are
particularly abundant in the dentate gyrus of the hippocampus. Such inclusions are also frequent in CA1 but much less common
in CA2-4 and in the presubiculum in Pick disease. None of the other "tauopathies" mentioned (Alzheimer disease, argyrophilic
grain disease, corticobasal degeneration, or progressive supranuclear palsy) have such prominent Pick bodies in the dentate
fascia.

References:

Dickson D. Neurodegeneration: the molecular pathology of dementia and movement disorders.. Basel: ISN Neuropath Press,
2003.

Question 241: Pathology - Cerebrovascular Disease

Discussion:
The gross findings are those of a remote hemorrhagic stroke. Microscopically, the blood vessels of the meninges and superficial
cortex showed amorphous eosinophilic material indicative of cerebral amyloid angiopathy. Neither primary nor secondary
systemic amyloidoses cause amyloid deposits within cerebral blood vessels. Cerebral autosomal dominant arteriopathy with
subcortical infarcts and leukoencephalopathy (CADASIL) deposits would not stain with amyloid stains such as thioflavin S or
Congo red and would not be as homogeneous or affect the superficial cortex and meninges. Chronic hypertension causes
thickened walls of small blood vessels due to hyalinization and lipohyalinosis; no amyloid is deposited.

References:

Graham DI, Lantos PL. Greenfield's neuropathology. 7th ed. London: Arnold, 2002.

Question 248: Pathology - Critical Care/Trauma

Discussion:

The image shows a colloid cyst of the third ventricle. Such lesions may cause intermittent hydrocephalus due to a "ball-valve"
effect, thus causing positional headaches. Sudden death has been reported with this "benign" lesion.

References:

Burger P, Scheithauer B, Vogel FS. Surgical pathology of the nervous system and its coverings. 4th ed. Philadelphia: Churchill
Livingstone, 2002.

Question 252: Pathology - Neuromuscular Disease

Discussion:

The photomicrograph shows a Trichinella spiralis organism acquired by ingestion of undercooked pork.

References:

Love S, Louis DN, Ellison DW. Greenfield's neuropathology. 8th ed. London: Hodder and Arnold, 2008.

Question 260: Pathology - Neurodegenerative Disease

Discussion:

The histologic hallmarks of Alzheimer disease are neuritic plaques, neurofibrillary tangles, Hirano bodies, neuronal
granulovacuolar degeneration, and the deposition of amyloid in the walls of blood vessels. The pyramidal neuron illustrated
showed granulovacuolar degeneration, a neurofibrillary tangle, and a Hirano body.

References:

Cochran EJ. Neurodegenerative Diseases. In: Prayson R, editor. Neuropathology: a volume in the foundations in diagnostic
pathology series. Philadelphia: Elsevier, 2005.

Question 265: Pathology - Developmental

Discussion:

This brain shows multiple areas of cavitation (liquefaction necrosis) involving the deeper cortical layers and underlying white
matter, a condition known as multicystic encephalopathy or multicystic encephalomalacia. The lesions are most pronounced in
the distribution of the anterior and middle cerebral arteries. They are the result of circulatory disturbances during the latter half of
pregnancy or the neonatal period.
References:

Nelson J, Mena H, Parisi JE, Schochet SS, editors. Principles and practice of neuropathology. 2nd ed. New York: Oxford
University Press, 2003.

Question 269: Pathology - Neurodegenerative Disease

Discussion:

The graphic shows severe pontine atrophy and would most likely be seen in olivopontocerebellar atrophy (OPCA). OPCA is part
of multiple system atrophy. Glia, (especially oligodendroglia), in multiple system atrophy exhibit cytoplasmic, flame-shaped,
silver-positive inclusions. The inclusions are also immunoreactive for ubiquitin and alpha-synuclein.

References:

Cochran EJ. Neurodegenerative Diseases. In: Prayson R, editor. Neuropathology: a volume in the foundations in diagnostic
pathology series. Philadelphia: Elsevier, 2005.

Love S, Louis DN, Ellison DW. Greenfield's neuropathology. 8th ed. London: Hodder and Arnold, 2008.

Question 270: Pathology - Toxic/Metabolic Disease

Discussion:

Kernicterus, meaning yellow nuclei, is seen in infants who die with severe neonatal jaundice. Bilirubin is a breakdown product of
blood that is normally conjugated in the liver to form bilirubin diglucuronide, which then is carried into bile and the intestine and
converted to urobilinogens by gut flora. Neonates have more unconjugated bilirubin since they initially lack the enzymes
necessary to convert the unconjugated bilirubin. The unconjugated bilirubin passes into selected nuclei, especially globus
pallidus, subthalamus and Ammon's horn in term infants. How precisely this causes cellular injury is uncertain.

References:

Love S, Louis DN, Ellison DW. Greenfield's neuropathology. 8th ed. London: Hodder and Arnold, 2008.

Question 273: Pathology - Neuromuscular Disease

Discussion:

Tomaculous neuropathy (also known as pressure sensitive neuropathy) is characterized by focal hypermyelination (tomacula)
with redundant myelin folds best seen on teased-fiber nerve preparations. Afflicted individuals have susceptibility to pressure
palsies following relatively trivial compression.

References:

Love S, Louis DN, Ellison DW. Greenfield's neuropathology. 8th ed. London: Hodder and Arnold, 2008.

Midroni G, Bilbao JM. Biopsy diagnosis of peripheral neuropathy. Boston: Butterworth-Heinemann, 1995.

Question 277: Pathology - Cerebrovascular Disease

Discussion:

The graphic illustrates bilateral, nearly symmetric, parasagittal, cortical and basal ganglia venous infarctions due to thrombosis of
the superior sagittal sinus and deep cerebral veins.
References:

Love S, Louis DN, Ellison DW. Greenfield's neuropathology. 8th ed. London: Hodder and Arnold, 2008.

Question 278: Pathology - Demyelinating Disease

Discussion:

The spinal cord shows tract degeneration in the dorsal spinocerebellar columns and posterior columns, with myelin loss
highlighted by the Luxol fast blue-periodic acid Schiff stain for myelin. Tract degeneration is seen in Friedreich ataxia, but not
multiple sclerosis or acute disseminated encephalomyelitis. Vanishing white matter disease predominantly affects the cerebrum.

References:

Love S, Louis DN, Ellison DW. Greenfield's neuropathology. 8th ed. London: Hodder and Arnold, 2008.

Graham DI, Lantos PL. Greenfield's neuropathology. 7th ed. London: Arnold, 2002.

Question 279: Pathology - Epilepsy

Discussion:

Examination of the cortex shows a Lafora body, characterized by a rounded, targetoid inclusion often found in the cerebellar
cortex. This microscopic feature is seen with Lafora body disease, myoclonic epilepsy.

References:

Love S, Louis DN, Ellison DW. Greenfield's neuropathology. 8th ed. London: Hodder and Arnold, 2008.

Question 282: Pathology - Toxic/Metabolic Disease

Discussion:

The biopsy shows a severe vacuolar myopathy due to extensive glycogen storage. These findings are consistent with Pompe
disease (glycogen storage disease type II), which is caused by a genetic deficiency of acid-alpha-glucosidase (acid maltase). In
it's most severe form, the disease presents in infancy with cardiomegaly, hepatomegaly, progressive muscle weakness,
macroglossia, and hypotonia ("floppy baby"). Of the choices provided, Pompe disease is the only disorder that presents findings
of vacuolar myopathy on a muscle biopsy. Pathologic findings of Haltia-Santavuori disease, (infantile neuronal ceroid
lipofuscinosis), and Alexander disease primarily affect the brain. Krabbe disease is a leukodystrophy affecting CNS white matter.
Kufs disease is an adult form of neuronal ceroid lipofuscinosis.

References:

Love S, Louis DN, Ellison DW. Greenfield's neuropathology. 8th ed. London: Hodder and Arnold, 2008.

Question 290: Pathology - Toxic/Metabolic Disease

Discussion:

A coronal section of the brain at autopsy shows mammillary bodies that are congested and discolored, which is a characteristic
finding in thiamine deficiency manifesting as Wernicke encephalopathy. Few other disorders affect the mamillary bodies.
Vitamin B12 deficiency can cause subacute combined degeneration, a spinal cord lesion. Methanol poisoning classically causes
hemorrhagic putaminal lesions.
References:

Agamanolis DP. Metabolic and toxic disorders. In: Prayson RA, Goldblum JR, editors. Neuropathology. Philadelphia: Elsevier,
Churchill, Livingstone 2005.

Question 294: Pathology - Basic Reactions

Discussion:

The muscle shows the features of denervation atrophy, which include clusters of muscle nuclei (so called "nuclear clumps")
within atrophic fibers. These changes should not be mistaken for inflammation or neoplasm.

References:

Cohen MA. Skeletal muscle and peripheral nerve disorders. In: Prayson RA, editor. Neuropathology: a volume in the foundations
in diagnostic pathology series. Philadelphia: Elsevier, 2005.

Question 296: Pathology - Tumors

Discussion:

The microscopic appearance is that of a capillary-rich neoplasm with abundant foamy cells containing lipid. This is a
hemangioblastoma. The cell origin of the hemangioblastoma is still unknown, despite many investigative efforts. The tumor
arises independently or in conjunction with von Hippel-Lindau disease. The most common location is in the cerebellum, but the
spinal cord can also be a site of origin. Pilocytic astrocytomas, also commonly found in the cerebellum, are composed of
astrocytes with elongate, bipolar, eosinophilic cell processes; these tumors often show Rosenthal fibers. Medulloblastomas are
malignant small blue cell tumors of neuronal origin that manifest little visible cytoplasm. Ependymomas are glial tumors which
show formation of ependymal canals, or more commonly, perivascular pseudorosettes.

References:

Burger P, Scheithauer B, Vogel FS. Surgical pathology of the nervous system and its coverings. 4th ed. Philadelphia: Churchill
Livingstone, 2002.

Question 297: Pathology - Tumors

Discussion:

The photo shows a poorly circumscribed variegated mass with foci of necrosis and hemorrhage most consistent with
glioblastoma multiforme. The lack of central purulent material is against an abscess. The lesion is not confined within a single
vascular territory, nor is it hemorrhagic, arguing against embolic infarct. The intra-axial location argues against most
meningiomas. The lesion is a mass, ruling out Huntington disease.

References:

Louis DN, Ohgaki H, Wiestler OD, Cavenee WK, editors. WHO classification of tumors of the central nervous system. 4th ed.
Lyon: International Agency for Research on Cancer, 2007.

Question 298: Pathology - Neuromuscular Disease

Discussion:

The muscle biopsy specimen shows a necrotizing vasculitis with fibrinoid necrosis of the vessel walls, not amyloid angiopathy.
Vasculitis may be seen in amphetamine-induced vasculitis, polyarteritis nodosa, rheumatoid vasculitis, and Wegener
granulomatosis, but not in polymyalgia rheumatica which is associated with temporal arteritis and usually type II fiber atrophy in
the muscle biopsy, not vasculitis. Takayasu arteritis affects large blood vessels such as aortic arch, not small intramuscular
vessels. ALS causes no inflammation of intramuscular blood vessels, but does show neurogenic atrophy.

References:

Love S, Louis DN, Ellison DW. Greenfield's neuropathology. 8th ed. London: Hodder and Arnold, 2008.

Alan Pestronk, MD, website author. Neuromuscular Disease Center.


http://www.neuro.wustl.edu/NEUROMUSCULAR/antibody/pnimax.html#pan Accessed November 1, 2007.
http://www.neuro.wustl.edu/NEUROMUSCULAR/time/nmacute.htm#neuropathy Accessed November 1, 2007.

Engel AG, Franzini-Armstrong C. Myology. 3rd ed. New York: McGraw-Hill, 2004.

Question 299: Pathology - Cerebrovascular Disease

Discussion:

The lesion shown in this image is a remote infarct. The preservation of the outer layer of the cortex would argue against a remote
contusion. An old hematoma cavity would be smoother, and would not necessarily be confined to the vascular distribution
territory of a branch of the middle cerebral artery as this lesion is. Both an abscess and metastatic carcinoma would be
accompanied by brain swelling rather than shrinkage; note the lack of mass effect on the ventricular system.

References:

Love S, Louis DN, Ellison D. Greenfield's neuropathology. London: Hodder Arnold, 2008.

Question 302: Pathology - Epilepsy

Discussion:

The dysembryoplastic neuroepithelial tumor (DNT) is composed of neurons and oligodendroglial-like cells clustered within
intracortical nodules. A prominent myxoid stroma is typically present within the tumor nodules. This indolent tumor is frequently
associated with cortical migration abnormalities. Surgery is curative.

References:

Perry A. Glial and glioneuronal tumors. In: Prayson RA. Neuropathology: a volume in the foundations in diagnostic pathology
series. Philadelphia: Elsevier, 2005.

Question 306: Pathology - Infectious Disease

Discussion:

The photograph shows abundant fibrinopurulent exudate in the subarachnoid space next to the cerebral surface (lower right of
image). This is most consistent with acute purulent leptomeningitis. Haemophilus influenzae rarely causes meningitis in the
adult, and pneumococcal meningitis is more common than meningococcal meningitis in the elderly patient. With this history of
alcoholism and asplenism, the most likely diagnosis is pneumococcal meningitis.

References:

Nelson J, Mena H, Parisi JE, Schochet SS, editors. Principles and practice of neuropathology. 2nd ed. New York: Oxford
University Press, 2003.

Question 309: Pathology - Hypothalamus/Pituitary


Discussion:

The image shows non-necrotizing granulomas with multinucleated giant cells. All other choices for this question, such as
pituicytoma, optic nerve meningioma, and spindle cell oncocytoma, were neoplasms, not inflammatory conditions and wouldn't
contain granulomas. The differential diagnosis for lesions in this anatomic location does include all of the items in the question
since optic nerve can harbor gliomas or meningiomas and the pituitary/hypothalamic region can be the site of pituitary adenoma,
pituicytoma or spindle cell oncocytoma.

References:

Prayson RA. Non-glial tumors. In: Prayson RA, editor. Neuropathology: a volume in the foundations in diagnostic pathology
series. Philadelphia: Elsevier, 2005.

Question 310: Pathology - Developmental

Discussion:

The face demonstrates cyclopia with a fused single eyeball and a superior proboscis. Cyclopia occurs with midline cleavage
defects and clefting of the lip or the palate is usually also present. Alobar holoprosencephaly is also part of this spectrum of
midline cleavage abnormalities with a single, globular hemisphere, a single ventricle and fused basal ganglia.

References:

Love S, Louis DN, Ellison DW. Greenfield's neuropathology. 8th ed. London: Hodder and Arnold, 2008.

Question 312: Pathology - Tumors

Discussion:

The photograph shows a biphasic cellular population consisting of small reactive lymphocytes and large neoplastic germ cells,
which is characteristic of germinoma - the most common pineal region tumor. Pineocytomas, in contrast, are composed of small
mature pineocytes that form large rosettes with fibrillary cores. Pineoblastomas are densely cellular primitive neuroectodermal
tumors that often form Homer Wright (medulloblastoma-type) rosettes and occasionally fleurettes. Pineal astrocytomas show
prominent eosinophilic cytoplasmic processes. Regarding terminology, the designation "pinealoma" is an inaccurate, arcane and
imprecise historical relic that should not be used; it dates to a time before a distinction was made between germinoma and
pineoblastoma.

References:

Louis DN, Ohgaki H, Wiestler OD, Cavenee WK, editors. WHO classification of tumors of the central nervous system. 4th ed.
Lyon: International Agency for Research on Cancer, 2007.

Question 319: Pathology - Developmental

Discussion:

A significant percentage of neural tube defects are associated with maternal folate deficiency, and reduced rates of this congenital
abnormality occur in women given folate. The thymus is enlarged in anencephalics due to lack of adrenal-driven involution,
which in turn results from absence or derangement of the hypothalamic-pituitary axis. The risk of anencephaly in subsequent
pregnancies is 5% to 7%. Females are more commonly afflicted and there is no compelling evidence of a role of maternal
exposure to toxins in the genesis of this condition.

References:

Love S, Louis DN, Ellison DW. Greenfield's neuropathology. 8th ed. London: Hodder and Arnold, 2008.
Question 321: Pathology - Neurodegenerative Disease

Discussion:

In the figure, myelin-stained cross sections of the spinal cord show pallor and Wallerian degeneration of the corticospinal tracts
with sparing of the posterior columns and spinocerebellar tracts. These findings are most compatible with amyotrophic lateral
sclerosis.

References:

Cochran EJ. Neurodegenerative Diseases. In: Prayson R, editor. Neuropathology: a volume in the foundations in diagnostic
pathology series. Philadelphia: Elsevier, 2005.

Question 322: Pathology - Infectious Disease

Discussion:

The CSF findings of pressure elevation, hypoglycorrhachia, mildly elevated protein, and mononuclear pleocytosis are most
consistent with chronic meningitis due to fungi or mycobacteria. The photomicrograph shows a microorganism located in a cell
within the subarachnoid space. This patient was a 55-year-old man from Arizona who presented with chronic headaches. Fungal
infections with Coccidioides occur in regions of semi-arid climate. In the United States, the Southwest and California are the
most common locations for these infections. Mature fungi in tissue are endospores, such as the one pictured, which are round and
have a refractile wall.

The reduced glucose is especially helpful in distinguishing viral infections such as herpes encephalitis and progressive multifocal
leukoencephalopathy from chronic fungal or tuberculous meningitis.

References:

Love S, Louis DN, Ellison DW. Greenfield's neuropathology. 8th ed. London: Hodder and Arnold, 2008.

Question 330: Pathology - Cerebrovascular Disease

Discussion:

The myelin-stained section at the level of the inferior olivary nuclei shows infarction (pallor on this myelin stain) of the
dorsolateral portion of the medulla in the distribution of the posterior inferior cerebellar artery. This usually results from
occlusion of the vertebral artery and can present clinically as "lateral medullary syndrome".

References:

Love S, Louis DN, Ellison DW. Greenfield's neuropathology. 8th ed. London: Hodder and Arnold, 2008.

Question 332: Pathology - Critical Care/Trauma

Discussion:

The graphic shows an acute subdural hematoma. Subdural hematomas result from disruption of bridging veins and are seen more
commonly in older persons with cortical atrophy with resultant tension on these veins. The inciting trauma often is a fall, but may
not be recalled.

References:

Love S, Louis DN, Ellison DW. Greenfield's neuropathology. 8th ed. London: Hodder and Arnold, 2008.
Graham DI, Lantos PL. Greenfield's neuropathology. 7th ed. London: Arnold, 2002.

Question 335: Pathology - Neuromuscular Disease

Discussion:

The illustration depicts a myofiber with red subsarcolemmal staining on the Gomori Trichrome preparation. This histologic
appearance of a ragged-red fiber corresponds to abnormal subsarcolemmal mitochondrial accumulations. Although ragged-red
fibers can occasionally be seen in other conditions, they are most commonly observed in the mitochondrial myopathies. Electron
microscopic examination in such cases confirms the presence of abnormal or too numerous mitochondria. Of the choices offered
in the question, only Kearns-Sayre syndrome is a mitochondrial myopathy. Werdnig-Hoffman is a motor neuron disease;
Duchenne muscular dystrophy is an X-linked-inherited myopathy involving abnormalities of dystrophin; central core myopathy
is a genetic condition where muscle biopsy demonstrates the presence of core-structures in the myofibrils; and dermatomyositis is
an autoimmune inflammatory myopathy.

References:

Love S, Louis DN, Ellison DW. Greenfield's neuropathology. 8th ed. London: Hodder and Arnold, 2008.

Question 336: Pathology - Tumors

Discussion:

The image shows classic Antoni A histology of a schwannoma with Verocay bodies.

References:

Burger P, Scheithauer B, Vogel FS. Surgical pathology of the nervous system and its coverings. 4th ed. Philadelphia: Churchill
Livingstone, 2002.

Question 340: Pathology - Tumors

Discussion:

The biopsy of this cystic mass showed layers of flattened, anucleate squamous cells. This type of flaky keratin is characteristic of
epidermoid cysts. In contrast, the cyst contents of adamantinomatous craniopharyngiomas typically consist of nodular clusters of
plump necrotic keratinocytes referred to as wet keratin. The lining of Rathke cleft cysts, colloid cysts, and neurenteric cysts
consists of ciliated pseudostratified columnar epithelium with scattered goblet cells. These cysts have watery or mucinous
contents, but no keratin.

References:

Prayson RA. Non-glial tumors. In: Prayson RA, editor. Neuropathology: a volume in the foundations in diagnostic pathology
series. Philadelphia: Elsevier, 2005.

Question 357: Pathology - Demyelinating Disease

Discussion:

Megalencephaly is seen in a wide variety of conditions including some other inborn errors of metabolism, but among the choices
the only correct answer would be Canavan disease.

References:

Love S, Louis DN, Ellison DW. Greenfield's neuropathology. 8th ed. London: Hodder and Arnold, 2008.
Question 413: Pathology - Neuromuscular Disease

Discussion:

Onion bulbs are the result of repeated episodes of demyelination and remyelination and are composed of concentric rings of
Schwann cells. They are prominent in demyelinating neuropathies such as Charcot-Marie-Tooth disease type 1A.

References:

Love S, Louis DN, Ellison D. Greenfield's neuropathology. London: Hodder Arnold, 2008.

Question 427: Pathology - Demyelinating Disease

Discussion:

The active plaque in multiple sclerosis is characterized by destruction of myelin with macrophages in the lesion containing both
early and late myelin degradation products. These macrophages must be present in order to diagnose the lesion as being "active".
In chronic active plaques a few macrophages are scattered throughout the lesion but most are concentrated at the perimeter of the
lesion near the expanding plaque edge.

References:

Lucchinetti, C. Advances in the neuropathology of multiple sclerosis: evolving pathogenic insights. Continuum: lifelong learning
in neurology 2007;13:86-118.

Question 433: Pathology - Neuromuscular Disease

Discussion:

Although increased internal nuclei are seen in many myopathies, this alteration is particularly noteworthy in myotonic dystrophy
in which about 30% to 60% of myofibers may bear internal nuclei. As in all muscle biopsies, care must be taken not to sample
the muscle too close to a tendinous insertion as this area is normally rich in internal nuclei.

References:

Love S, Louis DN, Ellison DW. Greenfield's neuropathology. 8th ed. London: Hodder and Arnold, 2008.

Pharmacology/Chemistry

Question 14: Pharmacology/Chemistry - Movement Disorders

Discussion:

Of the choices, amantadine is the only agent with demonstrable anti-dyskinetic effect, sometimes requiring higher doses than for
typical anti-Parkinson effects.

References:

Olanow CW, Stern MB, Sethi K. The scientific and clinical basis for the treatment of Parkinson disease. Neurology 2009;72
(Suppl 4):S1-S136.

Question 15: Pharmacology/Chemistry - Demyelinating Disorders

Discussion:
Fampridine is an inhibitor of voltage-dependent potassium channels that improves conduction along demyelinated axons of the
central nervous system in animal studies. Several studies in patients with multiple sclerosis have indicated that treatment with
fampridine may be associated with improvements in visual function, strength, ambulation, fatigue, and endurance.

References:

AD Goodman, TR Brown, JA Cohen, et al.. Dose comparison trial of sustained-release fampridine in multiple sclerosis.
Neurology 2008;71:1134-1141

Question 18: Pharmacology/Chemistry - Neuromuscular Disorders

Discussion:

Myotonia (impaired relaxation of muscles) is associated with several disorders and is a hallmark symptom of type I Myotonic
Dystrophy. Myotonia is commonly treated by various Na channel blockers such as mexiletine, phenytoin, and carbamazepine, but
can be limited due to undesirable side effects, especially the antiarrhythmics.

References:

Kurihara T.. New classification and treatment for myotonic disorders. Intern. Med. 2005 Oct;44(10):1027-32

Question 22: Pharmacology/Chemistry - Cerebrovascular Disease

Discussion:

One of the contraindications to the use of recombinant tissue type plasminogen activator in acute stroke therapy is if the patient is
taking an anticoagulant, then patient taking warfarin with international normalized ratio must be less than 1.7. Other
contraindications include major surgery in the 14 days prior to stroke; platelet count less than 100,000 / mm3; CT showing no
multilobar infarction (hypo density greater than one-third cerebral hemisphere); onset of symptoms greater than 180 minutes
prior to administration.

References:

Khatri P, Levine J, Jovin T. Intravenous thrombolytic therapy for acute ischemic stroke. Continuum: lifelong learning neurology
2008;14:46-60.

Question 36: Pharmacology/Chemistry - Movement Disorders

Discussion:

Amiodarone has been associated with drug-induced postural tremor. Beta-blockers such as propanolol and sotalol may improve
postural tremor. Diltiazem and digoxin have not been associated with tremor.

References:

Louis ED. Treatment of tremor. In Frucht S, editor. Movement disorders. Continuum: lifelong learning in neurology 2007;13:58-
59, 63-67.

Question 38: Pharmacology/Chemistry - Toxic/Metabolic Disease

Discussion:

Poisoning with ethylene glycol or methanol can occur through attempted inebriation, unintentional ingestion, or intentional self-
harm. Methanol poisoning most often occurs from the ingestion of windshield-washer fluid. Methanol is also used in copy
machines and as an ingredient in canned heating products, embalming fluids, and paint removers. Methanol poisoning is a well-
known consequence of ingesting moonshine liquor.

Methanol is metabolized to formaldehyde and formic acid by alcohol dehydrogenase. Formic acid is believed to be the cause of
retinal toxicity, that mainly affects the ganglion cells. Methanol itself is not responsible for the major adverse effects of its
ingestion; rather, it is metabolized to formaldehyde, which is subsequently oxidized to formic acid. Formic acid is the cause of
the retinal and optic-nerve damage seen in patients who survive serious methanol poisoning. Patients usually have metabolic
acidosis, but Kussmaul respiration is uncommon because of the respiratory depression caused by the intoxication. Fomepizole is
an inhibitor of alcohol dehydrogenase that appears to be safe and effective in managing methanol poisoning.

References:

Brent J. Fomepizole for ethylene glycol and methanol poisoning. N Engl J Med 2009;360:2216-2223.

Question 40: Pharmacology/Chemistry - Other Pain Syndromes

Discussion:

Pregabalin was approved in 2007 by the US Food and Drug Administration (FDA) as the first drug for the treatment of
fibromyalgia, effective in reducing symptoms of pain, disturbed sleep and fatigue. Pregabalin had been approved earlier by the
FDA for the treatment of neuropathic pain associated with diabetic peripheral neuropathy, postherpetic neuralgia, and as
adjunctive therapy for refractory partial seizures. Pregabalin also has proven efficacy in adjunctive therapy of generalized anxiety
disorder, social anxiety disorder, and acute pain. It has been demonstrated that although pregabalin is a structural derivative of the
inhibitory neurotransmitter GABA, it does not bind directly to GABAA, GABAB, or benzodiazepine receptors; it does not
augment GABAA responses in vitro, nor does it alter GABA concentration in the animal brain, or have acute effects on GABA
uptake or degradation. Pregabalin does not block sodium channels, is not active at opiate receptors, and does not alter
cyclooxygenase enzyme activity. It is inactive at serotonin and dopamine receptors and does not inhibit dopamine, serotonin, or
noradrenaline reuptake.

However, a single common mechanism, inhibition of calcium currents via high-voltage-activated channels containing the
alpha2delta-1 subunit, leading in turn to reduced neurotransmitter release and attenuation of postsynaptic excitability, is believed
to have a predominate effect on its mechanism of action.

References:

Zareba G. New treatment options in the management of fibromyalgia: role of pregabalin. Neuropsychiatr Dis Treat 2008;4:1193-
1201.

Sills GJ. The mechanisms of action of gabapentin and pregabalin. Curr Opin Pharmacol 2006;6:108-113.

Question 50: Pharmacology/Chemistry - Movement Disorders

Discussion:

Tardive dyskinesia (TD) primarily involves the tongue, lips,and jaw. A combination of tongue twisting and protrusion, lip
smacking and puckering, and chewing movements in a repetitive and stereotypic fashion is often observed. TD results from
chronic exposure to dopamine receptor blocking agents drugs primarily used to treat psychosis. TD has not been reported with
agents that deplete dopamine (such as reserpine) and is only rarely reportedwith   atypical  antipsychotic drugs (such as
clozapine). Some drugs for nausea (such as metoclopramide or prochlorperazine) and depression (such as amoxapine) are
actually dopamine receptor blocking agents and therefore can cause TD.

The weight of evidence favors the notion that chronic blockade of dopamine receptors leads to increased receptor sensitivity.
After 2 weeks of therapy with dopamine receptor blocking agents, the affinity and number of dopamine D2 receptors increased in
rodent models of TD. Yet, it remains unclear how this leads to the development of TD, or why only a minority of patients with
similar drug exposure develop TD.

References:
Soares-Weiser K, Fernandez HH. Tardive dyskinesia. Semin Neurol 2007;27:159-169.

Question 62: Pharmacology/Chemistry - Epilepsy

Discussion:

Both lamotrigine and carbamazepine are associated with black box warnings from the manufacturers, in conjunction with the
FDA, concerning rash.

References:

http://www.fda.gov/cder/drug/infopage/antiepileptics/default.htm. accessed July 14, 2009

Question 63: Pharmacology/Chemistry - Epilepsy

Discussion:

The symptoms are common in patients with hyperammonemia. Disorders of urea cycle are common in mentally retarded or
developmentally abnormal persons with epilepsy. VPA increases levels of gamma-aminobutyric acid and prolongs the recovery
of inactivated sodium channels. These properties may be responsible for its action as a CNS depressant. VPA may also cause
impairments in fatty-acid metabolism and disrupt the urea cycle, leading to hyperammonemia. VPA also alters fatty-acid
metabolism, impairs beta-oxidation (a mitochondrial process), and disrupts the urea cycle. Hyperammonemia and other metabolic
effects, as well as end-organ effects (hepatitis, pancreatitis, hemodynamic compromise), may be the result of severe toxicity due
to these impaired metabolic processes.

There are no known drug-drug interactions between valproate and levetiracetam. No reports of levetiracetam-induced hepatitis
are in the literature.

References:

Seger DL. Anticonvulsant medications. In: Dart R, ed. Medical Toxicology.. 3rd ed . Philadelphia, PA: Lippincott Williams &
Wilkins; 2004: 798-801.

Question 64: Pharmacology/Chemistry - Sleep Disorders

Discussion:

Ramelteon is the first in a new class of sleep agents that selectively binds to the MT1 and MT2 receptors in the suprachiasmatic
nucleus (SCN), versus binding to GABA A receptors, such as with drugs like zolpidem, eszopiclone, and zaleplon. Ramelteon is
approved by the FDA for long-term use. Ramelteon does not show any appreciable binding to GABA-A receptors, which are
associated with anxiolytic, myorelaxant, and amnesic effects.

References:

Pandi-Perumal SR, Srinivasan V, Poeggeler B, et al. Drug insight: the use of melatonergic agonists for the treatment of insomnia-
focus on ramelteon. Nat Clin Pract Neurol 2007;3:221-228.

Question 67: Pharmacology/Chemistry - Headache

Discussion:

This patient has episodic cluster headache and is in a cluster cycle. A brief course of corticosteroids can achieve rapid, short-term
headache remission while titrating prophylactic treatment. Sumatriptan (parenteral or intranasal) and oxygen inhalation are
effective for abortive therapy for acute cluster headache. Verapamil and lithium carbonate are used for long-term prophylaxis for
patients whose cluster headache cycles last more than 2 weeks.
References:

Rozen TD. Trigeminal autonomic cephalgias.. Neurol Clin 2009; 27:537--556.

Question 69: Pharmacology/Chemistry - Headache

Discussion:

Of the medications listed only topiramate has proven efficacy as a migraine preventive therapy. The others are either unproven or
proven to not be effective.

References:

Silberstein SD. Topiramate in migraine prevention: evidence-based medicine from clinical trials. Neurol Sci 2004;25 (suppl
3):244-245.

Storey JR, Calder CS, Hart DE, Potter DL. Topiramate in migraine prevention: a double-blind, placebo-controlled study.
Headache 2001;41:968-975.

Question 71: Pharmacology/Chemistry - Other Pain Syndromes

Discussion:

Duloxetine acts as an analgesic (and antidepressant) via its selective inhibition of norepinephrine- and serotonin-reuptake.

References:

Luszczki JJ. Third-generation antiepileptic drugs: mechanisms of action, pharmacokinetics and interactions. Pharmacol Rep
2009;61;197-216.

Question 76: Pharmacology/Chemistry - Movement Disorders

Discussion:

Subcutaneous injections of apomorphine activate dopamine receptors within about 15 minutes and can be very effective to
"rescue" patients with Parkinson disease that have severe off episodes. Oral pramipexole and ropinirole are absorbed relatively
slowly and usually do not have dramatic effects with single doses. Controlled release carbidopa/levodopa is also observed very
slowly. Subcutaneous metoclopramide, while acting quickly, blocks dopamine receptors and will make his symptoms worse.

References:

Kolls BJ, Stacy M. Apomorphine: a rapid rescue agent for the management of motor fluctuations in advanced Parkinson disease.
Clin Neuropharmacol 2006;29:292-301.

Question 83: Pharmacology/Chemistry - Epilepsy

Discussion:

Drugs that can raise carbamazepine levels include isoniazid, erythromycin, cimetidine, calcium channel blockers (such as
verapamil), and propoxyphene. Carbamazepine levels are lowered by phenobarbital, phenytoin, and primidone. Warfarin,
chlorpromazine, digoxin, and gabapentin have no significant effect on carbamazepine levels.

References:
Holland KD. Efficacy, pharmacology, and adverse effects of antiepileptic drugs. Neurol Clin 2001;19:313-345.

Question 108: Pharmacology/Chemistry - Demyelinating Disorders

Discussion:

This patient has clinically isolated right optic neuritis, with a normal brain MRI. Based upon the optic neuritis treatment trial, a
short course of intravenous steroids followed by an oral steroid taper is appropriate to hasten clinical recovery; oral prednisone
increased the risk of developing optic neuritis in the fellow eye. Patients who present with a clinically isolated demyelinating
syndrome and a normal brain MRI are at relatively low risk to develop clinically definite multiple sclerosis and generally should
not be offered MS disease-modifying therapy initially.

References:

Goodin DS, Frohman EM, Garmany GP Jr, et al. Disease modifying therapies in multiple sclerosis: report of the Therapeutics
and Technology Assessment Subcommittee of the AAN and the MS Council for Clinical Practice Guidelines. Neurology
2002;58:169-178.

Jacobson D. Optic neuritis. In: Samuels MA, Feske SK, editors. Office practice of neurology. Philadelphia: Churchill-
Livingstone, 2003;416-420.

Question 114: Pharmacology/Chemistry - Neuromuscular Disorders

Discussion:

Malignant hyperthermia (MH) is a pharmacogenetic disorder of skeletal muscle that presents as a hypermetabolic response to
potent volatile anesthetic gases such as halothane, sevoflurane, desflurane and the depolarizing muscle relaxant succinylcholine,
and rarely, in humans, to stresses such as vigorous exercise and heat. The incidence of MH reactions ranges from 1:5,000 to
1:50,000-100,000 anesthesias. However, the prevalence of the genetic abnormalities may be as great as one in 3,000 individuals.
Rhabdomyolysis refers to the breakdown of skeletal muscle which is associated with excretion of myoglobin in the urine.
Classically, MH presents with hypercarbia, tachycardia, cardiac arrhythmias, pyrexia, rigidity and metabolic acidosis, and
rhabdomyolysis as a late sign. The essential points in the treatment of acute MH crisis are the immediate discontinuation of
trigger agents, hyperventilation, administration of dantrolene in doses of 2.5 mg/kg repeated prn (pro re nata) to limit MH,
cooling by all routes available (especially nasogastric lavage), and treating hyperkalemia in a standard fashion. Bromocriptine is
indicated for neuroleptic malignant hyperthermia. Baclofen and clonazepam are indicated for spasticity management. Sodium
bicarbonate may be indicated as adjunct for treatment of metabolic acidosis.

References:

Rosenberg H, Davis M, James D, et al. Malignant hyperthermia. Orphanet J Rare Dis 2007;2:21.

Question 128: Pharmacology/Chemistry - Epilepsy

Discussion:

Of the medications listed, only valproic acid is associated with neural tube defects. It is a known human teratogen. Exposure in
pregnancy is associated with approximately three-fold increase in the rate of major anomalies, mainly spina bifida and only rarely
anencephaly (NTD), cardiac, craniofacial, skeletal and limb defects and a possible set of dysmorphic features, the "valproate
syndrome" with decreased intrauterine growth.

References:

Ornoy A. Valproic acid in pregnancy: how much are we endangering the embryo and fetus. Reprod Toxicol 2009;28:1-10.

Wyllie E. The treatment of epilepsy, principles and practices. 4th ed. Philadelphia: Lippincott, Williams & Wilkins, 2006.
Question 142: Pharmacology/Chemistry - Neuromuscular Disorders

Discussion:

Central and peripheral nervous system toxicity are frequent complications of most chemotherapy regimens, often leading to
reduction of dosages or cessation of the responsible drugs. The most common chemotherapeutic agents that might cause CNS
toxicity manifested as encephalopathy of various severities include methotrexate, vincristine, ifosfamide, cyclosporine,
Xudarabine, cytarabine, 5- Xuorouracil, cisplatin and the interferons (alpha > beta). Involvement of the peripheral nervous
system manifested as distal peripheral neuropathy results after therapy with cisplatin, vincristine, taxanes, suramin and
thalidomide. Symptoms and signs of peripheral neuropathy may occur after administration of numerous anticancer drugs. The
main culprits, however, are vinca alkaloids, which produce an axonal dying back neuropathy, platinum derivatives (cisplatin and
oxaliplatin), which cause sensory neuronopathy, and taxanes (paclitaxel, docetaxel), which cause a sensory-motor neuropathy.

For all these drugs, the severity of chronic peripheral neuropathy is related to total dose and to shorter interval of drug
administration. Patients with preexisting neuropathy are at particular risk. Symptoms and signs of vinca alkaloid and taxane
neuropathy predominate in the lower limbs, and this has been attributed to nerve length. Taxanes are antineoplastic agents that
promote the assembly of microtubules as well as stabilizing their formation by preventing depolymerization. Taxanes produce a
symmetric, axonal mixed, predominantly sensory, distal neuropathy. There is currently no effective symptomatic treatment for
paclitaxel-associated pain secondary to neuropathy, but tricyclic antidepressants and anticonvulsants have been used as
symptomatic treatment with some success.

References:

Sioka C, Kyritsis AP. Central and peripheral nervous system toxicity of common chemotherapeutic agents. Cancer Chemother
Pharmacol 2009;63:761-767. Epub 2008 Nov 25.

Hildebrand J. Neurological complications of cancer chemotherapy. Curr Opin Oncol 2006;18:321 324.

Question 143: Pharmacology/Chemistry - Other Pain Syndromes

Discussion:

Peripheral neuropathy is a well recognized toxicity of taxanes, usually resulting to dose modification and changes in the
treatment plan. Taxanes produce a symmetric, axonal predominantly sensory distal neuropathy with less prominent motor
involvement. Studies in preclinical models demonstrated that the administration of paclitaxel resulted in accumulation of
microtubules in Schwann cells and axons of sciatic nerve. A resulting "dying back" process occurs, starting from distal nerve
endings followed by effects on Schwann cells, neuronal body or axonal transport changes and a disturbed cytoplasmatic flow in
the affected neurons is the most widely accepted mechanism of taxanes neurotoxicity. The incidence of taxanes-induced
peripheral neuropathy is related to causal factors, such as single dose per course and cumulative dose and risk factors including
treatment schedule, prior or concomitant administration of platinum compounds or vinca alkaloids, age and pre-existing
peripheral neuropathy of other causes.

References:

Argyriou AA, Koltzenburg M, Polychronopoulos P, et al. Peripheral nerve damage associated with administration of taxanes in
patients with cancer. Crit Rev Oncol Hematol 2008;66:218-228. Epub 2008 Mar 7.

Question 149: Pharmacology/Chemistry - Epilepsy

Discussion:

Topamax has been known to cause a potentially serious condition known as metabolic acidosis (an increase of acid in the blood).
In children, chronic metabolic acidosis may affect growth or cause rickets (a softening or weakness of the bones that can lead to
bone deformities). A doctor should be contacted immediately if one experiences symptoms of metabolic acidosis such as rapid
breathing, an irregular heartbeat, confusion, lethargy, fatigue, or loss of appetite.

References:
Available from. :www.topamax.com/topamax/tools-resources--safety-information.html. accessed March 1, 2010.

Question 150: Pharmacology/Chemistry - Other Pain Syndromes

Discussion:

Type 1 (Andrade) familial amyloidosis, inherited as an autosomal dominant, is associated with amyloid deposition in peripheral
nerves. It is characterized by progressive loss of pain and temperature sensation, lancinating pain, and severe generalized
autonomic failure. Mutations have been demonstrated in the gene that codes for transthyretin, the protein that transports
thyroxine and retinol binding protein.

References:

Bosch EP, Smith BE. Disorders of peripheral nerves. In: Bradley WG, Daroff RB, Fenichel GM, et al, editors. Neurology in
clinical practice. 3rd ed. Boston: Butterworth-Heinemann, 2000.

Question 156: Pharmacology/Chemistry - Movement Disorders

Discussion:

Tetrabenazine "poisons" catecholaminergic vesicles by binding with high affinity to vesicular monamine transporter 2 (VMAT2)
resulting in depletion of neuronal stores of dopamine and other catecholamines. It is FDA approved for the treatment of chorea
related to Huntington disease.

References:

Zheng G, Dwoskin LP, Crooks PA. Vesicular monoamine transporter 2: role as a novel target for drug development. AAPS J
2006;8:E682-692.

Question 171: Pharmacology/Chemistry - Epilepsy

Discussion:

The pre-ovulatory surge of estrogen that is unaccompanied by a rise in progesterone contributes to the proconvulsant effects.
Estrogen reduces the effectiveness of GABA-mediated neuronal inhibition and activates NMDA receptor complex to increase
seizure tendencies. Progesterone is protective through potentiation of GABA activity and reducing glutamate activity.

References:

Wyllie E. The treatment of epilepsy, principles and practices. 4th ed. Philadelphia: Lippincott, Williams & Wilkins, 2006.

Question 172: Pharmacology/Chemistry - Neuromuscular Disorders

Discussion:

Hypokalemic periodic paralysis is the most common PP and is clinically characterized by episodic attacks of muscle weakness,
which are associated with a drop in serum potassium levels. Onset of the attacks is usually in childhood (1 to 20 years). Ocular,
bulbar and respiratory muscles are usually spared. Weakness lasts for hours to days. Attacks may be triggered or enhanced by
hypokalemia, physical activity, night rest, a carbohydrate-rich diet, alcohol or cold. Corticosteroids and beta agonists should be
given with caution and glucose infusions should be avoided.

Hypokalemic periodic paralysis is genetically heterogeneous and may be due to mutations in the a-subunit of the DHP-receptor
(CACNA1S) gene on chromosome 1q31 (HypoPP1, 55% to 70%) sodium channel (SCN4A) gene on chromosome 17q23
(HypoPP2, 8% to 10%). Transmission of HypoPP1 follows an autosomal dominant trait. The pathophysiological basis for
HypoPP1 is still not understood. CACNA1S mutations cause a loss of function, manifesting as reduced current density and
slower inactivation. CACNA1S mutations have some similarities to SCN4A mutations as they modify channel inactivation but
not voltage-dependent activation. How inactivation is related to hypokalemia-induced attacks is not understood. A failure of
excitation is more likely than a failure of excitation contraction coupling. Hypokalemia induced depolarization may reduce
calcium release, affecting the voltage control of the channel directly or indirectly by inactivation of the sodium channel.

Acute treatment involves oral supplementation of potassium is preferable to intravenous supplementation. Carboanhydrase
inhibitors, in particular, dichlorphenamide or acetazolamide have been repeatedly shown to be effective. To prevent the
occurrence of attacks, a diet low in sodium and carbohydrates, but rich in potassium is recommended.

References:

Finsterer J.. Primary periodic paralyses. Acta Neurol Scand 2008: 117: 145 158.

Question 175: Pharmacology/Chemistry - Epilepsy

Discussion:

Carbamazepine, lamotrigine, phenytoin, topiramate, valproic acid, zonisamide all have in common slow recovery of voltage-
gated sodium channels.

References:

Gibbs JW and McNamara JO. Chapter 37. The epilepsies: phenotype and mechanisms. Chapter 37. The epilepsies: phenotype
and mechanisms. Page 634 In, Basic Neurochemistry: molecular, cellular and medical aspects., seventh edition; academic press;
Amsterdam; Siegel G. J., editor-in-chief; 2006

Question 177: Pharmacology/Chemistry - Neurogenetics

Discussion:

Ataxia telangiectasia (AT) is characterized by progressive cerebellar ataxia, oculocutaneous telangiectasia, progressive cerebellar
dysfunction, and recurrent sinopulmonary infections secondary to progressive immunological and neurologic dysfunction. AT
patients are significantly predisposed to cancer, particularly lymphomas and leukemia. Other manifestations of the disease
include sensitivity to ionizing radiation, premature aging, and hypogonadism. AT has been a major interest of scientists since the
1960's because it may yield an insight into numerous other major health problems, such as cancer, neurologic disease,
immunodeficiency, and aging.

AT is characterized by: early-onset progressive cerebellar ataxia, oculocutaneous telangiectasia (dilated blood vessels in the eyes
and skin), immunodeficiency mostly thorough lowering of IgA, IgG and IgE levels, chromosomal instability, hypersensitivity to
ionizing radiation, increased incidence of malignancies primarily lymphoid, and raised alpha-fetoprotein levels.

References:

Verhagen MM, Abdo WF, Willemsen MA, et al. Clinical spectrum of ataxia-telangiectasia in adulthood. Neurology
2009;73:430-437.

Stray-Pedersen A, Borresen-Dale AL, Paus E, et al. Alpha fetoprotein is increasing with age in ataxia-telangiectasia. Eur J
Paediatr Neurol 2007;11:375-380.

Question 178: Pharmacology/Chemistry - Aging, Degenerative Diseases

Discussion:

The discovery of elevated CSF glutamate levels in patients with ALS led to the study of glutamate release inhibitors, such as
riluzole, in the treatment of ALS. Studies have shown that riluzole has an 8% benefit on survival at 12 months and a 9% benefit
on survival at 18 months. Both baclofen and dantrolene are effective in the symptomatic treatment of spasticity. However,
baclofen has fewer adverse effects and is, therefore, a more appropriate initial therapeutic choice. Diazepam or tizanidine could
also be used.

References:

Brooks BR. Emerging directions in ALS therapeutics: palliative therapies at the advent of the twenty-first century. Clin Neurosci
1995-1996;3:386-392.

Mitsumoto H, Chad DA, Pioro EP. Amyotrophic lateral sclerosis. Philadelphia: FA Davis, 1998.

Question 182: Pharmacology/Chemistry - Epilepsy

Discussion:

Hypohidrosis is an infrequent side effect of zonisamide, particularly in children. Hypohidrosis can lead to symptoms of heat
stroke due to impaired heat-dissipation mechanisms and produce hyperthermia.

References:

Low P, James S, Peschel T, et al. Zonisamide and associated oligohidrosis and hyperthermia. Epilepsy Res 2004;62:27-34.

Question 197: Pharmacology/Chemistry - Other Pain Syndromes

Discussion:

The lidocaine patch is not approved for use in conditions with wide-spread or non-focal pain syndromes. It is also not approved
for episodic pain syndromes. It has been approved for use in postherpetic neuralgia to treat focal, persistent, neuropathic pain.

References:

none. PDR.NET. www.pdr.net (accessed July 14, 2007)

Question 198: Pharmacology/Chemistry - Toxic/Metabolic Disease

Discussion:

The serotonin syndrome (SS) is a potentially life-threatening drug interaction that results from excess central serotonergic tone. It
is chiefly a clinical diagnosis based in part on recognition of a clinical triad of mental status changes, hyperautonomaticity and
neuromuscular changes. Much attention has been devoted lately to the abuse potential of dextromethorphan, a popular
antitussive, whose primary metabolite possesses a pharmacodynamic profile similar to phencyclidine. Dextromethorphan also
enhances central serotonergic tone by blocking its reuptake and increasing its release. Cases of SS resulting from combining
dextromethorphan with serotonin reuptake inhibitors and monoamine oxidase inhibitor medications have been reported

References:

Schwartz AR, Pizon AF, Brooks DE.. Dextromethorphan-induced serotonin syndrome.. Clin Toxicol (Phila). 2008
Sep;46(8):771-3.

Question 358: Pharmacology/Chemistry - Epilepsy

Discussion:

Oxcarbazepine is a derivative of carbamazepine and shares many similarities with carbamazepine, including its mechanism of
action, ability to induce hepatic metabolism of oral contraceptives and other drugs, risk of hyponatremia, and indication for
treatment of partial epilepsy. Unlike carbamazepine, which is metabolized to an active metabolite (10-11 epoxide), oxcarbazepine
is not metabolized. This may explain its fewer side effects as compared to carbamazepine.

References:

Schmidt D, Elger CE. What is the evidence that oxcarbazepine and carbamazepine are distinctly different antiepileptic drugs?
Epilepsy Behav 2004;5(5):627-635.

Question 367: Pharmacology/Chemistry - Neurogenetics

Discussion:

Enzyme replacement therapy with alpha galactosidase A has now been available for several years and received approval in
Europe in 2001 and the USA in 2003 for the treatment of Fabry disease.

The advent of specific treatment has increased the need for early recognition to help treatment and delay or prevent
complications. Studies have reported decreased cardiac mass, decreased frequency of pain crisis, and clearance of storage in skin
and kidneys. Some evidence exists of increased sweating, improved hearing and sensation, improved pulmonary symptoms, and
decreased gastrointestinal symptoms, but the evidence is not strong enough to clearly show long-term benefit for these problems.

Studies have generally shown the greatest benefit when treatment is started at an early stage of the disease before extensive
fibrosis or other irreversible tissue damage takes place. Benefit is more likely to be reported when enzyme replacement therapy is
given before severe kidney dysfunction at baseline and with mild degrees of LVH. In Europe, two preparations of the enzyme are
available. In the USA, only one preparation has been approved by the US Food and Drug Administration. However, no
convincing evidence exists to show that the two enzymes are functionally different.

References:

Zarate YA, Hopkin RJ.. Fabry's disease.. Lancet. 2008 Oct 18;372(9647):1427-35.

Question 375: Pharmacology/Chemistry - Headache

Discussion:

Idiopathic intracranial hypertension (IIH), also called pseudotumor cerebri, is a condition in which intracranial pressure (ICP) is
elevated without apparent cause. There is a predilection for obese women of childbearing age. Although the annual incidence of
IIH is low (0.9/100 000), the incidence in women between 20 and 44 years of age and above 20% over ideal body weight is
nearly 20 times higher than the general population. Vitamin A toxicity has long been acknowledged as a cause of severe
headaches and papilledema associated with intracranial hypertension. The finding of increased levels of unbound retinol in the
CSF of IIH patients suggests the involvement of vitamin A in its pathogenesis. There has been increased attention paid to sleep
disorders in IIH. Sleep apnea is a well documented association, particularly in men, suggesting that in such patients, alveolar
hypoventilation and nocturnal hypoxia may play a role in raised ICP.

References:

Randhawa S, Van Stavern GP.. Idiopathic intracranial hypertension (pseudotumor cerebri).. Curr Opin Ophthalmol. 2008
Nov;19(6):445-53.

Question 378: Pharmacology/Chemistry - Movement Disorders

Discussion:

Tardive dyskinesia (TD) has been used to refer to the tardive syndrome that presents with rapid, repetitive, stereotypic
movements involving the oral, buccal, and lingual areas. A combination of tongue twisting and protrusion, lip smacking and
puckering, and chewing movements in a repetitive and stereotypic fashion is often observed. TD results from chronic exposure to
dopamine receptor blocking agents drugs primarily used to treat psychosis. TD has not been reported with agents that deplete
dopamine (such as reserpine) and is only rarely reported with   atypical  antipsychotic drugs (such as clozapine). Some
drugs for nausea (such as metoclopramide or prochlorperazine) and depression (such as amoxapine) are actually dopamine
receptor blocking agents and therefore can cause TD. The annual incidence rates range from 5% in the younger population (mean
age, 28 years)to 8% to 12% in the older group (mean age, 56 years). In general, at least 20% of patients treated with neuroleptics
are affected with TD, and roughly 5% are expected to develop TD with each year of neuroleptic treatment. The weight of
evidence favors the notion that chronic blockade of dopamine receptors leads to increased receptor sensitivity. After 2 weeks of
therapy with dopamine receptor blocking agents, the affinity and number of dopamine D2 receptors increased in rodent models of
TD. Yet, it remains unclear how this leads to the development of TD, or why only a minority of patients with similar drug
exposure develop TD.

References:

Soares-Weiser K, Fernandez HH. Tardive dyskinesia. Semin Neurol 2007;27:159-169.

Question 382: Pharmacology/Chemistry - Demyelinating Disorders

Discussion:

Glatiramer acetate therapy is not associated with the development clinically significant neutralizing antibodies. All formulations
of beta-interferon and natalizumab can result in the formation of neutralizing antibodies, which abrogate their clinical and MRI
efficacy against disease activity.

References:

Sorenson PS, Ross C, Clemmesen KM, et. al. Danish Multiple Sclerosis Study Group.. Clinical importance of neutralising
antibodies against interferon beta in patients with relapsing-remitting multiple sclerosis.. Lancet 2003;362(9391):1184-1191.

Goodin DS, Frohman EM, Garmany GP Jr, et al. Disease modifying therapies in multiple sclerosis: report of the Therapeutics
and Technology Assessment Subcommittee of the AAN and the MS Council for Clinical Practice Guidelines. Neurology
2002;58:169-178.

Question 384: Pharmacology/Chemistry - Movement Disorders

Discussion:

A three-base-pair deletion in the torsin A gene leads to generalized torsion dystonia (DYT1) in humans.

References:

Granata A, Schiavo G, Warner TT.. TorsinA and dystonia: from nuclear envelope to synapse.. J Neurochem. 2009
Jun;109(6):1596-609

Bressman SB.. Genetics of dystonia: an overview.. Parkinsonism Relat Disord. 2007;13 Suppl 3:S347-55

Question 391: Pharmacology/Chemistry - Dementia

Discussion:

Loss or dysfunction of the acetylcholine projection to the cerebral cortex has been shown experimentally in humans and animals
to result in cognitive disturbance. Loss of neurons from the nucleus Basalis of Meynert was shown early on to characterize
patients with Alzheimer disease. Recent studies however have shown that the depletion of cortical acetylcholine has been
significantly more severe in patients with dementia with Lewy bodies than even in Alzheimer disease. Frontotemporal dementia
nor Huntington disease have significant loss of cortical cholinergic projections, at least in early to moderate stages. Vascular
dementia shows variable loss, depending on where the vascular lesions are; in addition, many of these patients have mixed
Alzheimer/vascular pictures.
References:

Samuel W, Alford M, Hofstetter CR, Hansen L. Dementia with Lewy bodies versus pure Alzheimer disease: differences in
cognition, neuropathology, cholinergic dysfunction, and synapse density. J Neuropathol Exp Neurol 1997;56(5):499-508.

Question 399: Pharmacology/Chemistry - Headache

Discussion:

For migraine prevention, individual medications have been put into treatment groups based on their established clinical efficacy,
significant adverse events, safety profile, and clinical experience of the US Headache Consortium participants: Group 1
medications (amitriptyline, divalproex sodium, propranolol/timolol) have proven high efficacy and mild to moderate adverse
events. Group 2 medications (verapamil, nonsteroidal anti-inflammatory drugs, gabapentin, vitamin B2) with lower efficacy (ie,
limited number of studies, studies reporting conflicting results, efficacy suggesting only modest improvement) and mild to
moderate adverse events. Group 3 medication (topiramate, diltiazem, antidepressants) are used based on opinion, not randomized
controlled trials, and may have low to moderate or frequent or severe adverse events. Group 4 medication (methysergide) has
shown proven efficacy but frequent or severe adverse events (or safety concerns) or complex management issues. Group 5
(carbamazepine, clonidine, clonazepam) medications are proven to have limited or no efficacy.

References:

Silberstein S. Practice parameter: Evidence-based guidelines for migraine headache (an evidence-based review). Report of the
Quality Standards Subcommittee of the American Academy of Neurology. Neurology 2000;55:754-763.

Question 418: Pharmacology/Chemistry - Dementia

Discussion:

Frontotemporal dementia (FTD) is the most common syndrome in which the focus of neurodegeneration is the frontal lobes. FTD
is frequently familial. It is also often due to a susceptibility locus on chromosome 17q21-22. Some 17q21-22-linked families have
mutations in the tau gene and most have microscopically visible aggregates of hyperphosphorylated tau. Demonstrating that
mutations in tau can produce neurodegeneration will necessitate a reassessment of the role of tau in the pathogenesis of the many
diseases in which tau biology is disrupted.

References:

Wilhelmsen KC. Frontotemporal dementia genetics. J Geriatr Psychiatry Neurol 1998;11:55-60.

Neumann M, Tolnay M, Mackenzie IR. The molecular basis of frontotemporal dementia. Expert Rev Mol Med 2009;11:e23.

Question 424: Pharmacology/Chemistry - Cerebrovascular Disease

Discussion:

Animal studies suggest that brain ischemia causes an excessive increase of extracellular concentration of glutamate, an
excitotoxic amino acid, in the central nervous system. In neuronal cells, glutamate receptor activation by excitotoxic glutamate
causes the entry of extracellular Ca2+ resulting in intracellular acidification through unknown mechanisms This intracellular
accumulation of the electrolyte calcium thus leads to apoptotic and necrotic cell death by initiating a cascade of events at both the
cytoplasmic and nuclear levels.

References:

Jovin TG, Demchuk M, Gupta R.. Pathophysiology of acute ischemic stroke.. Continuum lifelong learning neurology 14 (2008)
28-45
Question 431: Pharmacology/Chemistry - Movement Disorders

Discussion:

Dopamine agonist treatment in Parkinson disease has been associated with several impulse control disorders, including
pathologic gambling, hypersexuality, compulsive eating, and compulsive shopping.

References:

Olanow CW, Stern MB, Sethi K. The scientific and clinical basis for the treatment of Parkinson disease. Neurology 2009;72
(Suppl 4):S1-S136.

Gschwandtner U, Aston J, Renaud S, Fuhr P. Pathologic gambling in patients with Parkinson's disease. Clin Neuropharmacol
2001;24:170-172.

Question 434: Pharmacology/Chemistry - Neuromuscular Disorders

Discussion:

There are many inherited and acquired neuromuscular disorders that may interfere with vital functions and with the response to
surgery and anaesthesia. Several conditions including upper and lower motor neuron lesions, like in ALS, cause denervation of
the striated muscle followed by upregulation of nicotinic acetylcholine receptors (AChRs). Also, muscle denervation (by muscle
relaxants, drugs, or toxins), immobilization, muscle inflammation and trauma, or both, as seen in burn injury or radiation injury,
cause upregulation of AChRs. This involves an increase in number of AChRs spreading throughout the muscle membrane.

There is now evidence that an isoform of AChR, neuronal (nicotinic) [alpha]7AChR, is expressed and upregulated in muscle with
these conditions. These [alpha]7AChRs are more sensitive to succinylcholine and its metabolite, choline, than normal AChRs.
This accounts for the persistence of hyperkalemia and the risk of cardiac arrest. The hyperkalemic response to succinylcholine is
proportional to the upregulation. The presence of two or more etiologic factors will magnify the upregulation.

Anesthetic associated adverse events are also associated with halothane anesthetics in muscle disorders such as dystrophies,
congenital myopathies, and mitochondrial disorders with associated rhabdomyolysis and malignant hyperthermia, neither
condition having been reported in ALS patients in association with halothane anesthetics.

References:

Driessen JJ. Neuromuscular and mitochondrial disorders: what is relevant to the anaesthesiologist. Curr Opin Anaesthesiol
2008;21:350-355.

Question 440: Pharmacology/Chemistry - Aging, Degenerative Diseases

Discussion:

Midodrine is a prodrug that is converted in the liver to an alpha agonist. Its predictable absorption and pharmacokinetics makes it
the drug of choice for treatment of orthostatic hypotension unresponsive to fludrocortisone.

References:

Riley DE. Orthostatic hypotension in multiple system atrophy. Curr Treat Options Neurol 2000;2(3):225-230.

Physiology

Question 4: Physiology - EEG

Discussion:
Absence seizures are characteristically seen in children 5 to 15 years of age, are associated with staring, and may have
automatisms such as repetitive chewing movements. Similar features may be seen in complex partial seizures, although absence
seizures are shorter and may occur more frequently. The characteristic electroencephalogram during an absence seizure would
show 3 Hz spike-and-wave discharges.

References:

Niedermeyer E, Lopes da Silva F. Electroencephalography: basic principles, clinical applications, and related fields. 5th ed.
Philadelphia: Lippincott, Williams and Wilkins, 2004.

Question 5: Physiology - EMG

Discussion:

Myotonic discharges are common in periodic paralysis. Nascent units are seen in nerve regeneration and fasciculations are seen
in any chronic neurogenic disorder. Myokymia can be seen in various neurologic conditions, but is very common in radiation-
induced plexopathy. Complex repetitive discharges (CRDs) can be seen in a variety of disorders as well.

References:

Katirji B, Kaminski HJ, Preston DC, et al. Neuromuscular disorders in clinical practice. Boston: Butterworth-Heinemann, 2002.

Question 20: Physiology - EMG

Discussion:

The extensor indicis proprius (EIP) is supplied by the lower trunk/posterior cord. The adductor pollicis, abductor pollicis brevis,
flexor pollicis brevis and FDP 4/5 are supplied by the lower trunk/medial cord.

References:

Preston DC, Shapiro BE. Electromyography and neuromuscular disorders: clinical-electrophysiologic correlations. 2nd ed.
Philadelphia: Elsevier, 2005.

Question 24: Physiology - EMG

Discussion:

The decremental response to 2 to 3 Hz repetitive nerve stimulation in patients with myasthenia gravis is due to failure of
neuromuscular transmission at a number of endplates resulting in fewer muscle fiber action potentials contributing to the
compound muscle action potential.

References:

Brown WF, Bolton CF, editors. Clinical electromyography. 2nd ed. Boston: Butterworth-Heinemann, 1993.

Question 43: Physiology - EMG

Discussion:

In Martin-Gruber anastomosis, the ulnar nerve fibers can piggy back along with the median nerve proximally and then go back to
the main nerve distally. This can cause an artificial increase in the amplitude proximally.

References:
Preston DC, Shapiro BE. Electromyography and neuromuscular disorders: clinical-electrophysiologic correlations. 2nd ed.
Philadelphia: Elsevier, 2005.

Question 44: Physiology - EEG

Discussion:

Alpha rhythm has a frequency of 9 to 13 Hz which slows to 7 to 8 Hz in drowsiness. It is the same dominant frequency in each
hemisphere with the highest amplitude in O1, O2, P3, P4, T5, T6. Symmetry of alpha rhythm is best measured on ear referential
montages. It is partially or completely blocked by eye opening or alerting.

References:

Daube JR, editor. Clinical neurophysiology. New York: Oxford University Press, 2002

Blume WT, Kaibara M. Atlas of adult electroencephalography. New York: Raven Press, 1995. page 19, 23, 25, 33, 18

Question 46: Physiology - Evoked Potentials

Discussion:

A unilateral P100 abnormality indicates an ipsilateral lesion of the visual pathway anterior to the optic chiasm such as a unilateral
demyelinating process or optic nerve glioma. A tumor of the occipital lobe or a thalamic hemorrhage would cause a bilateral
abnormality or little or no effect on the P100 latency.

References:

Chiappa KH. Evoked potentials in clinical medicine. 3rd ed. New York: Lippincott-Raven, 1997.

Question 48: Physiology - EMG

Discussion:

The amplitude of the facial nerve compound muscle action potential (CMAP) is correlated with the number of functional axons
in the facial nerve. The lower the CMAP amplitude, the more axonal degeneration and the poorer the prognosis.

References:

Preston DC, Shapiro BE. Electromyography and neuromuscular disorders: clinical-electrophysiologic correlations. 2nd ed.
Philadelphia: Elsevier, 2005.

Question 52: Physiology - EMG

Discussion:

The H reflex is performed by stimulating the tibial nerve in the popliteal fossa and recording over the soleus muscle.

References:

Preston DC, Shapiro BE. Electromyography and neuromuscular disorders: clinical-electrophysiologic correlations. 2nd ed.
Philadelphia: Elsevier, 2005.

Question 53: Physiology - EEG


Discussion:

Breach rhythm is a finding seen in patients with skull defects, which alters the conductance between the brain and the recording
electrode over the scalp.

References:

Daube JR, editor. Clinical neurophysiology. New York: Oxford University Press, 2002

Klass DW, Westmoreland BF. Electroencephalography: general principles and adult electroencephalograms. In: Daube JR,
editor. Clinical neurophysiology. Philadelphia: FA Davis, 1996.

Question 57: Physiology - Autonomic Studies

Discussion:

Normal findings on the quantitative sudomotor axon reflex test (QSART) indicate integrity of postganglionic sympathetic
sudomotor axon. Absent response indicates failure of this axon providing that iontophoresis is successful and that eccrine sweat
glands are present. Because the axonal segment mediating that axon is likely to be short, the test probably evaluates distal axonal
function.

References:

Daube JR, editor. Clinical neurophysiology. New York: Oxford University Press, 2002

Daube JR, editor. Clinical neurophysiology. Philadelphia: FA Davis, 1996; page 267, 269

Question 60: Physiology - Basic Physiology

Discussion:

The Achilles tendon reflex is a monosynaptic reflex with the efferent limb in the tibial nerve.

References:

Kandel ER, Schwartz JH, Jessel TM. Principles of neural science. 4th ed. New York: McGraw-Hill, 2000.

Question 68: Physiology - EEG

Discussion:

Medial temporal lobe epilepsy is the most common partial seizure disorder. Secondary generalization can occur. EEG in such
patients usually shows focal temporal spikes.

References:

Wyllie E. The treatment of epilepsy, principles and practices. 4th ed. Philadelphia: Lippincott, Williams & Wilkins, 2006.

Question 77: Physiology - Sleep

Discussion:

Polysomnograms are recorded during the normal sleeping hours of a patient. Patients sleep 6 to 8 hours before either awakening
spontaneously or being awakened by a technician. The goal of polysomnography is to quantitate the amount of time spent in
various stages of sleep during the night and to document clinically relevant events such as cardiopulmonary abnormalities or
sleep related abnormal motor activity. Polysomnograms are typically recorded at a paper speed of 10 mm/sec on 12 to 22 channel
polygraphs.

References:

Daube JR, editor. Clinical neurophysiology. New York: Oxford University Press, 2002

Daube JR, editor. Clinical neurophysiology. Philadelphia: FA Davis, 1996; page 267, 269

Question 81: Physiology - Basic Physiology

Discussion:

Significant membrane depolarization is caused by elevated extracellular potassium ions.

References:

Kandel ER, Schwartz JH, Jessel TM. Principles of neural science. 4th ed. New York: McGraw-Hill, 2000.

Question 82: Physiology - EEG

Discussion:

The entity of benign seizures of childhood (Sylvian or Rolandic) is characterized by focal tingling or jerking of the side of the
face or hand, salivation, inability to speak and progression to a generalized tonic-clonic seizure. The EEG shows spike discharges
over the central midtemporal region which are characteristic of this entity.

References:

Daube JR, editor. Clinical neurophysiology. New York: Oxford University Press, 2002

Question 110: Physiology - EEG

Discussion:

Frontal intermittent rhythmic delta activity (FIRDA) can be seen with a variety of lesions including posterior fossa lesions,
encephalopathy, intracranial lesions, and increased intraventricular pressure. REM sleep is associated with a low-voltage
desynchronized EEG.

References:

Niedermeyer E, Lopes da Silva F. Electroencephalography. 3rd ed. Baltimore: Williams & Wilkins, 1993.

Question 123: Physiology - EEG

Discussion:

The burst suppression pattern has the worst prognosis. Most of the patients with this type of pattern following an anoxic insult
either die or do not regain a meaningful degree of neurologic function.

References:
Wijdicks EF, Parisi JE, Sharbrough FW. Prognostic value of myoclonus status in comatose survivors of cardiac arrest. Ann
Neurol 1994;35:239-243.

Question 126: Physiology - Evoked Potentials

Discussion:

Brainstem auditory evoked potentials (BAEP) may be normal or abnormal in a comatose patient, depending on the underlying
cause and presence of a brainstem lesion. They may provide prognostic information about patient outcome. Lesions confined to
the cerebral hemispheres are not associated with abnormal BAEPs unless the upper brainstem is functionally disrupted. The
brain-dead person invariably has abnormal BAEPs, with the characteristic finding being bilateral absence of all wave forms or
presence of wave I and absence of waves II-V bilaterally.

References:

Daube JR, editor. Clinical neurophysiology. New York: Oxford University Press, 2002

Daube JR, editor. Clinical neurophysiology. Philadelphia: FA Davis, 1996; page 267, 269

Question 127: Physiology - EEG

Discussion:

Nonconvulsive status can present with altered consciousness or cognition. It can develop in patients with seizure disorder,
occasionally in elderly without a previous history of seizures, after antiepileptic drug or sedative medication withdrawal.

EEG findings in Creutzfeldt-Jakob disease would be repetitive, generalized 1 Hz sharp waves. Hepatic coma presents with
triphasic waves on EEG. Generalized beta activity is seen with benzodiazepine toxicity. Alzheimer disease in early stages has
normal EEG or mild slowing of background activity.

References:

Wyllie E. The treatment of epilepsy, principles and practices. 4th ed. Philadelphia: Lippincott, Williams & Wilkins, 2006.

Question 134: Physiology - EEG

Discussion:

Sleep spindles are rhythmic waves in 2- to 3-second bursts with waxing and waning giving it the spindle shape. They are
bilaterally synchronous and symmetrical or asynchronous with symmetry of total spindle quantity. Infarction in the posterior
cerebral artery can produce reduction of sleep spindles on the ipsilateral side by involving the thalamocortical relay cells.

References:

Blume WT, Kaibara M. Atlas of adult electroencephalography. New York: Raven Press, 1995. page 19, 23, 25, 33, 18

Daube JR, editor. Clinical neurophysiology. New York: Oxford University Press, 2002

Question 139: Physiology - EMG

Discussion:

Hereditary neuropathy with liability to pressure palsies may present in childhood. Other family members may or may not be
symptomatic. Electrophysiologic studies reveal conduction blocks at sites of pressure and mild diffuse slowing.
References:

Jones HR, Bolton CF, Harper CM. Pediatric clinical electromyography. Philadelphia: Lippincott, Wiliams & Wilkins, 1996.

Katirji B, Kaminski HJ, Preston DC, et al. Neuromuscular disorders in clinical practice. Boston: Butterworth-Heinemann, 2002.

Question 145: Physiology - Autonomic Studies

Discussion:

The Valsalva maneuver evaluates the baroreflex arc and its sympathetic and parasympathetic responses. The maneuver consists
of four phases. In phase 4, there is overshoot of the blood pressure due to the persistent increase in peripheral resistance,
normalization of venous return, and stroke volume. The mean blood pressure can increase by more than 10 mm Hg. If the blood
pressure does not increase, this indicates dysfunction of the sympathetic response.

References:

Hilz MJ, Dutsch M. Quantitative studies of autonomic function. Muscle Nerve 2006;33:6-20.

Question 151: Physiology - Sleep

Discussion:

Periodic movements of the legs (more rarely the arms) at intervals of 10 to 90 seconds are one of the causes of disturbed sleep.
Periodic leg movements are only scored if at least 4, about equally spaced, occur in sequence. If the movements cause the sleeper
to awaken they can become a major reason for excessive daytime somnolence. A few periodic limb movements that do not lead
to evidence of arousal are considered normal. Periodic limb movements are often associated with disordered breathing.

References:

Daube JR, editor. Clinical neurophysiology. New York: Oxford University Press, 2002

Daube JR, editor. Clinical neurophysiology. Philadelphia: FA Davis, 1996; page 267, 269

Question 158: Physiology - EMG

Discussion:

Myokymic discharges can be seen as myokymia on the skin surface, are rhythmic and may be seen as singlets or multiplets. They
are sometimes associated with potassium channel antibodies.

References:

Kimura J. Electrodiagnosis in disease of nerve and muscle. 3rd ed. New York: Oxford University Press, 2001.

Question 160: Physiology - EEG

Discussion:

The location of the ictal focus is the most important factor in determining seizure onset during chronic intracranial EEG
monitoring.

References:
Cascino GD. Electroencephalographic recording for epilepsy surgery. In: Daube JR, editor. Clinical neurophysiology.
Philadelphia: FA Davis, 1996;132-140.

Question 161: Physiology - EMG

Discussion:

Decreased number of large motor units, firing rapidly, is consistent with a neurogenic process as in a chronic motor neuron
disorder. Hereditary motor sensory neuropathy (HMSN) would have sensory abnormalities in nerve conduction studies (NCS), as
would chronic inflammatory demyelinating polyneuropathy (CIDP) and tabes dorsalis. Cerebral palsy would give decreased
number of normal size motor unit action potentials (MUAP), firing slowly (decreased activation suggesting a central process).

References:

Preston DC, Shapiro BE. Electromyography and neuromuscular disorders: clinical-electrophysiologic correlations. 2nd ed.
Philadelphia: Elsevier, 2005.

Question 162: Physiology - EEG

Discussion:

Eye opening attenuates alpha activity revealing prominate mu rhythm at about the same frequency (10 Hz) as that of the alpha.
Mu rhythm is seen in C3,4 and Cz locations, occasionally involving P3,4. Movement of contralateral or ipsilateral extremities (or
its contemplation) blocks rhythm. Eye opening itself has no effect on mu rhythm.

References:

Daube JR, editor. Clinical neurophysiology. New York: Oxford University Press, 2002

Blume WT, Kaibara M. Atlas of adult electroencephalography. New York: Raven Press, 1995. page 19, 23, 25, 33, 18

Question 168: Physiology - EMG

Discussion:

F wave responses can be used to assess the entire length of the motor axon and can be absent or delayed in demyelinating
diseases of the peripheral nerve, such as in Guillain Barr頳yndrome.

References:

Kimura J. Electrodiagnosis in disease of nerve and muscle. 3rd ed. New York: Oxford University Press, 2001.

Question 179: Physiology - EMG

Discussion:

The short head of the biceps is supplied by the common peroneal branch off the sciatic nerve in the thigh. The gluteus medius and
tensor fascia lata are supplied by the superior gluteal nerve. The tibialis anterior is supplied by the deep peroneal nerve and the
adductor magnus is supplied by the obturator nerve.

References:

Preston DC, Shapiro BE. Electromyography and neuromuscular disorders: clinical-electrophysiologic correlations. 2nd ed.
Philadelphia: Elsevier, 2005.
Questions 209 - 213: Physiology - EMG

Discussion:

Neuromyotonia is a common feature of Isaac syndrome. This is a high frequency discharge and has a "ping" type sound on the
EMG recording. Myokymic discharges are grouped fasciculations and are common in radiation plexopathy. Conduction block
and/or temporal dispersion are features of acquired demyelinating polyneuropathies (AIDP, CIDP). Increased jitter is associated
with blocking at the neuromuscular junction. Increased fiber density is seen in neurogenic disorders such as SMA or ALS.

References:

Katirji B, Kaminski HJ, Preston DC, et al. Neuromuscular disorders in clinical practice. Boston: Butterworth-Heinemann, 2002.

Question 237: Physiology - EEG

Discussion:

In the absence of a skull defect the side of the lower alpha activity is usually the abnormal one. In this case the left hemispheric
attenuation is due to a large left hemispheric subdural hematoma. Any diffuse process such as metabolic derangements would
have produced bilaterally symmetrical and diffuse abnormalities.

References:

Daube JR, editor. Clinical neurophysiology. New York: Oxford University Press, 2002

Blume WT, Kaibara M. Atlas of adult electroencephalography. New York: Raven Press, 1995. page 19, 23, 25, 33, 18

Question 245: Physiology - EMG

Discussion:

Nerve conduction studies with temporal dispersion, conduction block, and slow conduction are seen in chronic inflammatory
(acquired) demyelinating polyneuropathy. In Charcot-Marie-Tooth disease, there are slow latencies and nerve conduction
velocities, but no temporal dispersion or block. In amyotrophic lateral sclerosis, nerve conduction studies may be normal or with
decreased compound muscle action potential amplitudes. Nerve conduction studies are normal in myasthenia gravis. Toxic
neuropathy, such as with alcoholism, usually presents as small fiber neuropathies.

References:

Katirji B, Kaminski HJ, Preston DC, et al. Neuromuscular disorders in clinical practice. Boston: Butterworth-Heinemann, 2002.

Question 249: Physiology - EEG

Discussion:

Focal polymorphic delta is seen with hemispheric lesions such as infarct or tumor and is nonspecific as to etiology. It generally is
seen with processes involving white matter. Subdural hematoma will typically produce attenuation of EEG amplitude in the
region of the lesion.

References:

Klass DW, Westmoreland BF. Electroencephalography: general principles and adult electroencephalograms. In: Daube JR,
editor. Clinical neurophysiology. Philadelphia: FA Davis, 1996.
Question 250: Physiology - EMG

Discussion:

Small polyphasic motor unit action potentials are seen in patients with myopathies. In a male patient who has large calves along
with many years of progressive proximal muscle weakness in his 20's to 40's, Becker muscular dystrophy is the most likely
diagnosis.

References:

Katirji B, Kaminski HJ, Preston DC, et al. Neuromuscular disorders in clinical practice. Boston: Butterworth-Heinemann, 2002.

Question 251: Physiology - Basic Physiology

Discussion:

Creatine kinase (CK) is elevated in patients with myopathies. In Becker muscular dystrophy, you would expect CK to be in the
thousands. Polymyositis may have an elevated CK but not usually over 1000, and may have an elevated sedimentation rate.

References:

Katirji B, Kaminski HJ, Preston DC, et al. Neuromuscular disorders in clinical practice. Boston: Butterworth-Heinemann, 2002.

Question 255: Physiology - EEG

Discussion:

Lateral eye movements are seen on the bipolar montages which are out-of-phase in derivations involving F7 and F8 electrodes as
an increase in positivity at one is associated with a decrease in positivity in the other. Muscle activity produces very brief
potentials. Movement of the tongue, whose tip is electrically negative with respect to its base, may produce widely distributed,
low frequency intermittent potentials that may resemble "projected rhythms". A burst of muscle potentials may precede such low
frequency waves, serving to differentiate glossokinetic potentials from "projected" activity. Rhythmic delta activity confined to a
single electrode position likely represents pulse artifact. Sequential eye blink artifacts are identifiable by their location at Fp1,
Fp2, their considerably lower amplitude at F3, F4 and their response to eye opening.

References:

Blume WT, Kaibara M. Atlas of adult electroencephalography. New York: Raven Press, 1995. page 19, 23, 25, 33, 18

Daube JR, editor. Clinical neurophysiology. New York: Oxford University Press, 2002

Question 262: Physiology - EMG

Discussion:

Relatively preserved amplitudes in distal segments in conjunction with prolonged latencies and reduced conduction velocities
suggest a primarily demyelinating cause. Temporal dispersion and conduction blocks are hallmarks of an acquired demyelinating
neuropathy and are not seen in congenital demyelinating neuropathies like Charcot-Marie-Tooth (CMT) disease.

References:

Daube JR, editor. Clinical neurophysiology. New York: Oxford University Press, 2002

Daube JR, editor. Clinical neurophysiology. Philadelphia: FA Davis, 1996; page 155
Question 274: Physiology - EEG

Discussion:

Electropositive potentials from occipital leads (O1, 2) may be produced by the R-wave of the ECG in patients. Rhythmic delta
activity confined to a single electrode position likely represents pulse artifact. Pulse artifact may appear as periodic sharply
contoured potentials which are time-locked to ECG. Defects in electrical and mechanical continuity of one electrode can produce
bizarre, often sudden electrode potentials which differ markedly from ongoing background activity, do not blend with other
simultaneously recorded activity, and appear only in derivations involving one electrode. Movement of the tongue, whose tip is
electrically negative with respect to its base, may produce widely distributed, low frequency intermittent potentials that may
resemble "projected rhythms". A burst of muscle potentials may precede such low frequency waves, serving to differentiate
glossokinetic potentials from "projected" activity.

References:

Daube JR, editor. Clinical neurophysiology. New York: Oxford University Press, 2002

Blume WT, Kaibara M. Atlas of adult electroencephalography. New York: Raven Press, 1995. page 19, 23, 25, 33, 18

Question 281: Physiology - EEG

Discussion:

The fully developed triphasic waves in this tracing begin with an initial brief negative wave (upward pen deflection), followed by
a prominent electropositive sharply contoured wave, and then a broader negative wave. The duration of each succeeding wave of
the complex is greater. Most of these waves have no anterior-posterior lag. Triphasic waves are bilaterally synchronous and
usually frontally predominant. In metabolic encephalopathies, they usually appear in groups.

References:

Blume WT, Kaibara M. Atlas of adult electroencephalography. New York: Raven Press, 1995. page 19, 23, 25, 33, 18

Daube JR, editor. Clinical neurophysiology. New York: Oxford University Press, 2002

Question 288: Physiology - EEG

Discussion:

Bilaterally synchronous spikes and spike-waves are seen with disorganized backround. These spikes repeat at a fairly regular
rate. A subacute dementia with abnormal startle response and the above EEG pattern are suggestive of Jakob-Creutzfeldt disease.

References:

Daube JR, editor. Clinical neurophysiology. New York: Oxford University Press, 2002

Blume WT, Kaibara M. Atlas of adult electroencephalography. New York: Raven Press, 1995. page 19, 23, 25, 33, 18

Question 300: Physiology - EMG

Discussion:

Progressive weakness and dry mouth with marked increase in compound muscle action potential amplitude after brief exercise is
consistent with Lambert-Eaton myasthenic syndrome. Calcium channel antibodies block calcium uptake presynaptically and
hinder acetylcholine release. Although a similar electrophysiologic finding can be seen with botulism, the clinical history is not
consistent with this.
References:

Preston DC, Shapiro BE. Electromyography and neuromuscular disorders: clinical-electrophysiologic correlations. 2nd ed.
Philadelphia: Elsevier, 2005.

Brown WF, Bolton CF, editors. Clinical electromyography. 2nd ed. Boston: Butterworth-Heinemann, 1993.

Question 324: Physiology - EEG

Discussion:

Multifocal high amplitude slow waves with irregular spikes and variable rhythm are called hypsarrhythmia and are associated
with infantile spasms. This rhythm is rarely seen beyond the age of 24 months.

References:

Niedermeyer E, Lopes da Silva F. Electroencephalography: basic principles, clinical applications, and related fields. 5th ed.
Philadelphia: Lippincott, Williams and Wilkins, 2004.

Question 345: Physiology - EEG

Discussion:

Fourteen and 6 positive spikes are sharply contoured and occur in the posterior head regions during light sleep. They are best
demonstrated on referential EEG montages and are most common in adolescent patients.

References:

Klass DW, Westmoreland BF. Electroencephalography: general principles and adult electroencephalograms. In: Daube JR,
editor. Clinical neurophysiology. Philadelphia: FA Davis, 1996.

Question 356: Physiology - EEG

Discussion:

Lennox-Gastaut syndrome is characterized clinically by frequent generalized seizures of mixed type in association with mental
retardation and a slow spike-and-wave EEG pattern with increasing disorganization during sleep.

References:

Niedermeyer E, Lopes da Silva F. Electroencephalography. 4th ed. Baltimore: Lippincott Williams & Wilkins, 1998.

Question 362: Physiology - EMG

Discussion:

Fasciculation potentials result from spontaneous discharges of a whole or possibly part of a motor unit. The generator source of
nearly all fasciculations has a motor axonal origin. Fasciculation potentials, although typically associated with diseases of
anterior horn cells, are also seen in radiculopathy, entrapment neuropathy, and the muscular pain-fasciculation syndrome.

References:

Preston DC, Shapiro BE. Electromyography and neuromuscular disorders: clinical-electrophysiologic correlations. 2nd ed.
Philadelphia: Elsevier, 2005.
Kimura J. Electrodiagnosis in disease of nerve and muscle: prinicples and practice. 2nd ed. Philadelphia: Davis, 1989

Question 366: Physiology - EMG

Discussion:

Footdrop is caused by either a peroneal neuropathy or L5 radiculopathy. With peroneal neuropathy the foot eversion is affected
and with L5 radiculopathy, foot inversion is affected. Fibrillation potentials may occur in muscle fibers that have lost their
innervation, have been sectioned transversely or divided longitudinally, are regenerating and have never been innervated.
Fasciculation potentials may occur in normal persons but are commonly seen in chronic neurogenic disorders. A femoral
neuropathy causes quadricep weakness with sparing of the adductors and iliopsoas muscles, absent or decreased knee jerk, and
loss of sensation of the anteromedial thigh and medial lower leg.

References:

Daube JR, editor. Clinical neurophysiology. Philadelphia: FA Davis, 1996; page 267, 269

Preston DC, Shapiro BE. Electromyography and neuromuscular disorders: clinical-electrophysiologic correlations. 2nd ed.
Philadelphia: Elsevier, 2005.

Dyck PJ, Thomas PK. Peripheral neuropathy. 4th ed. Philadelphia: Saunders, 2005.

Question 369: Physiology - Basic Physiology

Discussion:

Pacinian corpuscles are responsible for perception of vibration.

References:

Kandel ER, Schwartz JH, Jessel TM. Principles of neural science. 4th ed. New York: McGraw-Hill, 2000.

Question 383: Physiology - Sleep

Discussion:

REM sleep behavior disorder is seen in older patients and is characterized by REM sleep without muscle atonia and various
abnormal motor activities during sleep.

References:

Bradley WG, Daroff RB, Fenichel GM, et al, editors. Neurology in clinical practice.. 4th ed. New York: Butterworth-Heinemann,
2004

Question 387: Physiology - Evoked Potentials

Discussion:

In somatosensory evoked potentials of the median nerve, the scalp recorded P14 is generated in the brainstem.

References:

Chiappa KH. Evoked potentials in clinical medicine. 3rd ed. New York: Lippincott-Raven, 1997.
Question 389: Physiology - EMG

Discussion:

First dorsal interosseus and abductor digiti minimi muscles are innervated by the ulnar nerve and C8 and T1 nerve roots. Sensory
studies aid in the localization of predorsal or post dorsal root ganglion regions. Lesions proximal to dorsal root ganglia, like
radiculopathy, have normal sensory nerve action potentials. Sensory nerve action potentials are abnormal in lesions distal to
dorsal root ganglia. Fasciculation potentials may occur in normal persons but are commonly seen in chronic neurogenic
disorders.

References:

Kimura J. Electrodiagnosis in disease of nerve and muscle. 3rd ed. New York: Oxford University Press, 2001.

Daube JR. Assessing the motor unit with needle electromyography. In: Daube JR, editor. Clinical neurophysiology. 2nd ed. New
York: Oxford University Press, 2002.

Question 390: Physiology - EEG

Discussion:

Generalized beta activity (activity over 14 Hz) can be seen as a consequence of drug use, particularly benzodiazepines or
barbiturates.

References:

Niedermeyer E, Lopes da Silva F, editors. Electroencephalography: basic principles, clinical applications, and related fields. 4th
ed. Baltimore: Lippincott Williams & Wilkins, 1998.

Question 402: Physiology - EMG

Discussion:

A progressive increase in latency, duration, amplitude, and area of motor and sensory nerve action potentials accompanies a
physiological decline in temperature. This is reversed by warming.

References:

Brown WF, Bolton CF, Aminoff MJ, editors. Neuromuscular function and disease: basic, clinical, and electrodiagnostic aspects.
Philadelphia: WB Saunders, 2002.

Question 409: Physiology - EEG

Discussion:

Periodic lateralized epileptiform discharges (PLEDs) consist of focal or lateralized epileptiform discharges recurring every 1 to 4
seconds. They are seen after focal vascular insults, tumors, abscess, head trauma, or herpes simplex encephalitis. Burst
suppression pattern is seen with barbiturate toxicity, after anesthesia, post cardiac arrest, and with hypothermia. Triphasic waves
are typically seen in metabolic encephalopathy.

References:

Wyllie E. The treatment of epilepsy, principles and practices. 4th ed. Philadelphia: Lippincott, Williams & Wilkins, 2006.
Question 414: Physiology - EEG

Discussion:

The EEG can provide confirmatory evidence of brain death, which is manifested by an absence of spontaneous or induced
electrical activity of cerebral origin. Electrocerebral inactivity (ECI) is defined as "no EEG activity over 2 microvolts." There are
important minimal technical criteria for recording in patients with suspected cerebral death, one of which is a minimum of 8
recording scalp electrodes.

References:

Daube JR, editor. Clinical neurophysiology. New York: Oxford University Press, 2002

Daube JR, editor. Clinical neurophysiology. Philadelphia: FA Davis, 1996; page 267, 269

Question 415: Physiology - EEG

Discussion:

Small sharp spikes, also known as benign epileptiform transients of sleep, are benign sporadic sleep spikes occurring mainly in
adults during drowsiness and light sleep. They are usually low voltage, short duration, diphasic spikes with steep descending
limb. They do not distort the background and are not associated with rhythmic slow wave activity. They have no significance for
the diagnosis of epileptic seizures.

References:

Daube JR, editor. Clinical neurophysiology. Philadelphia: FA Davis, 1996; page 267, 269

Daube JR, editor. Clinical neurophysiology. New York: Oxford University Press, 2002

Question 436: Physiology - EEG

Discussion:

Light sensitivity manifested by photomyoclonus induced by photic stimulation can occur with abrupt alcohol withdrawal.

References:

Niedermeyer E, Lopes da Silva F. Electroencephalography. 4th ed. Baltimore: Lippincott, Williams & Wilkins, 1998.

Question 438: Physiology - EMG

Discussion:

Increased jitter is seen in a variety of neuromuscular disorders and is not specific to any particular disease. Jitter is not increased
in metabolic myopathies and steroid myopathies.

References:

Stalberg E, Trontelj JV. Single fiber electromyography. 2nd ed. New York: Raven Press, 1994.

Question 442: Physiology - EEG

Discussion:
Long-term use of chlorpromazine has been reported to lead to generalized paroxysmal bursts in EEGs.

References:

Engel J, Pedley TA, editors. Epilepsy: a comprehensive textbook. Philadelphia: Lippincott-Raven, 1998.

Question 443: Physiology - Sleep

Discussion:

In the presence of a history that is suggestive of narcolepsy, a finding of excessive daytime somnolence (mean MSLT sleep
latency of 5 minutes or less) in combination with at least 2 sleep onset REM periods during the 5 naps in the MSLT suggest
narcolepsy.

References:

Daube JR, editor. Clinical neurophysiology. New York: Oxford University Press, 2002

Daube JR, editor. Clinical neurophysiology. Philadelphia: FA Davis, 1996; page 267, 269

Question 444: Physiology - EMG

Discussion:

Inclusion body myositis is an inflammatory myopathy with common clinical presentation of distal upper extremity and proximal
lower extremity weakness. It occurs more frequently in older patients. EMG shows diffuse irritability and fibrillation potentials,
as in amyotrophic lateral sclerosis, but the motor unit action potentials are small and polyphasic.

References:

Day JW, Ranum LPW. Myotonic disorders. In: Katirji B, Kaminski HJ, Preston DC, et al, editors. Neuromuscular disorders in
clinical practice. Woburn, Mass: Butterworth-Heinemann, 2002;1078-1091.

You might also like